Multidisciplinary Self Assessment Examination Flashcards

1
Q

Clinical features of the Brown-Sequard syndrome include
all of the following EXCEPT?
A. Contralateral loss of pain and temperature sensation
beginning one to two spinal segments below the lesion
B. Ipsilateral loss of proprioception and vibratory sense
below the level of the lesion
C. Ipsilateral Horner’s syndrome if the lesion is cervical
D. Ipsilateral loss of crude touch below the level of the lesion
E. Ipsilateral loss of sweating below the level of the lesion

A

D. Ipsilateral loss of crude touch below the level of the lesion

How well did you know this?
1
Not at all
2
3
4
5
Perfectly
2
Q

What is depicted in the photomicrograph below (Figure 8.2Q)?
A. Palisading cells around a necrotic region in a patient
with glioblastoma
B. Spongiform change in a patient with prion disease
C. Homer-Wright rosette in a 3-year-old male with
medulloblastoma
D. Acute infarct in a patient with myoclonic epilepsy with
ragged red fibers (MERRF)
E. Fibrinoid necrosis in a patient with acute hemorrhagic
leulwencephalopathy

A

**A. Palisading cells around a necrotic region in a patient
with glioblastoma **

Note the “picket fence” arrangement (pseudopalisading) of the nuclei surrounding a region of necrosis in this
photomicrograph, which depicts a glioblastoma (Ellison,
pp.628- 631).

How well did you know this?
1
Not at all
2
3
4
5
Perfectly
3
Q

What is the most sensitive laboratory test for the detection of neurocysticercosis (NCC)?
A. Peripheral eosinophil count
B. Complete serum white blood cell count
C. Stool for ova and parasites
D. Enzyme-linked immunosorbent assay (ELISA)
E. Electroimmunotransfer blot (EITB)

A

E. Electroimmunotransfer blot (EITB)

Complete white blood cell count, peripheral eosinophil
level, and serum anticysticercal antibody levels should be
obtained in all patients suspected of having NCC. Patients
requiring ventriculostomy placement should have cerebrospinal fluid (CSF) analyzed for eosinophil and anticysticercal antibody levels. Stool testing for ova and parasites is
helpful in patients with simultaneous intestinal tapeworm
infection but is insensitive and nonspecific for T. SOhll:1ll
species and is found in less than 33% of cases. Severallaboratory methods have been developed to detect host antibodies
against circulating cysticercal antigens. From the many
tests performed , current data indicate that enzyme-linked
immunosorbent assay (ELISA) and electroimmunotransfer
blot (EITB) tests are the most effective. Studies comparing
these diagnostic modalities have shown that the EITB assay
is more sensitive overall than ELISA, especially when serum
is being tested. Both techniques are more sensitive in cases
with multiple cysts than in cases with solitary or confined
lesions. Additionally, no globa l difference among cases
was found with parasites located in different compartments
(ventricles, subarachnoid space, parenchyma) of the central nervous system (Greenberg, pp. 236- 238; Proano-Narvaez
et aI., p. 2118)

How well did you know this?
1
Not at all
2
3
4
5
Perfectly
4
Q

Match each of the following spinal cord lesions
with the appropriate clinical syndrome (Figure 8.4-8.9Q),
using each answer once, more than once, or not at all.

Early subacute combined degeneration

A

E

How well did you know this?
1
Not at all
2
3
4
5
Perfectly
5
Q

Match each of the following spinal cord lesions
with the appropriate clinical syndrome (Figure 8.4-8.9Q),
using each answer once, more than once, or not at all.

Syringomyelia

A

F

How well did you know this?
1
Not at all
2
3
4
5
Perfectly
6
Q

Match each of the following spinal cord lesions
with the appropriate clinical syndrome (Figure 8.4-8.9Q),
using each answer once, more than once, or not at all.

Tabes dorsalis

A

D

How well did you know this?
1
Not at all
2
3
4
5
Perfectly
7
Q

Match each of the following spinal cord lesions
with the appropriate clinical syndrome (Figure 8.4-8.9Q),
using each answer once, more than once, or not at all.

PoliomyelitiS

A

A

How well did you know this?
1
Not at all
2
3
4
5
Perfectly
8
Q

Match each of the following spinal cord lesions
with the appropriate clinical syndrome (Figure 8.4-8.9Q),
using each answer once, more than once, or not at all.

Amyotrophic lateral sclerosis

A

C

How well did you know this?
1
Not at all
2
3
4
5
Perfectly
9
Q

Match each of the following spinal cord lesions
with the appropriate clinical syndrome (Figure 8.4-8.9Q),
using each answer once, more than once, or not at all.

Familial spastic paraplegia

A

B

How well did you know this?
1
Not at all
2
3
4
5
Perfectly
10
Q

\Vhich of the following tumors may share certain
histopathologic features with the lesion depicted below
(Figure S.10Q) ?
1. Clear cell ependymoma
2. Central neurocytoma
3. Dysembryoplastic neuroepithelial tumor
4. Fibrous meningioma

A. 1,2, and3
B. 1 and3
C. 2 and 4
D. Only 4 is correct
E. All of the above

A

**A. 1,2, and3 **

The differential diagnosis of oligodendroglia I tllI110rS
includes clear cell ependymoma, central neurocytoma, and
dysembryoplastic neuroepithelial tumor. All of these entities
exhibit the presence of neoplastic cells with a uniform round
nucleus and clear cytoplasm. A rare differential diagnosis
of oligodendroglioma is clear cell meningioma (not fibrous
meningioma), which can be differentiated from oligodendroglioma by abundant diastase-sensitive PAS positivity
and immunoreactivity for EMA. Note the prominent calcification , “chicken wire” capillaries (prominent branching),
“fried egg” cells with round monomorphic nuclei, and perinuclear halos arranged in a back-to-back fashion in this
photomicrograph depicting an oligodendroglioma (Ellison,
pp. 641- 644; WHO, p. 59).

How well did you know this?
1
Not at all
2
3
4
5
Perfectly
11
Q

Which of the following abdominal wall layers will best
hold suture (highest tensile strength) during placement of a
ventriculoperitoneal shunt?

A. Colles fascia
B. Cruveilhier’s fascia
C. Buck’s fascia
**D. Scarpa’s fascia **
E. Camper’s fascia

A

**D. Scarpa’s fascia **

The anterior abdominal wall consists of the epidermis, superficial layer of superficial fascia (of Camper), the
deep layer of superficial fascia (of Scarpa), the deep fascia
(investing fascia of musculature), the external and internal
oblique muscles, the transverse abclominis muscle , transversalis fascia, loose extraperitoneal connective tissue , and
peritoneum. Camper’s fascia is predominately an adipose layer that contains most of the fat of the subdermis. It continues over the pubis as the superficial layer (of Cruveilhier) of
the superficial perineal faSCia, crosses the inguinal ligament
to merge with the superficial fascia of the thigh, and continues over the chest as the superficial layer of superficial
thoracic fascia. Scarpa’s fascia is a fibrous layer that will best
hold sutures (highest tensile strength). It continues over the
pubis as the deep layer of superficial perineal fascia (of
Colles) and passes into the upper thigh, where it attaches to
the fascia lata. The deep fascia is the investing fascia of the
musculature, aponeuroses, and large neurovascular structures and is not easily separated from the underlying epimysium of muscle. It extends into the penis as Buck’s faSCia,
continues over the spermatic cord as the externa l spermatic
faSCia, and passes over the pubis and perineal musculature as
the deep perineal fascia of Gallaudet (April, p. 173)

How well did you know this?
1
Not at all
2
3
4
5
Perfectly
12
Q

What is the diagnosis ?
A. Fatty filum with tethered cord
B. Myxopapillaryependymoma
C. Dermal sinus tract
D. Epidural hematoma
E. Dermoid tumor

A

**A. Fatty filum with tethered cord **

Note the cord tethering and fatty filum on this sagittal
MRl (Ramsey, pp. 104- 106).

How well did you know this?
1
Not at all
2
3
4
5
Perfectly
13
Q

Which of the following is correct about the lesion
depicted on the angiogram below (Figure 8.13Q)?
A. Annual risk of bleeding is approximately 3% per year
B. Associated with cranial bruit and congestive heart
failure during the neonatal period
C. The loss of a tumor suppressor gene on chromosome 22
**D. Tills lesion is usually found within normal brain
parenchyma **
E. Represents an extreme anatomic variant of cortical
arterial blood supply

A

**D. Tills lesion is usually found within normal brain
parenchyma **

This angiogram depicts the classic “caput medusae”
pattern of a venous angioma , which is an extreme anatomic
variant of medullary (white matter) venous drainage. The
precise etiology of this lesion remains unclear, although
some authors have proposed that it results from arrested
development of parts of the venous vasculature at a time
when normal arterial development is nearly complete. This
results in the retention of primitive venous channels that
typically empty into a single large draining vein (Osborn,
pp.294- 295).

How well did you know this?
1
Not at all
2
3
4
5
Perfectly
14
Q

Which of the following structures are connected by the stria medullaris thalami ?
A. Nucleus basalis (of Meynert) and septal nuclei
B. Septal nuclei and habenular nuclei
C. Habenular nuclei and occipital cortex
D. Septal nuclei and anterior thalamic nuclei
E. Pineal gland and anterior commissure

A

**B. Septal nuclei and habenular nuclei **

The stria medullaris thalami contains projections
that originate in the septal nuclei, anterior thalamic nuclei,
and hypothalamus (preoptic region) and terminate in the
habenular nuclei. The habenular nuclei then project to the
raphe nuclei of the midbrain via the fasciculus retroflexus.
In this manner, the stria medullaris thalami act as a relay
point for limbic system information that is transmitted to the
midbrain (Carpenter, p. 252; Martin, p. 473).

How well did you know this?
1
Not at all
2
3
4
5
Perfectly
15
Q

Which retinal cell provides a mechanism for mediating
opposite responses in adjacen t groups of photoreceptor cells
that is used to enhance contrast between objects?
A. Plexiform cells
B. Amacrine
C. Horizontal cells
D. Ganglion
E. Bipolar cells

A

C. Horizontal cells

Visual information flows vertically from photoreceptor cells (outer nuclear layer) to bipolar cells (inner nuclear
layer) to ganglion cells (ganglion cell layer) as well as laterally via horizontal cells (outer plexiform layer) and amacrine
cells (inner plexiform layer). Light produces opposite effects
on the rate of bipolar cell firing depending on whether it stimulates the center or surrounding part of the cell’s receptive
field. Additionally, a lateral network of horizontal cells that
directly interconnect neighboring groups of photoreceptor
cells helps mediate this antagonist property. Hence, horizontal cells provide a mechanism for mediating opposite
responses in adjacent photoreceptor cells, which is used to
enhance luminance contrast. The precise role of amacrine
cells remains unclear, although some amacrine cells function like horizontal cells. They mediate antagonistic inputs
between bipolar cells and ganglion cells in the inner plexiform layer. Other amacrine cells have been implicated in
shaping the complex receptive field properties of various
types of ganglion cells, such as M-type cells that process
orientation information (Pritchard, pp. 292- 302; Kandel,
p.515).

How well did you know this?
1
Not at all
2
3
4
5
Perfectly
16
Q

What deficit may result from damage to Exner’s area?
A. Alexia
B. Aphasia
C. Agraphia
D. Anosmia
E. Apathy

A

C. Agraphia

How well did you know this?
1
Not at all
2
3
4
5
Perfectly
17
Q

Match the following structures with the appropriate letterhead on the following axial CT scans (Figures 8.17-
8.24Q a, b, c) of the right petrous temporal bone, using each
answer only once

Vestibule

A

F

How well did you know this?
1
Not at all
2
3
4
5
Perfectly
18
Q

Match the following structures with the appropriate letterhead on the following axial CT scans (Figures 8.17-
8.24Q a, b, c) of the right petrous temporal bone, using each
answer only once

Cochlea

A

H

How well did you know this?
1
Not at all
2
3
4
5
Perfectly
19
Q

Match the following structures with the appropriate letterhead on the following axial CT scans (Figures 8.17-
8.24Q a, b, c) of the right petrous temporal bone, using each
answer only once

Posterior semicircular canal

A

E

How well did you know this?
1
Not at all
2
3
4
5
Perfectly
20
Q

Match the following structures with the appropriate letterhead on the following axial CT scans (Figures 8.17-
8.24Q a, b, c) of the right petrous temporal bone, using each
answer only once

Lateral semicircular cana

A

A

How well did you know this?
1
Not at all
2
3
4
5
Perfectly
21
Q

Match the following structures with the appropriate letterhead on the following axial CT scans (Figures 8.17-
8.24Q a, b, c) of the right petrous temporal bone, using each
answer only once

Vestibular aqueduct

A

D

How well did you know this?
1
Not at all
2
3
4
5
Perfectly
22
Q

Match the following structures with the appropriate letterhead on the following axial CT scans (Figures 8.17-
8.24Q a, b, c) of the right petrous temporal bone, using each
answer only once

Facial nerve

A

B

How well did you know this?
1
Not at all
2
3
4
5
Perfectly
23
Q

Match the following structures with the appropriate letterhead on the following axial CT scans (Figures 8.17-
8.24Q a, b, c) of the right petrous temporal bone, using each
answer only once

Superior semicircular canal

A

B

How well did you know this?
1
Not at all
2
3
4
5
Perfectly
24
Q

Match the following structures with the appropriate letterhead on the following axial CT scans (Figures 8.17-
8.24Q a, b, c) of the right petrous temporal bone, using each
answer only once

Endolymphatic duct

A

I

How well did you know this?
1
Not at all
2
3
4
5
Perfectly
25
Q

Which of the following is a flow-regulated valve?
A. Orbis-Sigma valve
B. PS Medical Delta valve
C. Cordis horizontal-vertical valve
D. Codman Hakim programmable valve
E. Holter-Hausner valve

A

**A. Orbis-Sigma valve **

How well did you know this?
1
Not at all
2
3
4
5
Perfectly
26
Q

What is the diagnosis ?
A. Porencephaly
B. Cortical dysplasia
C. Open-lip schizencephaly
D. Arachnoid cyst
E. Closed-lip schizencephaly

A

**C. Open-lip schizencephaly **

How well did you know this?
1
Not at all
2
3
4
5
Perfectly
27
Q

This abnormality is believed to result from failure of
what embryologic stage of development?
A. Primary neurulation
B. Secondary neurulation
C. Disjunction
**D. Cellular migration **
E. Myelination

A

**D. Cellular migration **

How well did you know this?
1
Not at all
2
3
4
5
Perfectly
28
Q

All of the following are derived from a common precursor
EXCEPT?
A. ACTH
B. ~I’lelanocyte-stimulating hormone
C. Beta lipotropin
D. Beta endorphin
E. Leucine-enkephalin

A

E. Leucine-enkephalin

How well did you know this?
1
Not at all
2
3
4
5
Perfectly
29
Q

A 62-year-old female undergoes uncomplicated transsphenoidal resection of a pituitary macroadenoma and is recovering in the intensive care unit. Postoperatively, she
develops increased thirst, nausea, elevated urine output (> 300
mL for 3 consecutive hours), hyperna tremia (149 mEq/L),
and a serum osmolarity of 323 mEq/L. At this point, optimal
trea tment for this patient should include what?
A. Fludrocortisone acetate
B. Urea
C. Oral desmopressin acetate (DDAVP)
D. r.rgilline vasopressin (aqueous Pitressin) intravenously
E. Pitressin in tannic oil suspension intramuscularly

A

D. r.rgilline vasopressin (aqueous Pitressin) intravenously

How well did you know this?
1
Not at all
2
3
4
5
Perfectly
30
Q

What is depicted on the EEG below (Figure S.30Q)?
A. Absence seizure
B. Left temporal lobe spike-and-wave discharges
C. Alpha rhythm
D. Theta rhythm
E. K complexes

A

**C. Alpha rhythm **

How well did you know this?
1
Not at all
2
3
4
5
Perfectly
31
Q

A 54-year-old female awakens from surgery for an
elective right ophthalmic artery aneurysm clipping with
complete right eye blindness and no other neurologic deficit.
A cerebral angiogram reveals incorporation of the ophthalmic
artery origin into the clip construct. What other finding(s)
may be present on the angiogram?
A. Occlusion of the right internal carotid artery with
inadequate posterior or anterior communicating
artery collaterals
B. Vasospasm of the right internal carotid artery
C. Poor collateral filling of the right globe from the
maxillary and facial arteries
D. Inadequate ascending pharyngeal artery collateral flow
to the right globe
E. All of the above

A

**C. Poor collateral filling of the right globe from the
maxillary and facial arteries **

How well did you know this?
1
Not at all
2
3
4
5
Perfectly
32
Q

A 28-year-old obese male presents with a 2-montl1 history of headaches and diplopia. He is found to harbor the
lesion depicted in the photomicrograph below (Figure 8.32-
8.33Q). What should be the next course of treatment after
surgical resection of this lesion?

A. Whole-brain radiation therapy
B. Radiosurgery
C. Chemotherapy
D. Observation and serial MRI
E. Proton-beam radiotherapy

A

D. Observation and serial MRI

How well did you know this?
1
Not at all
2
3
4
5
Perfectly
33
Q

What does the arrow in this photomicrograph depict?
A. Capillary telangiectasia
B. Gemistocytes
C. Rosenthal fibers
D. Normal blood vessels
E. Melanin granules

A

**C. Rosenthal fibers **

How well did you know this?
1
Not at all
2
3
4
5
Perfectly
34
Q

An infant is able to transfer objects from hand to hand,
bear some of his weight, lift his head off the table prior to
being pulled up, and turn his head to voice. ‘'’hat is the
approximate age of this child?
A. 2 months
B. 4 months
C. 6 months
D. 8 months
E. 10 months

A

C. 6 months

How well did you know this?
1
Not at all
2
3
4
5
Perfectly
35
Q

All of the following reflexes generally disappear by 4 to
6 months of age EXCEPT?
A. Suck
B. Palmar grasp
C. Tonic neck
D. Ventral suspension (Landau)
E. Placing/stepping

A

**D. Ventral suspension (Landau) **

How well did you know this?
1
Not at all
2
3
4
5
Perfectly
36
Q

What is the earliest visual field cut experienced by
patients with ophthalmic artery aneurysms?
A. Monocular inferior temporal quadrantanopsia
B. Monocular superior temporal quadrantanopsia
C. lvlonocular superior nasal quadrantanopsia
D. Binocular inferior nasal quadrantanopsia
E. Binocular temporal hemianopsia

A

**D. Binocular inferior nasal quadrantanopsia **

How well did you know this?
1
Not at all
2
3
4
5
Perfectly
37
Q

Which of the following structures are usually either
drilled or sectioned during surgical exposure of large
ophthalmic artery aneurysms?
1. Falciform ligament
2. Distal dural ring
3. Anterior clinoid process
4. Optic strut

A. 1,2, and 3 are correct
B. 1 and 3 are correct
C. 2 and 4 are correct
D. Only 4 is correct
E. All of the above

A

**E. All of the above **

How well did you know this?
1
Not at all
2
3
4
5
Perfectly
38
Q

A 42-year-old right-handed male presents to an emergency department with seizures. I-lis CT and MRI studies
show a fairly well circumscribed, heterogenously enhancing
right frontal lesion with patchy calcification and surrounding
edema suggestive of an oligodendroglioma. All of the following are true about this tumor EXCEPT?
A. The polypeptide glial fibrillary acidic protein (GFAP) is
not expressed by oligodendrocytes
B. They account for approximately 5% of all primary
intracranial neoplasms
C. Identifying the oligodendroglial component on frozen
section is usually facilitated by the classic “fried egg”
appearance of the perinuclear halo
D. The greater the degree of anaplasia, the shorter the
survival
E. There is a strong association between response to
PCV (procarbazine, CCNU, and vincristine) chemotherapy and allelic loss on 1p/19q in anaplastic
oligodendrogliomas

A

**C. Identifying the oligodendroglial component on frozen
section is usually facilitated by the classic “fried egg”
appearance of the perinuclear halo **

How well did you know this?
1
Not at all
2
3
4
5
Perfectly
39
Q

Which of the following structures contains second-order
neurons of the spinocerebellar tracts ?
A. Clarke’s nucleus
B. Nucleus gracilis/cuneatus
C. Accessory cuneate nucleus
D. Inferior olives
E. Both A and C

A

**E. Both A and C **

How well did you know this?
1
Not at all
2
3
4
5
Perfectly
40
Q

Where is the cortical representation of macular vision?
A. Occipital poles
B. Lower bank of the calcarine sulcus
C. Temporoparieto-occipital junction
D. Precuneus
E. Superior bank of calcarine sulcus

A

**A. Occipital poles **

How well did you know this?
1
Not at all
2
3
4
5
Perfectly
41
Q

What is the first site of binaural convergence within the
auditory pathway?
A. Dorsal cochlear nuclei
B. Laterallemniscus
C. Superior olive
D. ivredial geniculate bodies
E. Inferior colliculi

A

A. Dorsal cochlear nuclei

The superior olivary nuclei receive the ventral acoustic striae and contain third-order auditory neurons that subsequently project to the contralateral lateral lemniscus. The
superior olives are the initial sites of binaural convergence
within the auditory pathway (Kandel, pp. 606- 608)

How well did you know this?
1
Not at all
2
3
4
5
Perfectly
42
Q

A 42-year-old male underwent a therapeutic interventional neuroradiologic procedure. What is depicted on his
angiogram below (Figure 8.42Q)?
A. Intraprocedural aneurysmal rupture
B. Poor distal middle cerebral artery perfusion
C. An enlarged tentorial artery supplying a lateral pontine
arterial-venous malformation
D. A dural arterial-venous fistula
E. Type II carotid-cavernous fistula

A

**A. Intraprocedural aneurysmal rupture **

The angiogram depicts a right posterior carotid wall
aneurysm as well as extravasation of contrast dye from
the aneurysm in a patient about to be treated with GDC
embolization (microcatheter evident in internal carotid
artery). Intraprocedural aneurysmal rupture is reported to
occur in 2 to 8% of patients treated with GDC embolization.
It seems to be more prevalent during treatment of smaller
aneurysms, especially in the acute phase following SAl-I. It
may occur during several phases of the embolization procedure. When the microcatheter is responsible for the rupture,
it is important to avoid withdrawing the device prematurely,
as the offending device often plugs the ruptured site and
prevents additional extravasation of blood. Similarly, if the
aneurysm ruptures during the coiling phase, it is important
that the clinician deploy the coil in an attempt to seal
the leak. In general, once rupture occurs, the remaining
aneurysmal sac should be packed as quickly as possible. In
refractory cases, temporary or permanent balloon occlusion
of the parent vessel or immediate surgical clipping may be
warranted (Youmans, pp. 2071-2072).

How well did you know this?
1
Not at all
2
3
4
5
Perfectly
43
Q

What is depicted on the ECG below (Figure 8.43Q)
A. Myocardial infarction
B. Hyperkalemia
C. Torsades de pointes
D. Digoxin toxicity
E. Atrial flutter

A

**A. Myocardial infarction **

Note the prominent ST-segment elevation in leads VI
through V” on this ECG, depicting an anterior wallmyocardial infarction. In general, ST-segment and T-wave changes
appear over the first minutes to hours of an infarction, and Q
waves appear over hours to days. An evolving myocardial
infarction may first manifest with peaked ‘1’ waves followed
by ST segment elevation and T-wave inversion. Eventually Q
waves may appear. In a large anterior wall infarction, these
changes are most apparent in leads VI through V(“ while in
an inferior infarction, these changes often occur in leads II,
III, and aVF. Of note, if a patient’s T waves are chronically
inverted> the peaking may make them appear normal-a
process referred to as pseudonormalization. T waves are
the least reliable of ST- and T-wave segment abnormalities
because many noncardiac events may influence them (i.e.,
elevated IC+) . Dying myocardial cells release their enzymes
into the bloodstream, and the increased concentration
should be confirmed in the peripheral blood (Fishman,
pp . 9 - 24; Marino, pp. 301-313).

How well did you know this?
1
Not at all
2
3
4
5
Perfectly
44
Q

A 16-year-old male with the MRl depicted
below (Figure 8.44-8.45Q) is referred to your office for
surgical evaluation. His laboratory studies reveal that he has
hypothyroidism, cortisol deficiency, and a prolactin level of
69. I-lis family states that they have noted behavior changes
and a recent increase in his weight. I-Ie has no vision with the
left eye and a dense temporal field cut of the right eye.

What is the most likely diagnosis?
A. Pituitary macroadenoma
B. Metastatic tumor invading the posterior pituitary gland
C. Craniopharyngioma
D. Sphenoid sinusitis
E. Invasive mucocele of the sphenoid sinus

A

**C. Craniopharyngioma **

The clinical history and MRl are most consistent
with a cystic craniopharyngioma. The modestly elevated
prolactin level is likely the result of the “stalk effect,”
whereby injury of the hypothalamus or pituitary stalk (i.e.,
from large tumors) results in modest elevations of prolactin
from reduced prolactin inhibitory factor levels (dopamine).
As a general rule, prolactin levels> 150 ng/mL are rarely
secondary to a stalk effect, whereas levels < 90 usually suggest a stalk injury. Large components of this tumor
extend inferiorly into the sphenoid sinus and superiorly into
the suprasellar space. Moreover, the optic chiasm appears
draped over the rostral margin of the tumor. Although these
are worrisome findings that warrant special concern, they
are not uncommon with craniopharyngioma. This sagittal
MRl shows that the posterior component of this tumor has
eroded through a significant portion of the tuberculum sella
and clivus, a relatively rare but significant finding. This latter
detail is especially concerning, since failure to recognize this
degree of bony erosion on preoperative MRl may result
in inadvertent injury to major posterior fossa structures
(basilar artery, perforating vessels) during transsphenoidal
tumor resection. Thyroxin (‘1’4) is generally preferred over
thyroid extract (‘1’.1) because blood levels are often more predictable. This is especially true for patients with concomitant
liver injury, as ‘1’.1 is converted to T4 in the liver. Patients with
ci’rrhosis may remain hypothyroid even while taking T..
Although it is preferable to correct hypothyroidism preoperatively, it is important to correct any cortisol deficiency as
well, as premature thyroid replacement can precipitate an
adrenal crisis in this group of patients. Thyroxine has been
shown to decrease phenytoin levels (Committee on Education
in Neurological Surgery, p. 102; Greenberg, pp. 419- 436).

How well did you know this?
1
Not at all
2
3
4
5
Perfectly
45
Q

A 16-year-old male with the MRl depicted
below (Figure 8.44-8.45Q) is referred to your office for
surgical evaluation. His laboratory studies reveal that he has
hypothyroidism, cortisol deficiency, and a prolactin level of
69. I-lis family states that they have noted behavior changes
and a recent increase in his weight. I-Ie has no vision with the
left eye and a dense temporal field cut of the right eye.

The prolactin level is elevated most likely secondary to what process?
A. Hook effect
B. Stalk effect
C. Avengaard effect
D. Tumor secretion
E. Prolactin-secreting lung nodule

A

**B. Stalk effect **

The clinical history and MRl are most consistent
with a cystic craniopharyngioma. The modestly elevated
prolactin level is likely the result of the “stalk effect,”
whereby injury of the hypothalamus or pituitary stalk (i.e.,
from large tumors) results in modest elevations of prolactin
from reduced prolactin inhibitory factor levels (dopamine).
As a general rule, prolactin levels> 150 ng/mL are rarely
secondary to a stalk effect, whereas levels < 90 usually suggest a stalk injury. Large components of this tumor
extend inferiorly into the sphenoid sinus and superiorly into
the suprasellar space. Moreover, the optic chiasm appears
draped over the rostral margin of the tumor. Although these
are worrisome findings that warrant special concern, they
are not uncommon with craniopharyngioma. This sagittal
MRl shows that the posterior component of this tumor has
eroded through a significant portion of the tuberculum sella
and clivus, a relatively rare but significant finding. This latter
detail is especially concerning, since failure to recognize this
degree of bony erosion on preoperative MRl may result
in inadvertent injury to major posterior fossa structures
(basilar artery, perforating vessels) during transsphenoidal
tumor resection. Thyroxin (‘1’4) is generally preferred over
thyroid extract (‘1’.1) because blood levels are often more predictable. This is especially true for patients with concomitant
liver injury, as ‘1’.1 is converted to T4 in the liver. Patients with
ci’rrhosis may remain hypothyroid even while taking T..
Although it is preferable to correct hypothyroidism preoperatively, it is important to correct any cortisol deficiency as
well, as premature thyroid replacement can precipitate an
adrenal crisis in this group of patients. Thyroxine has been
shown to decrease phenytoin levels (Committee on Education
in Neurological Surgery, p. 102; Greenberg, pp. 419- 436).

How well did you know this?
1
Not at all
2
3
4
5
Perfectly
46
Q

All of the following are typically associated with Behget’s
syndrome EXCEPT?
A. Uveitis
B. Genital ulcers
C. Aphthous stomatitis
D. Arthritis
E. Elevation of serum angiotensin-converting enzyme

A

E. Elevation of serum angiotensin-converting enzyme

Sarcoidosis, not Behget’s syndrome, is associated
with elevated levels of angiotensin-converting enzyme
(Merritt, pp. 121- 122).

How well did you know this?
1
Not at all
2
3
4
5
Perfectly
47
Q

Match the following questions with the associated EMG finding, using each answer only ONCE

Postexercise facilitation
A. Myasthenia gravis
B. Lambert-Eaton syndrome
C. Polymyositis
D. Carpal tunnel syndrome
E. Myotonia
F. None of the above

A

B. Lambert-Eaton syndrome

How well did you know this?
1
Not at all
2
3
4
5
Perfectly
48
Q

Match the following questions with the associated EMG finding, using each answer only ONCE

Decrementalmotor response
**A. Myasthenia gravis **
B. Lambert-Eaton syndrome
C. Polymyositis
D. Carpal tunnel syndrome
E. Myotonia
F. None of the above

A

**A. Myasthenia gravis **

How well did you know this?
1
Not at all
2
3
4
5
Perfectly
49
Q

Match the following questions with the associated EMG finding, using each answer only ONCE

“Dive bomber” frequency
A. Myasthenia gravis
B. Lambert-Eaton syndrome
C. Polymyositis
D. Carpal tunnel syndrome
E. Myotonia
F. None of the above

A

**E. Myotonia **

How well did you know this?
1
Not at all
2
3
4
5
Perfectly
50
Q

Match the following questions with the associated EMG finding, using each answer only ONCE

Myopathic motor units, fibrillation, pseudo myotonia
A. Myasthenia gravis
B. Lambert-Eaton syndrome
C. Polymyositis
D. Carpal tunnel syndrome
E. Myotonia
F. None of the above

A

**C. Polymyositis **

How well did you know this?
1
Not at all
2
3
4
5
Perfectly
51
Q

Match the following questions with the associated EMG finding, using each answer only ONCE

Sensory> motor latencies
A. Myasthenia gravis
B. Lambert-Eaton syndrome
C. Polymyositis
D. Carpal tunnel syndrome
E. Myotonia
F. None of the above

A

**D. Carpal tunnel syndrome **

How well did you know this?
1
Not at all
2
3
4
5
Perfectly
52
Q

A 13-year-old boy with a lytic skull lesion presents
with diabetes insipidus and the coronal Mill depicted below.

What is the most likely diagnosis?
A. Granular cell tumor
B. Sarcoidosis
C. Pituitary adenoma
D. Langerhans’ cell histiocytosis
E. Germinoma

A

D. Langerhans’ cell histiocytosis

Note the abnormally thickened stalk
with high signal intensity on this coronal MRl depicting
Langerhans’ cell histiocytosis. The etiology of this condition
is unlmown, but it is believed to result from overproliferation
of an antigen-presenting dendritic cell of bone marrow
origin. Although it is usually treated as a neoplastic process,
some speculate that it is due to malfunction of the immune
system. Other manifestations of this disease may include
lytic skull lesions (approximately 80% of cases) as well
as hematopoietic, hepatic, and pulmonary abnormalities.
A pathognomic finding of this condition on electron
microscopy is the presence of Birbeck granules, a unique
organelle of the Langerhans’ cell (Ramsey, pp. 381- 385;
Merritt, p. 872).

How well did you know this?
1
Not at all
2
3
4
5
Perfectly
53
Q

A 13-year-old boy with a lytic skull lesion presents
with diabetes insipidus and the coronal Mill depicted below.

This disorder is marked by proliferation of what cell type?
A. Fibroblasts
B. T-ceillymphocytes
C. Antigen-presenting dendritic cells
D. Eosinophils
E. Cells derived from Rathke’s pouch

A

**C. Antigen-presenting dendritic cells **

Note the abnormally thickened stalk
with high signal intensity on this coronal MRl depicting
Langerhans’ cell histiocytosis. The etiology of this condition
is unlmown, but it is believed to result from overproliferation
of an antigen-presenting dendritic cell of bone marrow
origin. Although it is usually treated as a neoplastic process,
some speculate that it is due to malfunction of the immune
system. Other manifestations of this disease may include
lytic skull lesions (approximately 80% of cases) as well
as hematopoietic, hepatic, and pulmonary abnormalities.
A pathognomic finding of this condition on electron
microscopy is the presence of Birbeck granules, a unique
organelle of the Langerhans’ cell (Ramsey, pp. 381- 385;
Merritt, p. 872).

How well did you know this?
1
Not at all
2
3
4
5
Perfectly
54
Q

A 13-year-old boy with a lytic skull lesion presents
with diabetes insipidus and the coronal Mill depicted below.

A pathognomonic finding of this condition on microscopy includes the presence of
A. Birbeck granules
B. Junctional complexes
c. Cholesterol crystals
D. Keratohyaline granules
E. Stippled chromatin

A

**A. Birbeck granules **

Note the abnormally thickened stalk
with high signal intensity on this coronal MRl depicting
Langerhans’ cell histiocytosis. The etiology of this condition
is unlmown, but it is believed to result from overproliferation
of an antigen-presenting dendritic cell of bone marrow
origin. Although it is usually treated as a neoplastic process,
some speculate that it is due to malfunction of the immune
system. Other manifestations of this disease may include
lytic skull lesions (approximately 80% of cases) as well
as hematopoietic, hepatic, and pulmonary abnormalities.
A pathognomic finding of this condition on electron
microscopy is the presence of Birbeck granules, a unique
organelle of the Langerhans’ cell (Ramsey, pp. 381- 385;
Merritt, p. 872).

How well did you know this?
1
Not at all
2
3
4
5
Perfectly
55
Q

What is depicted in the photolnicrograph below?
A. N eurofi broma
B. Transitional meningioma
C. Acoustic neuroma
D. Pilocytic astrocytoma
E. Pleomorphic xanthoastrocytoma

A

C. Acoustic neuroma

Acoustic neuroma. Note the palisading of nuclei
(picket fence-like arrangement) separated by an anuclear
area (arrow) on this photomicrograph, which depicts a
Verocay body (Ellison, pp. 695- 699).

How well did you know this?
1
Not at all
2
3
4
5
Perfectly
56
Q

What does the arrow depict?
A. Verocay body
B. Whorls
C. Psammoma body
D. Pseudopalisading
E. Antoni B area

A

**A. Verocay body **

Acoustic neuroma. Note the palisading of nuclei
(picket fence-like arrangement) separated by an anuclear
area (arrow) on this photomicrograph, which depicts a
Verocay body (Ellison, pp. 695- 699).

How well did you know this?
1
Not at all
2
3
4
5
Perfectly
57
Q

When a patient cannot adduct the right eye in attempting to.look to the left but the eye adducts on convergence, the
lesion is most likely in what location?
A. Right medial longitudinal fasciculus
B. Left mecliallongituclinal fasciculus
C. Left abducens nucleus
D. Right abducens nucleus
E. Nucleus of cranial nerve III

A

**A. Right medial longitudinal fasciculus **

A lesion of the MLF does not allow for transfer of information from the abducens nucleus (CN VI) to the opoculomotor nucleus (CN III) and results in internuclear
ophthalmoplegia (INO). It is characterized by deficient
adduction during attempted conjugate gaze away from the
side of the lvILF lesion and monocular nystagmus of the
abducting eye. An i’vILF lesion is on the same side as
the eye with the adduction wealmess, and INO is named
for the side of the J’vILF lesion. 1\ lesion in the nucleus of CN
III would paralyze volitional movements and convergence
(Kline, pp. 63-64).

How well did you know this?
1
Not at all
2
3
4
5
Perfectly
58
Q

Match the following peripheral nerve injuries
with the appropriate hand abnormality (Figure 8.58-8.63Q),
using each answer once , more than once, or not at all.

Injury of the median nerve at the level of the elbow

A

B

Compressive lesions of the
ulnar nerve at the level of the elbow, forea rm, or wrist can
produce a “claw hand” (A) in severe cases. The ulnar half
of the flexor digitorum profundus, lumbricals 3 and 4, the
dorsal and palmar interossei, and the hypothenar muscles
are typically paralyzed. When the metacarpophalangeal
joints are extended, the distal and proximal interphalangeal
joints cannot be extended because the interossei and half the
IUl11bricais are not functional, which results in a “claw-like”
posture. Laceration of the ulnar nerve in the wrist leaves
the innervation of the ulnar side of the flexor digitorum
profundus intact but can also result in a claw hand. There is
also loss of abduction of the thumb, so that a piece of paper
cannot be held between the side of the thumb and the index
finger.

How well did you know this?
1
Not at all
2
3
4
5
Perfectly
59
Q

Match the following peripheral nerve injuries
with the appropriate hand abnormality (Figure 8.58-8.63Q),
using each answer once , more than once, or not at all.

Injury of the ulnar nerve at the elbow

A

A

Compressive lesions of the
ulnar nerve at the level of the elbow, forea rm, or wrist can
produce a “claw hand” (A) in severe cases. The ulnar half
of the flexor digitorum profundus, lumbricals 3 and 4, the
dorsal and palmar interossei, and the hypothenar muscles
are typically paralyzed. When the metacarpophalangeal
joints are extended, the distal and proximal interphalangeal
joints cannot be extended because the interossei and half the
IUl11bricais are not functional, which results in a “claw-like”
posture. Laceration of the ulnar nerve in the wrist leaves
the innervation of the ulnar side of the flexor digitorum
profundus intact but can also result in a claw hand. There is
also loss of abduction of the thumb, so that a piece of paper
cannot be held between the side of the thumb and the index
finger.

How well did you know this?
1
Not at all
2
3
4
5
Perfectly
60
Q

Match the following peripheral nerve injuries
with the appropriate hand abnormality (Figure 8.58-8.63Q),
using each answer once , more than once, or not at all.

Injury of the ulnar nerve at the wrist

A

A

Compressive lesions of the
ulnar nerve at the level of the elbow, forea rm, or wrist can
produce a “claw hand” (A) in severe cases. The ulnar half
of the flexor digitorum profundus, lumbricals 3 and 4, the
dorsal and palmar interossei, and the hypothenar muscles
are typically paralyzed. When the metacarpophalangeal
joints are extended, the distal and proximal interphalangeal
joints cannot be extended because the interossei and half the
IUl11bricais are not functional, which results in a “claw-like”
posture. Laceration of the ulnar nerve in the wrist leaves
the innervation of the ulnar side of the flexor digitorum
profundus intact but can also result in a claw hand. There is
also loss of abduction of the thumb, so that a piece of paper
cannot be held between the side of the thumb and the index
finger.

How well did you know this?
1
Not at all
2
3
4
5
Perfectly
61
Q

Match the following peripheral nerve injuries
with the appropriate hand abnormality (Figure 8.58-8.63Q),
using each answer once , more than once, or not at all.

Anterior interosseous nerve injury

A

C

Compressive lesions of the
ulnar nerve at the level of the elbow, forea rm, or wrist can
produce a “claw hand” (A) in severe cases. The ulnar half
of the flexor digitorum profundus, lumbricals 3 and 4, the
dorsal and palmar interossei, and the hypothenar muscles
are typically paralyzed. When the metacarpophalangeal
joints are extended, the distal and proximal interphalangeal
joints cannot be extended because the interossei and half the
IUl11bricais are not functional, which results in a “claw-like”
posture. Laceration of the ulnar nerve in the wrist leaves
the innervation of the ulnar side of the flexor digitorum
profundus intact but can also result in a claw hand. There is
also loss of abduction of the thumb, so that a piece of paper
cannot be held between the side of the thumb and the index
finger.

How well did you know this?
1
Not at all
2
3
4
5
Perfectly
62
Q

Match the following peripheral nerve injuries
with the appropriate hand abnormality (Figure 8.58-8.63Q),
using each answer once , more than once, or not at all.

Klumpke’s paralysis

A

D

Compressive lesions of the
ulnar nerve at the level of the elbow, forea rm, or wrist can
produce a “claw hand” (A) in severe cases. The ulnar half
of the flexor digitorum profundus, lumbricals 3 and 4, the
dorsal and palmar interossei, and the hypothenar muscles
are typically paralyzed. When the metacarpophalangeal
joints are extended, the distal and proximal interphalangeal
joints cannot be extended because the interossei and half the
IUl11bricais are not functional, which results in a “claw-like”
posture. Laceration of the ulnar nerve in the wrist leaves
the innervation of the ulnar side of the flexor digitorum
profundus intact but can also result in a claw hand. There is
also loss of abduction of the thumb, so that a piece of paper
cannot be held between the side of the thumb and the index
finger.

How well did you know this?
1
Not at all
2
3
4
5
Perfectly
63
Q

Match the following peripheral nerve injuries
with the appropriate hand abnormality (Figure 8.58-8.63Q),
using each answer once , more than once, or not at all.

C8 root lesion

A

C

Compressive lesions of the
ulnar nerve at the level of the elbow, forea rm, or wrist can
produce a “claw hand” (A) in severe cases. The ulnar half
of the flexor digitorum profundus, lumbricals 3 and 4, the
dorsal and palmar interossei, and the hypothenar muscles
are typically paralyzed. When the metacarpophalangeal
joints are extended, the distal and proximal interphalangeal
joints cannot be extended because the interossei and half the
IUl11bricais are not functional, which results in a “claw-like”
posture. Laceration of the ulnar nerve in the wrist leaves
the innervation of the ulnar side of the flexor digitorum
profundus intact but can also result in a claw hand. There is
also loss of abduction of the thumb, so that a piece of paper
cannot be held between the side of the thumb and the index
finger.

How well did you know this?
1
Not at all
2
3
4
5
Perfectly
64
Q

Match each of the following lesion sites with the
associated clinical deficit.

Posterior end of the inferior frontal gyrus
A. Dressing apncda
B. Writing apraxia
C. Speech apraxia
D. Gait apraxia
E. Prosopagnosia
F. Astereognosis
G. None of the above

A

**C. Speech apraxia **

Apraxia is defined as the
inability to execute a normal volitional act despite the fact
that the motor systems and mental status are relatively
intact. Speech apraxia often results from a lesion near the
posterior part of the inferior frontal gyrus (approximately
area 44), while writing apraxia or dysgraphia results from
damage in the left angular gyrus. Dressing apraxia results
from damage in the posterior right parietal lobe, while gait
apraxia is usually associated with diffuse cerebral disease
such as Alzheimer’s disease. Lesions that affect the inferomedial part of the temporo-occipital region tend to cause an
inability to recognize facial features (pl:osopagnosia), while
lesions of either parietal lobe rnay produce astereognosis, in
which patients fail to recognize the forms of objects when felt
but not when viewed (Brazis, pp. 481-508).

How well did you know this?
1
Not at all
2
3
4
5
Perfectly
65
Q

Match each of the following lesion sites with the
associated clinical deficit.

Left angular gyrus
A. Dressing apncda
B. Writing apraxia
C. Speech apraxia
D. Gait apraxia
E. Prosopagnosia
F. Astereognosis
G. None of the above

A

**B. Writing apraxia **

Apraxia is defined as the
inability to execute a normal volitional act despite the fact
that the motor systems and mental status are relatively
intact. Speech apraxia often results from a lesion near the
posterior part of the inferior frontal gyrus (approximately
area 44), while writing apraxia or dysgraphia results from
damage in the left angular gyrus. Dressing apraxia results
from damage in the posterior right parietal lobe, while gait
apraxia is usually associated with diffuse cerebral disease
such as Alzheimer’s disease. Lesions that affect the inferomedial part of the temporo-occipital region tend to cause an
inability to recognize facial features (pl:osopagnosia), while
lesions of either parietal lobe rnay produce astereognosis, in
which patients fail to recognize the forms of objects when felt
but not when viewed (Brazis, pp. 481-508).

How well did you know this?
1
Not at all
2
3
4
5
Perfectly
66
Q

Match each of the following lesion sites with the
associated clinical deficit.

Posterior part of right parietal lobe
A. Dressing apncda
B. Writing apraxia
C. Speech apraxia
D. Gait apraxia
E. Prosopagnosia
F. Astereognosis
G. None of the above

A

**A. Dressing apncda **

Apraxia is defined as the
inability to execute a normal volitional act despite the fact
that the motor systems and mental status are relatively
intact. Speech apraxia often results from a lesion near the
posterior part of the inferior frontal gyrus (approximately
area 44), while writing apraxia or dysgraphia results from
damage in the left angular gyrus. Dressing apraxia results
from damage in the posterior right parietal lobe, while gait
apraxia is usually associated with diffuse cerebral disease
such as Alzheimer’s disease. Lesions that affect the inferomedial part of the temporo-occipital region tend to cause an
inability to recognize facial features (pl:osopagnosia), while
lesions of either parietal lobe rnay produce astereognosis, in
which patients fail to recognize the forms of objects when felt
but not when viewed (Brazis, pp. 481-508).

How well did you know this?
1
Not at all
2
3
4
5
Perfectly
67
Q

Match each of the following lesion sites with the
associated clinical deficit.

Diffuse cerebral disease
A. Dressing apncda
B. Writing apraxia
C. Speech apraxia
D. Gait apraxia
E. Prosopagnosia
F. Astereognosis
G. None of the above

A

**D. Gait apraxia **

Apraxia is defined as the
inability to execute a normal volitional act despite the fact
that the motor systems and mental status are relatively
intact. Speech apraxia often results from a lesion near the
posterior part of the inferior frontal gyrus (approximately
area 44), while writing apraxia or dysgraphia results from
damage in the left angular gyrus. Dressing apraxia results
from damage in the posterior right parietal lobe, while gait
apraxia is usually associated with diffuse cerebral disease
such as Alzheimer’s disease. Lesions that affect the inferomedial part of the temporo-occipital region tend to cause an
inability to recognize facial features (pl:osopagnosia), while
lesions of either parietal lobe rnay produce astereognosis, in
which patients fail to recognize the forms of objects when felt
but not when viewed (Brazis, pp. 481-508).

How well did you know this?
1
Not at all
2
3
4
5
Perfectly
68
Q

Match each of the following lesion sites with the
associated clinical deficit.

Medial inferior temporo-occipital region
A. Dressing apncda
B. Writing apraxia
C. Speech apraxia
D. Gait apraxia
E. Prosopagnosia
F. Astereognosis
G. None of the above

A

**E. Prosopagnosia **

Apraxia is defined as the
inability to execute a normal volitional act despite the fact
that the motor systems and mental status are relatively
intact. Speech apraxia often results from a lesion near the
posterior part of the inferior frontal gyrus (approximately
area 44), while writing apraxia or dysgraphia results from
damage in the left angular gyrus. Dressing apraxia results
from damage in the posterior right parietal lobe, while gait
apraxia is usually associated with diffuse cerebral disease
such as Alzheimer’s disease. Lesions that affect the inferomedial part of the temporo-occipital region tend to cause an
inability to recognize facial features (pl:osopagnosia), while
lesions of either parietal lobe rnay produce astereognosis, in
which patients fail to recognize the forms of objects when felt
but not when viewed (Brazis, pp. 481-508).

How well did you know this?
1
Not at all
2
3
4
5
Perfectly
69
Q

Match each of the following lesion sites with the
associated clinical deficit.

Either parietal lobe
A. Dressing apncda
B. Writing apraxia
C. Speech apraxia
D. Gait apraxia
E. Prosopagnosia
F. Astereognosis
G. None of the above

A

**F. Astereognosis **

Apraxia is defined as the
inability to execute a normal volitional act despite the fact
that the motor systems and mental status are relatively
intact. Speech apraxia often results from a lesion near the
posterior part of the inferior frontal gyrus (approximately
area 44), while writing apraxia or dysgraphia results from
damage in the left angular gyrus. Dressing apraxia results
from damage in the posterior right parietal lobe, while gait
apraxia is usually associated with diffuse cerebral disease
such as Alzheimer’s disease. Lesions that affect the inferomedial part of the temporo-occipital region tend to cause an
inability to recognize facial features (pl:osopagnosia), while
lesions of either parietal lobe rnay produce astereognosis, in
which patients fail to recognize the forms of objects when felt
but not when viewed (Brazis, pp. 481-508).

How well did you know this?
1
Not at all
2
3
4
5
Perfectly
70
Q

Where do afferent axons serving the muscle stretch
reflex synapse?
A. Dorsal root ganglia
B. Dorsal horn neurons
C. Ventral motoneuron
D. Clarke’s nucleus
E. Rexedlamina III

A

**C. Ventral motoneuron **

The muscle stretch reflex is a monosynaptic circuit
that is dependent on two neurons. Afferent axons serving
the muscle stretch reflex synapse directly with ventralmotoneurons (Carpenter, p. 79).

How well did you know this?
1
Not at all
2
3
4
5
Perfectly
71
Q

A 35-year-old construction worker fell from a three-story
building while at work and suffered a complete spinal cord
injury at the C2 level. Which of the following functions may
be preserved after a complete spinal cord injury at this level?
1. J’dicturition
2. Ejaculation
3. Peristalsis
4. Breathing

A. 1,2, and 3 are correct
B. 1 and 3 are correct
C. 2 and 4 are correct
D. Only 4 is correct
E. All of the above

A

A. 1,2, and 3 are correct

After a complete spinal cord injury, all voluntary
movements and sensation below the level of the lesion are
lost, but a number of visceral reflexes may be preserved
in some cases. A patient with a complete C2 spinal cord
injury is unlikely to be able to breathe, since the spinal cord
does not contain intrinsic circuitry for breathing. Retained
reflexes may include micturition, defecation, peristalsis, and
possibly even ejaculation, although there may be no sensation of the sexual act (Brazis, pp. 85- 88; DeMyer, pp. 142-
143).

How well did you know this?
1
Not at all
2
3
4
5
Perfectly
72
Q

A 15-year-old-girl sees her physician for a physical
examination prior to the start of soccer season. The physician notices that the palate fails to elevate on the left side
when the patient says “Ah.” What other associated deficits
may be seen in this patient?
1. Swallowing
2. Phonation
3. Taste
4. Salivation

A. 1,2, and 3
B. 1 and 3
C. 2 and4
D. Only 4 is correct
E. All of the above

A

**A. 1,2, and 3 **

The most important cranial nerve for palatal elevation
is generally CN X. Interruption of the left CN X can cause
paralysis of palatal elevation on the left side. Taste, swallowing, and phonation are also partially subserved by CN Xi
therefore an insult to this cranial nerve may result in problems with speech, swallowing, and taste. Salivation problems
may be evident with deficits in CN VII and IX (Carpenter,
pp.137-144,172-173).

How well did you know this?
1
Not at all
2
3
4
5
Perfectly
73
Q

Match each of the spinal cord tracts with the
appropriate clinical correlate (Figure 8.73- 8.80), using each
answer once, more than once, or not at all. Major ascending
tracts are depicted on the left, while descending tracts are
shown on the right.

Small peripheral myelinated fibers of tills pathway
synapse in the substantia gelatinosa of the dorsal horn

A

I

The major ascending tracts of the spinal cord (left) are the dorsal columns,
spinothalamic tract, dOl’sal spinocerebellar tract, and ventral
spinocerebellar tract. The dOl’sal columns convey tactile
discrimination (Meissner corpuscles), vibration (pacinian
corpuscle), joint position sense (muscle spindles and Golgi
tendon organs), and conscious proprioception. First-order
neurons give rise to axons that ascend in the fasciculus
cuneatus (A, upper extremity fibers) and gracilis (E, lower
extremity), which terminate in the gracile and cuneate
nuclei of the medulla. Second-order neurons, known as
arcuate fibers, cross to the contralateral side as the medial
lemniscus and ascend to the ventral posterolateral (VPL)
nucleus of the thalamus. Synaptic connections are then
made in the thalamus with third-order neurons, which travel
through the posterior limb of the internal capsule to reach
the postcentral gyrus of the cerebral cortex.

How well did you know this?
1
Not at all
2
3
4
5
Perfectly
74
Q

Match each of the spinal cord tracts with the
appropriate clinical correlate (Figure 8.73- 8.80), using each
answer once, more than once, or not at all. Major ascending
tracts are depicted on the left, while descending tracts are
shown on the right.

These fibers pass through the superior cerebellar peduncle

A

J

The major ascending tracts of the spinal cord (left) are the dorsal columns,
spinothalamic tract, dOl’sal spinocerebellar tract, and ventral
spinocerebellar tract. The dOl’sal columns convey tactile
discrimination (Meissner corpuscles), vibration (pacinian
corpuscle), joint position sense (muscle spindles and Golgi
tendon organs), and conscious proprioception. First-order
neurons give rise to axons that ascend in the fasciculus
cuneatus (A, upper extremity fibers) and gracilis (E, lower
extremity), which terminate in the gracile and cuneate
nuclei of the medulla. Second-order neurons, known as
arcuate fibers, cross to the contralateral side as the medial
lemniscus and ascend to the ventral posterolateral (VPL)
nucleus of the thalamus. Synaptic connections are then
made in the thalamus with third-order neurons, which travel
through the posterior limb of the internal capsule to reach
the postcentral gyrus of the cerebral cortex.

How well did you know this?
1
Not at all
2
3
4
5
Perfectly
75
Q

Match each of the spinal cord tracts with the
appropriate clinical correlate (Figure 8.73- 8.80), using each
answer once, more than once, or not at all. Major ascending
tracts are depicted on the left, while descending tracts are
shown on the right.

This tract arises from Dieter’s nucleus

A

F

The major ascending tracts of the spinal cord (left) are the dorsal columns,
spinothalamic tract, dOl’sal spinocerebellar tract, and ventral
spinocerebellar tract. The dOl’sal columns convey tactile
discrimination (Meissner corpuscles), vibration (pacinian
corpuscle), joint position sense (muscle spindles and Golgi
tendon organs), and conscious proprioception. First-order
neurons give rise to axons that ascend in the fasciculus
cuneatus (A, upper extremity fibers) and gracilis (E, lower
extremity), which terminate in the gracile and cuneate
nuclei of the medulla. Second-order neurons, known as
arcuate fibers, cross to the contralateral side as the medial
lemniscus and ascend to the ventral posterolateral (VPL)
nucleus of the thalamus. Synaptic connections are then
made in the thalamus with third-order neurons, which travel
through the posterior limb of the internal capsule to reach
the postcentral gyrus of the cerebral cortex.

How well did you know this?
1
Not at all
2
3
4
5
Perfectly
76
Q

Match each of the spinal cord tracts with the
appropriate clinical correlate (Figure 8.73- 8.80), using each
answer once, more than once, or not at all. Major ascending
tracts are depicted on the left, while descending tracts are
shown on the right.

Conscious proprioception from the legs is mainly transmitted in this tract

A

B

The major ascending tracts of the spinal cord (left) are the dorsal columns,
spinothalamic tract, dOl’sal spinocerebellar tract, and ventral
spinocerebellar tract. The dOl’sal columns convey tactile
discrimination (Meissner corpuscles), vibration (pacinian
corpuscle), joint position sense (muscle spindles and Golgi
tendon organs), and conscious proprioception. First-order
neurons give rise to axons that ascend in the fasciculus
cuneatus (A, upper extremity fibers) and gracilis (E, lower
extremity), which terminate in the gracile and cuneate
nuclei of the medulla. Second-order neurons, known as
arcuate fibers, cross to the contralateral side as the medial
lemniscus and ascend to the ventral posterolateral (VPL)
nucleus of the thalamus. Synaptic connections are then
made in the thalamus with third-order neurons, which travel
through the posterior limb of the internal capsule to reach
the postcentral gyrus of the cerebral cortex.

How well did you know this?
1
Not at all
2
3
4
5
Perfectly
77
Q

Match each of the spinal cord tracts with the
appropriate clinical correlate (Figure 8.73- 8.80), using each
answer once, more than once, or not at all. Major ascending
tracts are depicted on the left, while descending tracts are
shown on the right.

Carries fibers from medial and inferior vestibular nuclei,
tectospinal tract, and interstitial nucleus of Cajal

A

G

The major ascending tracts of the spinal cord (left) are the dorsal columns,
spinothalamic tract, dOl’sal spinocerebellar tract, and ventral
spinocerebellar tract. The dOl’sal columns convey tactile
discrimination (Meissner corpuscles), vibration (pacinian
corpuscle), joint position sense (muscle spindles and Golgi
tendon organs), and conscious proprioception. First-order
neurons give rise to axons that ascend in the fasciculus
cuneatus (A, upper extremity fibers) and gracilis (E, lower
extremity), which terminate in the gracile and cuneate
nuclei of the medulla. Second-order neurons, known as
arcuate fibers, cross to the contralateral side as the medial
lemniscus and ascend to the ventral posterolateral (VPL)
nucleus of the thalamus. Synaptic connections are then
made in the thalamus with third-order neurons, which travel
through the posterior limb of the internal capsule to reach
the postcentral gyrus of the cerebral cortex.

How well did you know this?
1
Not at all
2
3
4
5
Perfectly
78
Q

Match each of the spinal cord tracts with the
appropriate clinical correlate (Figure 8.73- 8.80), using each
answer once, more than once, or not at all. Major ascending
tracts are depicted on the left, while descending tracts are
shown on the right.

Fibers from this tract originate from layer V of the
cerebral cortex

A

C

The major ascending tracts of the spinal cord (left) are the dorsal columns,
spinothalamic tract, dOl’sal spinocerebellar tract, and ventral
spinocerebellar tract. The dOl’sal columns convey tactile
discrimination (Meissner corpuscles), vibration (pacinian
corpuscle), joint position sense (muscle spindles and Golgi
tendon organs), and conscious proprioception. First-order
neurons give rise to axons that ascend in the fasciculus
cuneatus (A, upper extremity fibers) and gracilis (E, lower
extremity), which terminate in the gracile and cuneate
nuclei of the medulla. Second-order neurons, known as
arcuate fibers, cross to the contralateral side as the medial
lemniscus and ascend to the ventral posterolateral (VPL)
nucleus of the thalamus. Synaptic connections are then
made in the thalamus with third-order neurons, which travel
through the posterior limb of the internal capsule to reach
the postcentral gyrus of the cerebral cortex.

How well did you know this?
1
Not at all
2
3
4
5
Perfectly
79
Q

Match each of the spinal cord tracts with the
appropriate clinical correlate (Figure 8.73- 8.80), using each
answer once, more than once, or not at all. Major ascending
tracts are depicted on the left, while descending tracts are
shown on the right.

Carries fibers that ascend to either the thalamus, periaqueductal gray, reticular formation , or superior colliculus

A

I

The major ascending tracts of the spinal cord (left) are the dorsal columns,
spinothalamic tract, dOl’sal spinocerebellar tract, and ventral
spinocerebellar tract. The dOl’sal columns convey tactile
discrimination (Meissner corpuscles), vibration (pacinian
corpuscle), joint position sense (muscle spindles and Golgi
tendon organs), and conscious proprioception. First-order
neurons give rise to axons that ascend in the fasciculus
cuneatus (A, upper extremity fibers) and gracilis (E, lower
extremity), which terminate in the gracile and cuneate
nuclei of the medulla. Second-order neurons, known as
arcuate fibers, cross to the contralateral side as the medial
lemniscus and ascend to the ventral posterolateral (VPL)
nucleus of the thalamus. Synaptic connections are then
made in the thalamus with third-order neurons, which travel
through the posterior limb of the internal capsule to reach
the postcentral gyrus of the cerebral cortex.

How well did you know this?
1
Not at all
2
3
4
5
Perfectly
80
Q

Match each of the spinal cord tracts with the
appropriate clinical correlate (Figure 8.73- 8.80), using each
answer once, more than once, or not at all. Major ascending
tracts are depicted on the left, while descending tracts are
shown on the right.

Uncrossed pyramidal fibers mainly supplying the axial
musculature

A

H

The major ascending tracts of the spinal cord (left) are the dorsal columns,
spinothalamic tract, dOl’sal spinocerebellar tract, and ventral
spinocerebellar tract. The dOl’sal columns convey tactile
discrimination (Meissner corpuscles), vibration (pacinian
corpuscle), joint position sense (muscle spindles and Golgi
tendon organs), and conscious proprioception. First-order
neurons give rise to axons that ascend in the fasciculus
cuneatus (A, upper extremity fibers) and gracilis (E, lower
extremity), which terminate in the gracile and cuneate
nuclei of the medulla. Second-order neurons, known as
arcuate fibers, cross to the contralateral side as the medial
lemniscus and ascend to the ventral posterolateral (VPL)
nucleus of the thalamus. Synaptic connections are then
made in the thalamus with third-order neurons, which travel
through the posterior limb of the internal capsule to reach
the postcentral gyrus of the cerebral cortex.

How well did you know this?
1
Not at all
2
3
4
5
Perfectly
81
Q

Facial nerve displacement by an acoustic neuroma is most commonly (in decreasing order of frequency) in what
direction?
A. Inferior, followed by anterior, superior, and posterior
B. Anterior, followed by superior, inferior, and posterior
C. Anterior, followed by inferior, superior, and posterior
D. Posterior, followed by anterior, inferior, and rarely
superior
E. Superior, followed by inferior, anterior, and posterior

A

**B. Anterior, followed by superior, inferior, and posterior **

Facial nerve displacement by an acoustic neuroma is
most commonly (in deCI’easing order of frequency) anterior,
followed by superior, inferior, and posterior. The facial nerve
is often stretched during microdissection and is most susceptible to injury at the proximal rim of the porus acusticus
(Connolly, p. 475)

How well did you know this?
1
Not at all
2
3
4
5
Perfectly
82
Q

Paralysis of pelvic floor muscles, symmetric saddle
anesthesia, impaired erection and ejaculation, constipation,
and an autonomous neurogenic bladder best describe what spinal cord lesion?
A. Lesion of the first and second sacral segments
B. Cauda equina syndrome
C. Conus medullaris syndrome
D. Tethered cord syndrome
E. Syringomyelia

A

**C. Conus medullaris syndrome **

Paralysis of pelvic floor muscles, early sphincter
and bladder dysfunction, symmetric saddle anestheSia, impaired erection and ejaculation, constipation, and minimal
pain best characterize the conus medullaris syndrome.
A tethered cord may present with a combination of neurologic, urologic , orthopediC, and dermatologic manifestations.
Commonly patients present with numb feet, muscle atrophy,
upper motor neuron Signs, bowel and bladder dysfunction,
foot deformities, scoliOSiS, and cutaneous stigmata of spinal
dysraphism. Compression of the lumbar and sacral roots
below L3 often results in cauda equina syndrome, which is
characterized by early pain, asymmetric saddle anesthesia,
and a variable patellar reflex response . Sphincter changes
are often similar to those of the conus medullaris syndrome
but tend to occur late in the clinical course. With Sllesions,
there is weakness of the triceps surae, flexor digitorum
longus (FDL), flexor hallucis longus (FHL), and small foot
muscles. The Achilles reflexes are absent, whereas the patellar reflexes are preserved. There is complete sensory loss
over the sole, heel, and outer part of the foot and ankle .
The gastrocnemius and soleus muscles are stronger with S2
segmental lesions, however, the FDL, FHL, and foot muscles
remain weak. The sensory loss tends to involve the upper
part of the dOl’sal calf, dorsolateral thigh, and the saddle area
(Brazis, pp. 99-100)

How well did you know this?
1
Not at all
2
3
4
5
Perfectly
83
Q

The finding depicted on the CT scan below (Figure 8.83Q) is most likely to occur after
A. Berry aneurysm rupture
B. Infection
C. Extradural carotid artery dissection
D. Trauma
E. Contrast administration

A

**D. Trauma **

This CT demonstrates a subarachnoid hemorrhage
(SAH), which is most commonly seen after trauma. Although
the blood pattern may vary, traumatic SAH often involves the
convexities of the cerebral hemispheres, while aneurysmal
subarachnoid hemorrhages generally have a preponderance
of blood in the basal cisterns (Greenberg, p. 754)

How well did you know this?
1
Not at all
2
3
4
5
Perfectly
84
Q

Vhat is the region of cerebral cortex most closely associated with the conscious perception of smell?
A. Temporal association cortex
B. Cingulate gyrus
C. Limbic system
D. Orbitofrontal cortex
E. Amygdala

A

D. Orbitofrontal cortex

Experimental studies indicate the orbitofrontal cortex is a key region involved with the conscious perception
of smell, as lesions in this region have been shown to result
in failure to discriminate between various odorants (Kandel,
p.633).

How well did you know this?
1
Not at all
2
3
4
5
Perfectly
85
Q

What is the most likely
diagnosis ?
A. Pilocytic astrocytoma
B. Medulloblastoma
C. Subacute infarct
D. Lhermitte-Duclos disease
E. Ependymoma

A

D. Lhermitte-Duclos disease

This T2-weighted image shows the hyperintense
and thickened folia in a characteristic laminated pattern
that is most consistent with Lhermitte-Duclos disease. It is
associated with hypertrophy of granular cell neurons and
axonal hypennyelination in the molecular layer (Osborn ON,
pp.69-70).

How well did you know this?
1
Not at all
2
3
4
5
Perfectly
86
Q

Which of the following best characterizes this abnormality?
A. These lesions typically have an abundance of Rosenthal
fibers
B. Most often secondary to vertebral artery occlusion
C. Hypertrophy of granular-cell neurons and axonal
hypermyelination in the molecular layer
D. Evidence of Homer-Wright rosettes on histopathologic
sectioning
E. Pseudorosettes on histopathologic sectioning

A

**C. Hypertrophy of granular-cell neurons and axonal
hypermyelination in the molecular layer **

This T2-weighted image shows the hyperintense
and thickened folia in a characteristic laminated pattern
that is most consistent with Lhermitte-Duclos disease. It is
associated with hypertrophy of granular cell neurons and
axonal hypennyelination in the molecular layer (Osborn ON,
pp.69-70).

How well did you know this?
1
Not at all
2
3
4
5
Perfectly
87
Q

A 42-year-old female presents to the emergency
department with staring spells and the T2-weighted MR
image depicted below (Figure 8.87Q). What is the most likely
diagnosis ?
A. Temporal lobe ganglioglioma
B. Dysembryoplastic neuroepithelial tumor
C. Epidermoid cyst
D. Aneurysm
E. Neurocysticercosis

A

**D. Aneurysm **

This T2-weighted MRI shows a right temporal lobe
mass with signal loss (flow void), which is most consistent
with a large middle cerebral artery anemysm (Osborn ON,
pp. 266- 268)

How well did you know this?
1
Not at all
2
3
4
5
Perfectly
88
Q

Match each of the following types of nystagmus
with the specific lesion areas, using each answer once, more
than once, or not at all.

Posterior diencephalon/pretectum (interstitial nucleus
of Cajal); suprasellar region

A. Downbeat nystagmus
B. Upbeat nystagmus
C. Seesaw nystagmus
D. Spasmus nutans
E. Ocular bobbing
F. Ocular flutter
G. Convergence-retraction nystagmus
H. Ocular myoclonus
I. Abducting nystagmus
J. Bruns nystagmus

A

**C. Seesaw nystagmus **

How well did you know this?
1
Not at all
2
3
4
5
Perfectly
89
Q

Match each of the following types of nystagmus
with the specific lesion areas, using each answer once, more
than once, or not at all.

Dorsal midbrain
A. Downbeat nystagmus
B. Upbeat nystagmus
C. Seesaw nystagmus
D. Spasmus nutans
E. Ocular bobbing
F. Ocular flutter
G. Convergence-retraction nystagmus
H. Ocular myoclonus
I. Abducting nystagmus
J. Bruns nystagmus

A

**G. Convergence-retraction nystagmus **

How well did you know this?
1
Not at all
2
3
4
5
Perfectly
90
Q

Match each of the following types of nystagmus
with the specific lesion areas, using each answer once, more
than once, or not at all.

Pons (medial longitudinal faSCiculus)
A. Downbeat nystagmus
B. Upbeat nystagmus
C. Seesaw nystagmus
D. Spasmus nutans
E. Ocular bobbing
F. Ocular flutter
G. Convergence-retraction nystagmus
H. Ocular myoclonus
I. Abducting nystagmus
J. Bruns nystagmus

A

**I. Abducting nystagmus **

How well did you know this?
1
Not at all
2
3
4
5
Perfectly
91
Q

Match each of the following types of nystagmus
with the specific lesion areas, using each answer once, more
than once, or not at all.

Central pons
A. Downbeat nystagmus
B. Upbeat nystagmus
C. Seesaw nystagmus
D. Spasmus nutans
E. Ocular bobbing
F. Ocular flutter
G. Convergence-retraction nystagmus
H. Ocular myoclonus
I. Abducting nystagmus
J. Bruns nystagmus

A

**E. Ocular bobbing **

How well did you know this?
1
Not at all
2
3
4
5
Perfectly
92
Q

Match each of the following types of nystagmus
with the specific lesion areas, using each answer once, more
than once, or not at all.

Ipsilateral inferior olive, red nucleus, contralateral
dentate nucleus (Mollaret triangle)
A. Downbeat nystagmus
B. Upbeat nystagmus
C. Seesaw nystagmus
D. Spasmus nutans
E. Ocular bobbing
F. Ocular flutter
G. Convergence-retraction nystagmus
H. Ocular myoclonus
I. Abducting nystagmus
J. Bruns nystagmus

A

H. Ocular myoclonus

How well did you know this?
1
Not at all
2
3
4
5
Perfectly
93
Q

Match each of the following types of nystagmus
with the specific lesion areas, using each answer once, more
than once, or not at all.

Medulla , ventral tegmentum of pons, cerebellar pathway
A. Downbeat nystagmus
B. Upbeat nystagmus
C. Seesaw nystagmus
D. Spasmus nutans
E. Ocular bobbing
F. Ocular flutter
G. Convergence-retraction nystagmus
H. Ocular myoclonus
I. Abducting nystagmus
J. Bruns nystagmus

A

**B. Upbeat nystagmus **

How well did you know this?
1
Not at all
2
3
4
5
Perfectly
94
Q

Match each of the following types of nystagmus
with the specific lesion areas, using each answer once, more
than once, or not at all.

Cervicomedullary junction
A. Downbeat nystagmus
B. Upbeat nystagmus
C. Seesaw nystagmus
D. Spasmus nutans
E. Ocular bobbing
F. Ocular flutter
G. Convergence-retraction nystagmus
H. Ocular myoclonus
I. Abducting nystagmus
J. Bruns nystagmus

A

**A. Downbeat nystagmus **

How well did you know this?
1
Not at all
2
3
4
5
Perfectly
95
Q

Match each of the following types of nystagmus
with the specific lesion areas, using each answer once, more
than once, or not at all.

Pontomedullary junction, vestibular pathways
A. Downbeat nystagmus
B. Upbeat nystagmus
C. Seesaw nystagmus
D. Spasmus nutans
E. Ocular bobbing
F. Ocular flutter
G. Convergence-retraction nystagmus
H. Ocular myoclonus
I. Abducting nystagmus
J. Bruns nystagmus

A

**J. Bruns nystagmus **

How well did you know this?
1
Not at all
2
3
4
5
Perfectly
96
Q

A patient suffers a closed head injury after a motor
vehicle collision and is noted to have ecchymosis over the
right eye, with diplopia when looking down and to the left.
The diplopia most likely represents wealmess of what muscle?
A. Right superior oblique
B. Left superior rectus
C. Right inferior rectus
D. Left inferior oblique
E. Right inferior oblique

A

**A. Right superior oblique **

Orbital injuries often impair the action of the superior
oblique muscle because of displacement of the trochlea,
which attaches to the anterior rim of the orbit and acts as a
sling for the recurrent course of the trochlear tendon.
Looking down and to the left typically involves the right
superior oblique (trochlear nerve , IV) and left inferior rectus
(oculomotor nerve , III) muscles. \Vhen the eyes look conjuga tely toward any object, the muscle that is the prime mover works in unison with the muscle of the opposite eye (Kline,
pp. 105- 114).

How well did you know this?
1
Not at all
2
3
4
5
Perfectly
97
Q

The lesion depicted on the photomicrograph below (Figure
8.97Q) may be associated with all of the following EXCEPT?
A. Autosomal dominant inheritance
B. Renal cell carcinoma
C. Pancreatic cysts
D. Overproduction of erythropoietin
E. Tumor suppressor gene that maps to chromosome 9p25

A

**E. Tumor suppressor gene that maps to chromosome 9p25 **

Note the numerous capillaries and cells with a vacuolated appearance in this photomicrograph depicting a
hemangioblastoma. This tumor is associated with VI-IL in
about 25% of cases, is carried in an autosomal dominant
fashion (chromosome 3p25), and is associated with retinal
angioma, renal cell carcinoma, renal and pancreatic cysts,
pheochromocytoma, or epididymal papillary cystadenoma .
This tumor may cause polycythemia in about 10% of cases
due to inappropriate production of erythropoietin (Ellison,
pp. 736- 738).

How well did you know this?
1
Not at all
2
3
4
5
Perfectly
98
Q

A 47-year-old female underwent clipping of a
ruptured middle cerebral artery bifurcation aneurysm and
required a blood transfusion while recovering in the ICU.
The patient developed hypotension, fever, confusion, and
back pain shortly after receiving the first unit of packed red
blood cells (PRECs).

What is the most likely etiology of these findings?
A. A prior sensitization in a patient who had a nondetectable level of antibody at the time of blood typing
B. ABO incompatibility
C. Antileukocyte antibodies in a patient with a prior
blood transfusion
D. Viral contamination of the PRECs
E. None of the above

A

**B. ABO incompatibility **

Acute hemolytic transfusion reactions are
uncommon and are rarely life-threatening. They are produced by antibodies in the recipient that bind to ABO surface
antigens or erythrocytes of mismatched donor blood. These
antibodies /L: complement and can produce rapid cell lysis
within minutes. Lysis then provokes a severe inflammatory
reaction, which can lead to hypotension, multiorgan dysfunction, and a host of other clinical findings. This type of
transfusion reaction is often the result of identification
errors, leading to ABO mismatched blood . The transfusion
should be stopped immediately; the volume status, urine
output, and blood pressure should be monitored and maintained; and the patient’s blood sent for free hemoglobin,
haptoglobin levels, and a Coombs’ test. Treatment includes
supportive care with maintenance of good urine out.

How well did you know this?
1
Not at all
2
3
4
5
Perfectly
99
Q

A 47-year-old female underwent clipping of a
ruptured middle cerebral artery bifurcation aneurysm and
required a blood transfusion while recovering in the ICU.
The patient developed hypotension, fever, confusion, and
back pain shortly after receiving the first unit of packed red
blood cells (PRECs).

What should be the next course of management in this
patient’s care?
A. The transfusion should be continued, but hemoglobin
and bilirubin levels should be checked
B. Administer diphenhydramine (25 mg) IV immediately
and continue the transfusion
C. Administer epinephrine (1:1000) 0.5 mg every 10 to
15 minutes until the adverse reaction subsides
D. The transfusion should be stopped and the patient’s
blood sent for free hemoglobin, haptoglobin levels, and
Coombs’ test
E. Stop the transfusion; administer acetaminophen and
diphenhydramine 30 minutes prior to any subsequent
blood transfusion

A

**D. The transfusion should be stopped and the patient’s blood sent for free hemoglobin, haptoglobin levels, and Coombs’ test **

Acute hemolytic transfusion reactions are
uncommon and are rarely life-threatening. They are produced by antibodies in the recipient that bind to ABO surface
antigens or erythrocytes of mismatched donor blood. These
antibodies /L: complement and can produce rapid cell lysis
within minutes. Lysis then provokes a severe inflammatory
reaction, which can lead to hypotension, multiorgan dysfunction, and a host of other clinical findings. This type of
transfusion reaction is often the result of identification
errors, leading to ABO mismatched blood . The transfusion
should be stopped immediately; the volume status, urine
output, and blood pressure should be monitored and maintained; and the patient’s blood sent for free hemoglobin,
haptoglobin levels, and a Coombs’ test. Treatment includes
supportive care with maintenance of good urine out.

How well did you know this?
1
Not at all
2
3
4
5
Perfectly
100
Q

What is the most common intradural spinal cord tumor
in patients with neurofibromatosis type II (NF-2)?
A. Schwannoma
B. Meningioma
C. Paraganglioma
D. Astrocytoma
E. Ependymoma

A

E. Ependymoma

The presence of l11ultiple intradural spinal cord
tumors is relatively common with NF-2 and may include
ependymomas (most common), schwannomas, and meningiomas (Greenberg, p. 478).

How well did you know this?
1
Not at all
2
3
4
5
Perfectly
101
Q

Destruction of the pyramidal cells of Ammon’s horn
would most likely produce severe ~”onal projection loss to
what structure?
A. Subiculum and entorhinal cortex
B. Premotor cortex
C. Amygdala
D. Ventrolateral thalamus
E. Superior colliculus

A

A. Subiculum and entorhinal cortex

The pyramidal neurons of the hippocampus
(Aml11on’s horn) send numerous fiber projections to the
subiculuIll and entorhinal cortex (area 28), which form
the anterior part of the parahippocampal gyrus (Carpenter,
pp. 369- 382).

How well did you know this?
1
Not at all
2
3
4
5
Perfectly
102
Q

What is depicted in the photomicrograph below (Figure 8.102Q)?
A. Hemangiopericytoma
B. IvIedulioblastoma
C. Melanoma
D. Rhabdoid tumor
E. Germinoma

A

**A. Hemangiopericytoma **

Note the “staghorn” vascular channel in this grade II
hemangiopericytoma. These tumors are vimentin-positive ,
ENlA-negative, have a dense arrangement of sheet-like cells,
and have a high nuclear-cytoplasmic ratio. Other characteristics include focallobularity, paucicellular areas, and dense
periceIJular reticulin (Ellison, pp. 736- 738).

How well did you know this?
1
Not at all
2
3
4
5
Perfectly
103
Q

A patient presents to a neurologist with a 2-week
history of wealmess in the muscles of the lower right face. If
this patient was also aphasic, what type of aphasia is IllOSt
likely to accompany the facial wealmess?
A. Agraphia
B. Alexia without agraphia
C. Expressive aphasia
D. Fluent aphasia
E. Auditory word agnosia

A

**C. Expressive aphasia **

Lesions that occupy the anterior part of the left
parasylvian fissure may cause a nontluent type of aphasia
(Broca’s). This region may abut the parts of the motor
cortex that supply the upper motor neuron fibers for
the contralateral facial nucleus. Therefore a patient with
right-sided upper motor neuron facial deficit may also have
an expressive-type of aphasia originating from Broca’s area
(Brazis, pp. 511- 516)

How well did you know this?
1
Not at all
2
3
4
5
Perfectly
104
Q

A coronal section through the plane of the genu of the internal capsule would bisect what structure?
A. Putamen
B. Globus pallidus
C. Caudate nucleus
D. Hypothalamus
E. Thalamus

A

**B. Globus pallidus **

A coronal section through the genu of the internal
capsule would almost exclusively bisect the globus pallid us,
which is triangle-shaped, with its apex fitting into the genu of
the intcrnal capsule (Carpenter, pp. 337- 344).

How well did you know this?
1
Not at all
2
3
4
5
Perfectly
105
Q

The lesion depicted below (Figure 8.10SQ) most likely
originates from what blood vessel?
A. Accessory middle cerebral artery
B. Frontopolar artery
C. Anterior temporal artery
D. Posterior temporal artery
E. Lenticulostriate

A

**C. Anterior temporal artery **

The middle cerebral artery (MCA) is divided
anatomically into four major segments: M1 (horizontal) segment, M2 (insular segment), M3 (opercular segment), and
M4 (cortical) segment. The anterior temporal artery typically arises from the 1-11 segment of the MCA before the
bifurcation. It passes directly anteriorly and inferiorly over
the temporal tip and usually does not course toward the
sylvian fissure. Although relatively uncommon, aneurysms
can form at the origin or further distally along this vessel (as depicted here). An accessory middle cerebral artery is an
MCA branch that arises from either the ACA (more common)
or the ICA and parallels the M1 segment toward the sylvian
fissure. The lenticulostriate arteries are divided into a
smaller medial group and a larger lateral group that originate
from the distal half of the M1 segment and project superiorly
to enter the anterior perforated substance to supply parts of
the lentiform nuclei, caudate nucleus, and internal capsule.
The posterior temporal artery usually originates from the
M4 segment of the MCA and supplies the posterior temporal
lobe. The frontopolar artery is a branch of the anterior cerebral artery (A2 portion), which originates below the rostrum
or genu of the corpus callosum and extends anteriorly to
supply the frontal pole (Osborn DCA, pp. 135- 137).

How well did you know this?
1
Not at all
2
3
4
5
Perfectly
106
Q

What percentage of patients with subarachnoid hemorrhage secondary to aneurysmal rupture develops angiographic
vasospasm at some time during their hospital course?
A. 20%
B.30%
C. 70%
0.80%
E. 90%

A

**C. 70% **

Most patients develop some degree of vessel narrowing after aneurysmal subarachnoid hemorrhage. About 70%
will develop angiographic vasospasm, and approximately
30% will go on to develop symptomatic vasospasm (Youmans,
p.1545).

How well did you know this?
1
Not at all
2
3
4
5
Perfectly
107
Q

A 43-year-old female presents to your clinic with
acromegaly and an i’“IIU revealing a 3-cm pituitary macroadenoma with extension into the right cavernous sinus. The
patient has normal vision and a serum growth hormone
level after induced hyperglycemia of 220 mg/dL. The most
appropriate next step in the management of this patient may
include
A. Transsphenoidal surgery
B. Radiosurgery
C. Octreotide
D. Conventional radiation therapy
E. Aand C

A

E. Aand C

Over the past few decades a variety of medical,
surgical, and radiation interventions have evolved that have
proven effective in reducing GH levels. No one treatment is
uniformly effective, and often a combination of interventions
is required. \Then a macroadenoma is surgically resected
transsphenoidally, endocrine remission rates vary between
65 and 90%. When a macroadenoma is resected, immediate
postoperative remission is reported to be even lower (30 and
79%). The rate of remission is adversely affected by a higher
preoperative GH level and larger invasive tumors. Therefore
biochemical cure with surgery for large GH-secreting macroadenomas is typically not expected. Conventional radiation
therapy can usually shrinl, pituitary tumors when up to
50 Gy is delivered in 1.8-Gy fractions over 6 weeks, but a
decrease or normalization of GIl levels usually takes many
years. When initial GI-I levels are> 100 pg/mL, only 60% of
patients will attain GH levels < 5 ~lg/mL after 18 years following radiation, and about 50% will develop hypopituitarism
within 10 years. Radiosurgery has the same disadvantages as
conventional radiation and would add increased risk in this
patient due to the proximity of the lesion to the optic nerves
and chiasm. Although perhaps controversial, some endocrinologists are advocating primary drug therapy in patients
with GI-I-secreting macroadenomas (especially with normal
vision). Bromocriptine, a dopamine agonist, has been documented to lower GH levels in up to 71% of patients, and
octreotide, a somatostatin analogue, has achieved similar
results. If there is no shrinkage of the tumor after 16 weeks of
therapy, further use of medication has been shown to have
little impact. Patients with rapidly deteriorating vision or
other neurologic problems related to the lesion (unlike our
patient) are often not good candidates for a trial of medical
therapy and often require more urgent surgical intervention.
Nevertheless, the most effective treatment strategy for acromegaly secondary to a GIl-secreting macroadenoma
usually requires a combination of surgical and medical
options (Berger, pp. 405- 406).

How well did you know this?
1
Not at all
2
3
4
5
Perfectly
108
Q

Figure 8.108-8.116Q depicts an endoscopic
approach to the third ventricle. Iatch the following ana tomic
structures to the corresponding letterhead , using each
answer either once, more than once, or not at all

Thalamostriate vein

A

A

Figure 8.108-8.116Q demonstrates the surgical anatomy of
the right lateral and anterior third ventricle during endoscopic third ventriculostomy for aqueductal stenosis. The
ventriculostomy site is depicted in the floor of the third
ventricle in this figure. The veins of the third ventricular system collect into deeper veins that course in a subependymal
location as they travel through the margins of the choroidal
fissure to empty into the internal cerebral, basal, and great
veins. In general, the veins draining the frontal horn and
body of the third ventricle drain into the internal cerebral
vein (not depicted here) as it courses through the velum
interpositum; those draining the temporal horn drain into a
segment of the basal vein of Rosenthal coursing through the
ambient cistern; and the veins from the atrium drain into
segments of the basal, internal cerebral, and great veins
coursing through the quadrigeminal cistern. Of note , the
thalamostriate vein passes forward in the sulcus between the
caudate nucleus and thalamus toward the foramen of Monro
(FOM), where it turns sharply posterior to enter the velum
interpositum to join the internal cerebral vein. The angle
formed by the junction of the internal cerebral vein and
thalamostriate vein, referred to as the venous angle, approximates the level of the FOM 011 the lateral view of a cerebral
angiogram (Will~ins, pp. 1427-1429; Youmans, pp. 1237-
1240).

How well did you know this?
1
Not at all
2
3
4
5
Perfectly
109
Q

Figure 8.108-8.116Q depicts an endoscopic
approach to the third ventricle. Iatch the following ana tomic
structures to the corresponding letterhead , using each
answer either once, more than once, or not at all

Septal vein

A

D

Figure 8.108-8.116Q demonstrates the surgical anatomy of
the right lateral and anterior third ventricle during endoscopic third ventriculostomy for aqueductal stenosis. The
ventriculostomy site is depicted in the floor of the third
ventricle in this figure. The veins of the third ventricular system collect into deeper veins that course in a subependymal
location as they travel through the margins of the choroidal
fissure to empty into the internal cerebral, basal, and great
veins. In general, the veins draining the frontal horn and
body of the third ventricle drain into the internal cerebral
vein (not depicted here) as it courses through the velum
interpositum; those draining the temporal horn drain into a
segment of the basal vein of Rosenthal coursing through the
ambient cistern; and the veins from the atrium drain into
segments of the basal, internal cerebral, and great veins
coursing through the quadrigeminal cistern. Of note , the
thalamostriate vein passes forward in the sulcus between the
caudate nucleus and thalamus toward the foramen of Monro
(FOM), where it turns sharply posterior to enter the velum
interpositum to join the internal cerebral vein. The angle
formed by the junction of the internal cerebral vein and
thalamostriate vein, referred to as the venous angle, approximates the level of the FOM 011 the lateral view of a cerebral
angiogram (Will~ins, pp. 1427-1429; Youmans, pp. 1237-
1240).

How well did you know this?
1
Not at all
2
3
4
5
Perfectly
110
Q

Figure 8.108-8.116Q depicts an endoscopic
approach to the third ventricle. Iatch the following ana tomic
structures to the corresponding letterhead , using each
answer either once, more than once, or not at all

Thalamus

A

G

Figure 8.108-8.116Q demonstrates the surgical anatomy of
the right lateral and anterior third ventricle during endoscopic third ventriculostomy for aqueductal stenosis. The
ventriculostomy site is depicted in the floor of the third
ventricle in this figure. The veins of the third ventricular system collect into deeper veins that course in a subependymal
location as they travel through the margins of the choroidal
fissure to empty into the internal cerebral, basal, and great
veins. In general, the veins draining the frontal horn and
body of the third ventricle drain into the internal cerebral
vein (not depicted here) as it courses through the velum
interpositum; those draining the temporal horn drain into a
segment of the basal vein of Rosenthal coursing through the
ambient cistern; and the veins from the atrium drain into
segments of the basal, internal cerebral, and great veins
coursing through the quadrigeminal cistern. Of note , the
thalamostriate vein passes forward in the sulcus between the
caudate nucleus and thalamus toward the foramen of Monro
(FOM), where it turns sharply posterior to enter the velum
interpositum to join the internal cerebral vein. The angle
formed by the junction of the internal cerebral vein and
thalamostriate vein, referred to as the venous angle, approximates the level of the FOM 011 the lateral view of a cerebral
angiogram (Will~ins, pp. 1427-1429; Youmans, pp. 1237-
1240).

How well did you know this?
1
Not at all
2
3
4
5
Perfectly
111
Q

Figure 8.108-8.116Q depicts an endoscopic
approach to the third ventricle. Iatch the following ana tomic
structures to the corresponding letterhead , using each
answer either once, more than once, or not at all

Choroid plexus

A

B

Figure 8.108-8.116Q demonstrates the surgical anatomy of
the right lateral and anterior third ventricle during endoscopic third ventriculostomy for aqueductal stenosis. The
ventriculostomy site is depicted in the floor of the third
ventricle in this figure. The veins of the third ventricular system collect into deeper veins that course in a subependymal
location as they travel through the margins of the choroidal
fissure to empty into the internal cerebral, basal, and great
veins. In general, the veins draining the frontal horn and
body of the third ventricle drain into the internal cerebral
vein (not depicted here) as it courses through the velum
interpositum; those draining the temporal horn drain into a
segment of the basal vein of Rosenthal coursing through the
ambient cistern; and the veins from the atrium drain into
segments of the basal, internal cerebral, and great veins
coursing through the quadrigeminal cistern. Of note , the
thalamostriate vein passes forward in the sulcus between the
caudate nucleus and thalamus toward the foramen of Monro
(FOM), where it turns sharply posterior to enter the velum
interpositum to join the internal cerebral vein. The angle
formed by the junction of the internal cerebral vein and
thalamostriate vein, referred to as the venous angle, approximates the level of the FOM 011 the lateral view of a cerebral
angiogram (Will~ins, pp. 1427-1429; Youmans, pp. 1237-
1240).

How well did you know this?
1
Not at all
2
3
4
5
Perfectly
112
Q

Figure 8.108-8.116Q depicts an endoscopic
approach to the third ventricle. Iatch the following ana tomic
structures to the corresponding letterhead , using each
answer either once, more than once, or not at all

Anterior caudate vein

A

F

Figure 8.108-8.116Q demonstrates the surgical anatomy of
the right lateral and anterior third ventricle during endoscopic third ventriculostomy for aqueductal stenosis. The
ventriculostomy site is depicted in the floor of the third
ventricle in this figure. The veins of the third ventricular system collect into deeper veins that course in a subependymal
location as they travel through the margins of the choroidal
fissure to empty into the internal cerebral, basal, and great
veins. In general, the veins draining the frontal horn and
body of the third ventricle drain into the internal cerebral
vein (not depicted here) as it courses through the velum
interpositum; those draining the temporal horn drain into a
segment of the basal vein of Rosenthal coursing through the
ambient cistern; and the veins from the atrium drain into
segments of the basal, internal cerebral, and great veins
coursing through the quadrigeminal cistern. Of note , the
thalamostriate vein passes forward in the sulcus between the
caudate nucleus and thalamus toward the foramen of Monro
(FOM), where it turns sharply posterior to enter the velum
interpositum to join the internal cerebral vein. The angle
formed by the junction of the internal cerebral vein and
thalamostriate vein, referred to as the venous angle, approximates the level of the FOM 011 the lateral view of a cerebral
angiogram (Will~ins, pp. 1427-1429; Youmans, pp. 1237-
1240).

How well did you know this?
1
Not at all
2
3
4
5
Perfectly
113
Q

Figure 8.108-8.116Q depicts an endoscopic
approach to the third ventricle. Iatch the following ana tomic
structures to the corresponding letterhead , using each
answer either once, more than once, or not at all

Superior choroidal vein

A

C

Figure 8.108-8.116Q demonstrates the surgical anatomy of
the right lateral and anterior third ventricle during endoscopic third ventriculostomy for aqueductal stenosis. The
ventriculostomy site is depicted in the floor of the third
ventricle in this figure. The veins of the third ventricular system collect into deeper veins that course in a subependymal
location as they travel through the margins of the choroidal
fissure to empty into the internal cerebral, basal, and great
veins. In general, the veins draining the frontal horn and
body of the third ventricle drain into the internal cerebral
vein (not depicted here) as it courses through the velum
interpositum; those draining the temporal horn drain into a
segment of the basal vein of Rosenthal coursing through the
ambient cistern; and the veins from the atrium drain into
segments of the basal, internal cerebral, and great veins
coursing through the quadrigeminal cistern. Of note , the
thalamostriate vein passes forward in the sulcus between the
caudate nucleus and thalamus toward the foramen of Monro
(FOM), where it turns sharply posterior to enter the velum
interpositum to join the internal cerebral vein. The angle
formed by the junction of the internal cerebral vein and
thalamostriate vein, referred to as the venous angle, approximates the level of the FOM 011 the lateral view of a cerebral
angiogram (Will~ins, pp. 1427-1429; Youmans, pp. 1237-
1240).

How well did you know this?
1
Not at all
2
3
4
5
Perfectly
114
Q

Figure 8.108-8.116Q depicts an endoscopic
approach to the third ventricle. Iatch the following ana tomic
structures to the corresponding letterhead , using each
answer either once, more than once, or not at all

Fornix

A

E

Figure 8.108-8.116Q demonstrates the surgical anatomy of
the right lateral and anterior third ventricle during endoscopic third ventriculostomy for aqueductal stenosis. The
ventriculostomy site is depicted in the floor of the third
ventricle in this figure. The veins of the third ventricular system collect into deeper veins that course in a subependymal
location as they travel through the margins of the choroidal
fissure to empty into the internal cerebral, basal, and great
veins. In general, the veins draining the frontal horn and
body of the third ventricle drain into the internal cerebral
vein (not depicted here) as it courses through the velum
interpositum; those draining the temporal horn drain into a
segment of the basal vein of Rosenthal coursing through the
ambient cistern; and the veins from the atrium drain into
segments of the basal, internal cerebral, and great veins
coursing through the quadrigeminal cistern. Of note , the
thalamostriate vein passes forward in the sulcus between the
caudate nucleus and thalamus toward the foramen of Monro
(FOM), where it turns sharply posterior to enter the velum
interpositum to join the internal cerebral vein. The angle
formed by the junction of the internal cerebral vein and
thalamostriate vein, referred to as the venous angle, approximates the level of the FOM 011 the lateral view of a cerebral
angiogram (Will~ins, pp. 1427-1429; Youmans, pp. 1237-
1240).

How well did you know this?
1
Not at all
2
3
4
5
Perfectly
115
Q

Figure 8.108-8.116Q depicts an endoscopic
approach to the third ventricle. Iatch the following ana tomic
structures to the corresponding letterhead , using each
answer either once, more than once, or not at all

Superior superficial thalamic veins

A

H

Figure 8.108-8.116Q demonstrates the surgical anatomy of
the right lateral and anterior third ventricle during endoscopic third ventriculostomy for aqueductal stenosis. The
ventriculostomy site is depicted in the floor of the third
ventricle in this figure. The veins of the third ventricular system collect into deeper veins that course in a subependymal
location as they travel through the margins of the choroidal
fissure to empty into the internal cerebral, basal, and great
veins. In general, the veins draining the frontal horn and
body of the third ventricle drain into the internal cerebral
vein (not depicted here) as it courses through the velum
interpositum; those draining the temporal horn drain into a
segment of the basal vein of Rosenthal coursing through the
ambient cistern; and the veins from the atrium drain into
segments of the basal, internal cerebral, and great veins
coursing through the quadrigeminal cistern. Of note , the
thalamostriate vein passes forward in the sulcus between the
caudate nucleus and thalamus toward the foramen of Monro
(FOM), where it turns sharply posterior to enter the velum
interpositum to join the internal cerebral vein. The angle
formed by the junction of the internal cerebral vein and
thalamostriate vein, referred to as the venous angle, approximates the level of the FOM 011 the lateral view of a cerebral
angiogram (Will~ins, pp. 1427-1429; Youmans, pp. 1237-
1240).

How well did you know this?
1
Not at all
2
3
4
5
Perfectly
116
Q

Figure 8.108-8.116Q depicts an endoscopic
approach to the third ventricle. Iatch the following ana tomic
structures to the corresponding letterhead , using each
answer either once, more than once, or not at all

Caudate nucleus

A

I

Figure 8.108-8.116Q demonstrates the surgical anatomy of
the right lateral and anterior third ventricle during endoscopic third ventriculostomy for aqueductal stenosis. The
ventriculostomy site is depicted in the floor of the third
ventricle in this figure. The veins of the third ventricular system collect into deeper veins that course in a subependymal
location as they travel through the margins of the choroidal
fissure to empty into the internal cerebral, basal, and great
veins. In general, the veins draining the frontal horn and
body of the third ventricle drain into the internal cerebral
vein (not depicted here) as it courses through the velum
interpositum; those draining the temporal horn drain into a
segment of the basal vein of Rosenthal coursing through the
ambient cistern; and the veins from the atrium drain into
segments of the basal, internal cerebral, and great veins
coursing through the quadrigeminal cistern. Of note , the
thalamostriate vein passes forward in the sulcus between the
caudate nucleus and thalamus toward the foramen of Monro
(FOM), where it turns sharply posterior to enter the velum
interpositum to join the internal cerebral vein. The angle
formed by the junction of the internal cerebral vein and
thalamostriate vein, referred to as the venous angle, approximates the level of the FOM 011 the lateral view of a cerebral
angiogram (Will~ins, pp. 1427-1429; Youmans, pp. 1237-
1240).

How well did you know this?
1
Not at all
2
3
4
5
Perfectly
117
Q

Injury to the thalamostriate vein during surgery may
produce which of the following complications?
1. Drowsiness
2. Hemorrhagic infarct in the basal ganglia
3. Hemiparesis
4. Mutism

A. 1,2,and3
B. 1 and 3
C. 2 and4
D. Only 4 is correct
E. All of the above

A

E. All of the above

Occlusion or injury of the thalamostriate vein may
cause drowsiness, hemiplegia, mutism, and hemorrhagic
infarction of the basal ganglia (Will~ins, pp. 1427-1429).

How well did you know this?
1
Not at all
2
3
4
5
Perfectly
118
Q

Characteristic microscopic fea tures of diffuse a.-xonal
injury (DAI) 12 to 24 hours after the insult may include
1. Astrogliosis
2. A;wnal retraction balls
3. Hemosiderin-laden macrophages
4. Perivascular hemorrhages

A. 1,2, and 3 are correct
B. 1 and 3 are correct
C. 2 and 4 are correct
D. Only 4 is correct
E. All of the above

A

**C. 2 and 4 are correct **

Acute microscopic changes after DAI typically include
axonal retraction balls and perivascular hemorrhages, while
in later stages there can be astrogliosis, endothelial proliferation, and accumulation of hemosiderin-laden macrophages
(Marion, pp. 40-45; Ellison, pp. 249- 257; Ramsey, pp. 431-
434).

How well did you know this?
1
Not at all
2
3
4
5
Perfectly
119
Q

How far below the iliac crest does the sciatic notch lie?
A. 3 to 4 cm
B. 4 to 5 cm
C. 7 to B cm
D. 10 to 12 cm
E. 14 cm

A

**C. 7 to B cm **

In approaching the posterior ilium during autogenous iliac bone graft harvesting, a limited incision that stays
within 8 cm of the posterior superior iliac spine typically
avoids the superior cluneal nerves. Dissection is then carried
down to the gluteal fascia, which should be opened directly
above the iliac crest to facilitate fascial closure. During subcrestal exposure, the lateral subperiosteal dissection should
be carried to the gluteus medius and tensor fascia lata muscles. Subperiosteal dissection usually avoids damage to the
superior gluteal artery, which courses through the musculature. The sciatic notch usually lies approximately 7 to 8 cm
below the iliac crest and must not be violated, as it harbors
the main trunk of the sciatic artery, the sciatic nerve , and the ureter, which runs ventral to the superior gluteal artery.
j’dedially, the dissection should extend to the iliacus muscle ,
which prevents injury to the iliohypogastric and ilioinguinal
nerves (Connolly, pp. 819- 820)

How well did you know this?
1
Not at all
2
3
4
5
Perfectly
120
Q

The etiology of the abnormality depicted on the MRJ
scan below (Figure B.120-B.122Q) is most likely
A. Iatrogenic
B. Infectious
C. Traumatic
D. Developmental
E. Neoplastic

A

**D. Developmental **

Note the absence of the corpus
callosum and the high-riding third ventricle on this sagittal
MRI depicting agenesis of the corpus callosum. This condition is usually not associated with Chiari I malformation
but rather with the Chiari II malformation (Osborn DN,
pp.29- 33).

How well did you know this?
1
Not at all
2
3
4
5
Perfectly
121
Q

What is the diagnosis ?
A. Diffuse axonal injury
B. Obstruction of the aqueduct of Sylvius
C. Meningitis
D. Agenesis of the corpus callosum
E. Obstruction of the foramen of Monro

A

D. Agenesis of the corpus callosum

Note the absence of the corpus
callosum and the high-riding third ventricle on this sagittal
MRI depicting agenesis of the corpus callosum. This condition is usually not associated with Chiari I malformation
but rather with the Chiari II malformation (Osborn DN,
pp.29- 33).

122
Q

Associated conditions may include all of the following
EXCEPT?
A. Schizencephaly
B. Chiari I malformation
C. Dandy-Walker malformation
D. Cephaloceles
E. Azygos anterior cerebral artery

A

**B. Chiari I malformation **

Note the absence of the corpus
callosum and the high-riding third ventricle on this sagittal
MRI depicting agenesis of the corpus callosum. This condition is usually not associated with Chiari I malformation
but rather with the Chiari II malformation (Osborn DN,
pp.29- 33).

123
Q

What are the most important ligaments for maintaining
atlantoaxial stability?
1. Transverse atlantal ligament (horizontal portion of
crllciate ligament)
2. Apical ligament
3. Alar ligaments
4. Inferior band of cruciate ligament

A. 1,2, and3 are correct
B. 1 and 3 are correct
C. 2 and4 are correct
D. Only 4 is correct
E. All of the above

A

**B. 1 and 3 are correct **

Stability at the atlantoaxial unit is mainly proVided
by the horizontal portion of the cruciate ligament (transverse atlantal ligament), and the paired alar ligaments
(atlantoalar portion) that connect the dens with the lateral
masses of CI. With disruption of these ligaments, the
remaining cruciate and apical ligaments are insufficient to
maintain stability. Ligaments that connect the a.us to the
occiput include the tectorial membrane (rostral continuation of the posterior longitudinal ligament), occipitoalar portion of the alar ligament, and the apical ligament (connects
the tip of dens to the foramen magnum), while the ligaments
connecting the atlas to the occiput include the tectorial
membrane and anterior longitudinal ligament (Greenberg,
p. 701; Youmans, pp. 3528-3551).

124
Q

A 45-year-old telemarketing agent has noticed
a gradual decline in his hearing over the course of a few
months and, only recently, some right ann wealmess. Physical
exam discloses full extraocular motion , symmetric facial
movements, a Weber test that lateralizes to the right, a Rinne
test revealing bone conduction better than air conduction on
the left Side, a uvula that deviates slightly to the right, and a
tongue that deviates to the left. His MRl is depicted below
(Figure 8.124-8.125Q).

\Vha t is the most likely diagnosis ?
A. Foramen magnum meningioma
B. Acoustic neuroma
C. Glomus jugulare tumor
D. Hemangiopericytoma
E. Chordoma

A

**C. Glomus jugulare tumor **

This patient harbors a glomus jugulare tumor,
which often presents with unilate ral hearing loss or pulsatile
tinnitus. Intracranial extension usually affects multiple
cranial nerves, which may result in a number of clinical
problems including dysphagia . Angiography is essential
because it helps with surgical planning. It helps delineate the
blood supply to the tumor as well as collateral blood flow
to the brain. Often preoperative embolization is a useful
adjunct for these highly vascular and locally invasive tumors.
Although observation with serial imaging studies to determine tumor progression may be an appropriate option for
medically frail patients, gross total resection is considered
the trea tment of choice for symptomatic lesions (Youmans,
pp.1295-1308).

125
Q

A 45-year-old telemarketing agent has noticed
a gradual decline in his hearing over the course of a few
months and, only recently, some right ann wealmess. Physical
exam discloses full extraocular motion , symmetric facial
movements, a Weber test that lateralizes to the right, a Rinne
test revealing bone conduction better than air conduction on
the left Side, a uvula that deviates slightly to the right, and a
tongue that deviates to the left. His MRl is depicted below
(Figure 8.124-8.125Q).

The best treatment strategy for this patient wound entail
A. Radiosurgery
B. Proton-beam radiation
C. Observation with seriallvlRl scans
D. Attempted gross total resec tion
E. Conventional external-beam radiation

A

**D. Attempted gross total resec tion **

This patient harbors a glomus jugulare tumor,
which often presents with unilate ral hearing loss or pulsatile
tinnitus. Intracranial extension usually affects multiple
cranial nerves, which may result in a number of clinical
problems including dysphagia . Angiography is essential
because it helps with surgical planning. It helps delineate the
blood supply to the tumor as well as collateral blood flow
to the brain. Often preoperative embolization is a useful
adjunct for these highly vascular and locally invasive tumors.
Although observation with serial imaging studies to determine tumor progression may be an appropriate option for
medically frail patients, gross total resection is considered
the trea tment of choice for symptomatic lesions (Youmans,
pp.1295-1308).

126
Q

What is the most likely mechanism accounting for the
fracture pattern depicted in Figure 8.126-8.127Q)?
A. Direct axial load on a neutral neck
B. Direct axial load on a flexed neck
C. Distraction
D. Direct axial load on a laterally bent necll
E. Hyperextension

A

**A. Direct axial load on a neutral neck **

The fracture-pattern (Jefferson fracture)
depicted on this axial CT scan most often results from an
axial load on a neutral neck. These patients are usually neurologically intact due to the large diameter of the spinal canal
at this level. If the sum of overhang of both lateral masses on
C2 is :2: 7 mm, the transverse ligament is probably disrupted,
which requires rigid immobilization (usually with a halo
vest) or surgical n.xation if additional fractures are present
(Greenberg, pp. 702-703).

127
Q

What amount of C1 lateral l11ass excursion beyond the
axis indicates transverse ligament disruption ?
A. 4111111
B. 5 111111
C. 6 111m
**D. 7 nUll **
E. 9 nun

A

**D. 7 nUll **

The fracture-pattern (Jefferson fracture)
depicted on this axial CT scan most often results from an
axial load on a neutral neck. These patients are usually neurologically intact due to the large diameter of the spinal canal
at this level. If the sum of overhang of both lateral masses on
C2 is :2: 7 mm, the transverse ligament is probably disrupted,
which requires rigid immobilization (usually with a halo
vest) or surgical n.xation if additional fractures are present
(Greenberg, pp. 702-703).

128
Q

A lesion of the facial nerve distal to the geniculate
ganglion but proximal to the stylomastoid foramen will
produce all of the following deficits EXCEPT?
A. Inability to wrinlde the forehead
B. Diminished corneal reflex
C. Impairment of sublingual and submandibular gland
secretions
D. Hyperacusis
E. Impaired lacrimation

A

E. Impaired lacrimation

A lesion of the facial nerve distal to the geniculate ganglion but proximal to the stylomastOid foramen typically results in complete paralysis of all ipsilateral fmuscles, a diminished corneal reflex (with preserved corneal
sensation, CN V) , impaired sublingual and submandibular
salivary gland secretions, hyperacusis, and frequently loss of
taste in the anterior two-thirds of the tongue ipsilaterally.
Hyperacusis results from paralysis of the stapedius muscle,
while salivary secretions are impaired due to the interruption of preganglionic parasympathetic fibers. Lesions proximal to the geniculate ganglion produce all of the disturbances
described above and invariably loss of taste in the anterior
two-thirds of the tongue and decreased lacrimation. This
lesion interrupts all SVA fibers that course centrally and
all preganglionic (GVE) fibers as they pass to the pterygopalatine and submandibular ganglia . Taste is permanently
lost and no regeneration of sensory fibers takes place.
Preganglionic parasympathetic fibers may regenerate , but
this may occur in an aberrant fashion. Fibers that previously
projected to the submandibular ganglion may regrow and
enter the greater petrosal nerve, which may result in
lacrimation after a salivary stimulus (“crocodile tears”)
(Carpenter, pp. 172-173).

129
Q

“Crocodile tears” results from aberrant regeneration of
what fibers ?
A. Parasympathetic fibers that previously projected to
the submandibular ganglion may regrow and ente r the
greater petrosal nerve

B. Parasympathetic fibers that previously projected to
the submandibular ganglion may regrow and enter the
lesser petrosal nerve
C. Parasympathetic fibers from the Edinger-Westphal
nucleus aberrantly project to the greater petrosal
nerve
D. Trigeminal nerve fibers that aberrantly project to the
su perior saliva tory nucleus
E. Parasympathetic fibers that previously projected to
the lacrimal gland may regrow and enter the lesser
petrosal nerve

A

A. Parasympathetic fibers that previously projected to
the submandibular ganglion may regrow and ente r the
greater petrosal nerve

A lesion of the facial nerve distal to the geniculate ganglion but proximal to the stylomastOid foramen typically results in complete paralysis of all ipsilateral fmuscles, a diminished corneal reflex (with preserved corneal
sensation, CN V) , impaired sublingual and submandibular
salivary gland secretions, hyperacusis, and frequently loss of
taste in the anterior two-thirds of the tongue ipsilaterally.
Hyperacusis results from paralysis of the stapedius muscle,
while salivary secretions are impaired due to the interruption of preganglionic parasympathetic fibers. Lesions proximal to the geniculate ganglion produce all of the disturbances
described above and invariably loss of taste in the anterior
two-thirds of the tongue and decreased lacrimation. This
lesion interrupts all SVA fibers that course centrally and
all preganglionic (GVE) fibers as they pass to the pterygopalatine and submandibular ganglia . Taste is permanently
lost and no regeneration of sensory fibers takes place.
Preganglionic parasympathetic fibers may regenerate , but
this may occur in an aberrant fashion. Fibers that previously
projected to the submandibular ganglion may regrow and
enter the greater petrosal nerve, which may result in
lacrimation after a salivary stimulus (“crocodile tears”)
(Carpenter, pp. 172-173).

130
Q

The mammillothalamic tract contains fibers that project from the medial mammillary nucleus to what stru
A. Pulvinar
B. Anterior thalamic nucleus
C. Dorsal and ventral tegmental nuclei
D. Centromedian (C1'l) nucleus of the thalamus
E. Ventral posterior thalamic nucleus

A

**B. Anterior thalamic nucleus **

B. ~l’fanllllillary fibers projecting from the medialmammillary nucleus and, to a lesser extent, intermediate and
lateral mammillary nuclei form a fiber bundle called the
fasciculus mammillaris princes, which divides into two
components: the mammillothalamic tract and mammillotegmental tract. The mammillothalamic tract contains fibers
that originate from tbe medial mamnlillary nucleus and
project to the anterior thalamic nucleus. Fibers from the
hippocampus are also superimposed on this fiber bundle as
they travel to the anterior thalamic nucleus through the
fornLx. The mammillotegmental tract terminates in the
dorsal and ventral tegmental nuclei (Carpenter, p. 309).

131
Q

Crossed fibers from the fastigial nucleus emerge from
the cerebellum through what structure?
A. Fastigial-tectal tract
B. Uncinate fasciculus (of Russell)
C. Juxtarestifonl1 body
D. Middle cerebellar peduncle
E. Fastigial-rubral tract

A

**B. Uncinate fasciculus (of Russell) **

Crossed fibers from the fastigial nucleus emerge from
the cerebellum through the uncinate fasciculus of Russell,
which arches around the superior cerebellar peduncle.
Uncrossed fastigial efferents project to the brainstem in the
juxtarestifonn body. The largest number of fastigial efferents
project to structures in the lower brainstem (nucleus reticularis gigantocellularis, central pontine reticular formation,
dorsal paramedian reticular nucleus). A small number of
fibers ascend in the dorsolateral brainstem and send collaterals to the superior colliculus and nuclei of the posterior
commissure prior to terminating in various thalamic regions
(VLc and VPLo) (Carpenter, pp. 241- 243)

132
Q

Some excitatory effects of the sympathetic nervous
system that lack parasympathetic opposition include
1. Splenic capsule contraction
2. Sweating and piloerection
3. Elevation of the upper eyelid
4. Constriction of the bladder wall

A. 1,2, and 3 are correct
B. 1 and3 are correct
C. 2 and 4 are correct
D. Only 4 is correct
E. All of the above

A

**A. 1,2, and 3 are correct **

Some excitatory effects of the sympathetic nervous
system that lack parasympathetic opposition include splenic
capsule contraction, sweating and piloerection, and elevation of the upper eyelid by the superior tarsal muscle
of JVluller. Gastrointestinal peristalsis, bladder wall contraction, bronchial size, pupillary diameter, heart rate, and blood
pressure can all be oppositionally altered by the sympathetic
and parasympathetic nervous system (DeMyer, p. 92).

133
Q

What is the most likely mechanism of injury of the
abnormality depicted below (Figure 8.133-8.134Q)?
A. Hyperextension and ax ial loading
B. Ax.ialload on a neutral neck
C. A .. ‘tial loading on a laterally bent neck
D. Severe hyperflexion and axial loading
E. Hyperextension and distraction

A

**A. Hyperextension and axial loading **

The mechanism of most modern hangman’s
fractures results from hyperextension and axial loading
(diving and motor vehicle aCCidents), while judicial hangings
often resulted in hyperextension and distractive forces from
submeatal Imot placement. Patients rarely require surgical
intervention for this type of fracture, which is typically
reserved for irreducible fractures, failure of external immobilization, traumatic C2-3 disc herniation (with canal compromise and progressive neurologic deficit), and established
nonunion. All fusion techniques typically involve fusing C2
to C3, although the majority of patients can be successfully
treated with a cervical collar, SOMI brace, or halo vest (most
common) (Greenberg, pp. 704-706).

134
Q

Therapeutic options may include all of the following
EXCEPT?
A. Cervical collar
B. Halo vest immobilization
C. Open reduc tion with internal n….:ation (C2 to C3)
D. Odontoid screw
E. SOMI brace

A

**D. Odontoid screw **

The mechanism of most modern hangman’s
fractures results from hyperextension and axial loading
(diving and motor vehicle aCCidents), while judicial hangings
often resulted in hyperextension and distractive forces from
submeatal Imot placement. Patients rarely require surgical
intervention for this type of fracture, which is typically
reserved for irreducible fractures, failure of external immobilization, traumatic C2-3 disc herniation (with canal compromise and progressive neurologic deficit), and established
nonunion. All fusion techniques typically involve fusing C2
to C3, although the majority of patients can be successfully
treated with a cervical collar, SOMI brace, or halo vest (most
common) (Greenberg, pp. 704-706).

135
Q

All of the following are associated with IGippel-Feil
syndrome EXCEPT?
A. Sprengel’s deformity
B. Unilateral absence of a kidney
C. Deafness
D. Scoliosis
E. Naevus flammeus

A

E. Naevus flammeus

IGippel-Feil syndrome ranges from fusion of only a
few vertebral bodies to fusion of the entire spine. It results
from failure of somite segregation between 3 and 8 weeks’
gestation. The classic triad (usually present in < 50% of
people) includes low posterior hairline , shortened neck
(brevicollis), and limited necl( motion. It may occur in conjunction with various other abnormalities includil1g basilar
impression, atlantoaxial fusion, facial asymmetry, torticollis,
pterygium colli (neck webbing), scoliosis, and Sprengel’s
deformity (25% of cases). Sprengel’s deformity results in
a raised scapula due to failed migration from its region of
formation in the neck. Systemic congenital abnormalities
may also occur, including deafness (30%), unilateral absence
of a kidney, and cardiopulmonary complications. Treatment
is usually directed at detecting and managing the systemic
anomalies, which typically consists of a cardiac evaluation
(ECG), CXR, and a renal ultrasound. Nevus t1ammeus is
usually seen in patients with tethered cord syndrome
(Greenberg, p. 158)

136
Q

Match the fontanelle with the approximate age of
closure, using each answer once, more than once, or not at all

Anterior fontanelle
A. 2 to 3 months
B. 1 year
C. 2.5 years
D. None of the above

A

**C. 2.5 years **

The anterior fontanelle , the
largest fontanelle, is diamond-shaped and normally closes by
2.5 years of age. The posterior fontanelle is triangle-shaped
and closes by 2 to 3 months. The sphenoid and mastoid
fontanelles are smaller, more irregular, and usually close by
2 to 3 months and! year, respectively (Greenberg, p. 138).

137
Q

Match the fontanelle with the approximate age of
closure, using each answer once, more than once, or not at all

Posterior fontanelle
A. 2 to 3 months
B. 1 year
C. 2.5 years
D. None of the above

A

**A. 2 to 3 months **

The anterior fontanelle , the
largest fontanelle, is diamond-shaped and normally closes by
2.5 years of age. The posterior fontanelle is triangle-shaped
and closes by 2 to 3 months. The sphenoid and mastoid
fontanelles are smaller, more irregular, and usually close by
2 to 3 months and! year, respectively (Greenberg, p. 138).

138
Q

Match the fontanelle with the approximate age of
closure, using each answer once, more than once, or not at all

Sphenoid fontanelle
A. 2 to 3 months
B. 1 year
C. 2.5 years
D. None of the above

A

**A. 2 to 3 months **

The anterior fontanelle , the
largest fontanelle, is diamond-shaped and normally closes by
2.5 years of age. The posterior fontanelle is triangle-shaped
and closes by 2 to 3 months. The sphenoid and mastoid
fontanelles are smaller, more irregular, and usually close by
2 to 3 months and! year, respectively (Greenberg, p. 138).

139
Q

Match the fontanelle with the approximate age of
closure, using each answer once, more than once, or not at all

Mastoid fontanelle
A. 2 to 3 months
B. 1 year
C. 2.5 years
D. None of the above

A

**B. 1 year **

The anterior fontanelle , the
largest fontanelle, is diamond-shaped and normally closes by
2.5 years of age. The posterior fontanelle is triangle-shaped
and closes by 2 to 3 months. The sphenoid and mastoid
fontanelles are smaller, more irregular, and usually close by
2 to 3 months and! year, respectively (Greenberg, p. 138).

140
Q

Blood supply to the dura originates from what arteries?
1. Ophthalmic artery
2. Occipital artery
3. Vertebral artery
4. Maxillary artery

A. 1, 2, and3 are correct
B. 1 and 2 are correct
C. 2 and4 are correct
D. Only 4 is correct
E. All of the above

A

**E. All of the above **

Blood supply to the dura originates from the middle
meningeal artery, which is a branch of the maxillary artery.
It enters the skull through the foramen spinosum. The ophthalmic artery (anterior meningeal branches) and OCCipital
and vertebral arteries (posterior meningeal arteries) also
provide meningeal branches to the dura (Carpenter, pp. 1- 2).

141
Q

How many great vessels originate from the aortic arch?
A. 2
B. 3
C. 4
D. 5
E. 6

A

**B. 3 **

The typical order of the three great vessels that originate from the aortic arch is the brachiocephalic trunk
(innominate artery) followed by the left common carotid
artery (LCCA) and then left subclavian artery (LSCA). This
configuration is found in approximately two-thirds of all
cases. A shared origin of the brachiocephalic trunk and
left common carotid artery is seen in 27% (most frequent
variant) of cases, while in 7% of cases the left common
carotid artery arises from the proximal brachiocephalic
artery instead of the aortic arch. In 1 to 2% of cases, the
LCCA and LSCA share a common origin and form a left-sided
brachiocephalic trunk (Osborn DCA, p. 16)

142
Q

A 38-year-old female has been complaining of ptosis and
intermittent diplopia for approximately 4 months. She has no
extremity or respiratory muscle weakness, but antibodies to
myofibrillar proteins were found during her workup. All of the
following statements about this condition are tme EXCEPT?
A. Diagnosis can be confirmed with the administration
of edrophonil1ll1 chloride, which reverses the muscle
wealmess
B. Single-fiber electromyography (EMG) may show
“jitter” and “blocking”
C. Signs of denervation are often seen on standard Ei’“IG
during the later stages of the disease
D . . Antibodies to myofibrillar proteins may precede
clinical symptomatology
E. Prednisone, plasmapheresis, andlor IVIG therapy may
be used for patients with generalized disease or respiratory crisis

A

C. Signs of denervation are often seen on standard Ei’“IG
during the later stages of the disease

Formal diagnosis of myasthenia gravis depends on
demonstration of response to cholinergic medications, EMG
evidence of abnormal neuromuscular transmission (“jitter,”
“bloclting”), and identification of Circulating antibodies to
Ach receptors or myofibrillar proteins such as actin, titin,
myosin, and actinomycin (in approximately 85 to 90% of
cases). In Single-muscle-fiber EMG studies, an electrode
measures the interval between evoked potentials of muscle
fibers in the same unit. This interval normally varies by
“jitter”, for which the temporal limits have been defined. In
myasthenia gravis, “jitter” is increased. If muscle fibers are
not activated due to abnormal neuromuscular transmission,
it is called “bloclting.” Ivlyasthenia gravis is characterized by
both “bloclting” and increased “jitter,” although these findings are not specific for myasthenia gravis, as other disorders
may show a similar response on EMG studies. Signs of denervation are almost never seen in disorders of ACh release
unless other conditions supervene. Cholinergic medications
should be stopped once an endotracheal tube has been
placed to reduce the amount of pulmonary secretions.
Prednisone, plasmapheresis, and IVIG therapy have all been
used in patients with generalized disease and/or respiratory crisis. Thymectomy is generally recommended for patients
with generalized myasthenia gravis, not ocular myasthenia
(Merritt, pp. 723- 726).

143
Q

Match the disorder (mucopolysaccharidosis)
with the enzyme abnormality, using each answer once, more
than once, or not at all

Sulfatase B
A. I-Iurler
B. Hunter
C. Sanfilippo A
D. Sanfilippo B
E. MorquioA
F. Morquio B
G. Maroteaux-Lamy
H. Sly
I. None of the above

A

**G. Maroteaux-Lamy **

144
Q

Match the disorder (mucopolysaccharidosis)
with the enzyme abnormality, using each answer once, more
than once, or not at all

p-Galactosidase
A. I-Iurler
B. Hunter
C. Sanfilippo A
D. Sanfilippo B
E. MorquioA
F. Morquio B
G. Maroteaux-Lamy
H. Sly
I. None of the above

A

**F. Morquio B **

145
Q

Match the disorder (mucopolysaccharidosis)
with the enzyme abnormality, using each answer once, more
than once, or not at all

Sulfamidase
A. I-Iurler
B. Hunter
C. Sanfilippo A
D. Sanfilippo B
E. MorquioA
F. Morquio B
G. Maroteaux-Lamy
H. Sly
I. None of the above

A

**C. Sanfilippo A **

146
Q

Match the disorder (mucopolysaccharidosis)
with the enzyme abnormality, using each answer once, more
than once, or not at all

Galactose-6-sulfate sulfatase
A. I-Iurler
B. Hunter
C. Sanfilippo A
D. Sanfilippo B
E. MorquioA
F. Morquio B
G. Maroteaux-Lamy
H. Sly
I. None of the above

A

**E. MorquioA **

147
Q

Match the disorder (mucopolysaccharidosis)
with the enzyme abnormality, using each answer once, more
than once, or not at all

Iduronate-2-sulfate sulfatase
A. I-Iurler
B. Hunter
C. Sanfilippo A
D. Sanfilippo B
E. MorquioA
F. Morquio B
G. Maroteaux-Lamy
H. Sly
I. None of the above

A

**B. Hunter **

148
Q

Match the disorder (mucopolysaccharidosis)
with the enzyme abnormality, using each answer once, more
than once, or not at all

a-L-Iduronidase
A. I-Iurler
B. Hunter
C. Sanfilippo A
D. Sanfilippo B
E. MorquioA
F. Morquio B
G. Maroteaux-Lamy
H. Sly
I. None of the above

A

**A. I-Iurler **

149
Q

Which of the following structures sends the largest
number of projections to the striatum?
A. Subthalamic nucleus
B. Cerebral cortex
C. Substantia nigra
D. Cerebellum
E. Spinal cord

A

**B. Cerebral cortex **

The largest quantity of afferents projecting to the
striatum originate in the cerebral cortex, although the amygdala, intralaminar nuclei of the thalamus, the substantia
nigra , and dorsal raphe nucleus send fibers to various parts of
the striatum as well (Carpenter, pp. 332 - 336).

150
Q

Match each of the following lesion sites with the
appropriate clinical picture, using each answer either once,
more than once, or not at all

Athetosis
A. .Amygdala
B. Substantia nigra
C. Vicinity of red nucleus
D. Status marmoratus of the corpus striatum and
thalamus
E. Subthalamic nucleus

A

**D. Status marmoratus of the corpus striatum and
thalamus **

Athetoid movements are
slow, writhing movements predominately of the hand and
wrist, while dystonia characteristically affects the axial
muscles. The lesion “status marmoratus” characterizes one
form of cerebral palsy, which consists of a marbled appearance of the corpus striatum and thalamus secondary to
perinatal hypoxia. Hypoxic damage in these areas results in
overgrowth of astrocytes as part of the healing process.
Oligodendrocytes then mistakenly myelillate astrocytic processes, which causes white patches to appear in the nuclei,
giving them a marbled appearance . Damage to the striatum
and thalamus from other causes such as infarcts may also
result in athetosis. Lesions of the substantia nigra can result
in rigidity and resting tremor, which are some of the hallmarks of Parkinson’s disease. Lesions of the subthalami

151
Q

Match each of the following lesion sites with the
appropriate clinical picture, using each answer either once,
more than once, or not at all

Rigidity and resting tremor
A. .Amygdala
B. Substantia nigra
C. Vicinity of red nucleus
D. Status marmoratus of the corpus striatum and
thalamus
E. Subthalamic nucleus

A

**B. Substantia nigra **

Athetoid movements are
slow, writhing movements predominately of the hand and
wrist, while dystonia characteristically affects the axial
muscles. The lesion “status marmoratus” characterizes one
form of cerebral palsy, which consists of a marbled appearance of the corpus striatum and thalamus secondary to
perinatal hypoxia. Hypoxic damage in these areas results in
overgrowth of astrocytes as part of the healing process.
Oligodendrocytes then mistakenly myelillate astrocytic processes, which causes white patches to appear in the nuclei,
giving them a marbled appearance . Damage to the striatum
and thalamus from other causes such as infarcts may also
result in athetosis. Lesions of the substantia nigra can result
in rigidity and resting tremor, which are some of the hallmarks of Parkinson’s disease. Lesions of the subthalami

152
Q

Match each of the following lesion sites with the
appropriate clinical picture, using each answer either once,
more than once, or not at all

CN III palsy and terminal tremor
A. .Amygdala
B. Substantia nigra
C. Vicinity of red nucleus
D. Status marmoratus of the corpus striatum and
thalamus
E. Subthalamic nucleus

A

C. Vicinity of red nucleus

Athetoid movements are
slow, writhing movements predominately of the hand and
wrist, while dystonia characteristically affects the axial
muscles. The lesion “status marmoratus” characterizes one
form of cerebral palsy, which consists of a marbled appearance of the corpus striatum and thalamus secondary to
perinatal hypoxia. Hypoxic damage in these areas results in
overgrowth of astrocytes as part of the healing process.
Oligodendrocytes then mistakenly myelillate astrocytic processes, which causes white patches to appear in the nuclei,
giving them a marbled appearance . Damage to the striatum
and thalamus from other causes such as infarcts may also
result in athetosis. Lesions of the substantia nigra can result
in rigidity and resting tremor, which are some of the hallmarks of Parkinson’s disease. Lesions of the subthalami

153
Q

Match each of the following lesion sites with the
appropriate clinical picture, using each answer either once,
more than once, or not at all

IGiiver-Bucy syndrome
A. .Amygdala
B. Substantia nigra
C. Vicinity of red nucleus
D. Status marmoratus of the corpus striatum and
thalamus
E. Subthalamic nucleus

A

**A. .Amygdala **

Athetoid movements are
slow, writhing movements predominately of the hand and
wrist, while dystonia characteristically affects the axial
muscles. The lesion “status marmoratus” characterizes one
form of cerebral palsy, which consists of a marbled appearance of the corpus striatum and thalamus secondary to
perinatal hypoxia. Hypoxic damage in these areas results in
overgrowth of astrocytes as part of the healing process.
Oligodendrocytes then mistakenly myelillate astrocytic processes, which causes white patches to appear in the nuclei,
giving them a marbled appearance . Damage to the striatum
and thalamus from other causes such as infarcts may also
result in athetosis. Lesions of the substantia nigra can result
in rigidity and resting tremor, which are some of the hallmarks of Parkinson’s disease. Lesions of the subthalami

154
Q

Match each of the following lesion sites with the
appropriate clinical picture, using each answer either once,
more than once, or not at all

Hemiballism
A. .Amygdala
B. Substantia nigra
C. Vicinity of red nucleus
D. Status marmoratus of the corpus striatum and
thalamus
E. Subthalamic nucleus

A

**E. Subthalamic nucleus **

Athetoid movements are
slow, writhing movements predominately of the hand and
wrist, while dystonia characteristically affects the axial
muscles. The lesion “status marmoratus” characterizes one
form of cerebral palsy, which consists of a marbled appearance of the corpus striatum and thalamus secondary to
perinatal hypoxia. Hypoxic damage in these areas results in
overgrowth of astrocytes as part of the healing process.
Oligodendrocytes then mistakenly myelillate astrocytic processes, which causes white patches to appear in the nuclei,
giving them a marbled appearance . Damage to the striatum
and thalamus from other causes such as infarcts may also
result in athetosis. Lesions of the substantia nigra can result
in rigidity and resting tremor, which are some of the hallmarks of Parkinson’s disease. Lesions of the subthalami

155
Q

Arteries that supply the thalamus include?
1. Anterior choroidal artery
2. Posterior communicating artery
3. Medial posterior choroidal artery
4. Basilar artery

A. 1,2, and J are correct
B. 1 and J are correct
C. 2 and 4 are correct
D. Only 4 is correct
E. All of the above

A

E. All of the above

The thalamus receives its arterial supply from
the anterior choroidal artery (lCA) and thalamoperforating arteries (PComA and basilar artery) as well as the
thalamogeniculate and posterior choroidal arteries (PCA).
Infarctions may occur in each of these thalamic territories and cause various clinical syndromes, depending on
which thalamic nuclei are involved (Carpenter, pp. 441-
450).

156
Q

What is the most ill,ely diagnosis?
A. Toxoplasmosis
B. Alzheimer’s disease
C. Neurocysticercosis
D. Echinococctls
E. Spinal cord injury

A

**B. Alzheimer’s disease **

Note the prominent diffuse plaques traversed by
neuronal processes in this patient with Alzheimer’s disease .
Diffuse amyloid plaques are extracellular, ill-defined focal
aggregates of amyloid and preamyloid material and are
approximately 60 to 300 mm in diameter. Neuronal cell
processes traversing the plaque typically appear normal
and do not contain tau protein (Ellison, pp. 553- 557).

157
Q

Match the cervical spine fracture with the most
likely mechanism, using each response once, more than
once, or not at all.

Teardrop fracture
A. A.”jalloading/compression on neutral neck
B. Hyperextension and axial loading
C. Hyperextension and distraction
D. Hyperflexion and axial rotation
E. I-Iyperflexion and compression
F. Hypertlexion and distraction
G. I-Iypertlexion
H. Lateral bending and compression
I. None of the above

A

**E. I-Iyperflexion and compression **

Occipital
condylar and burst fractures are often the result of purely
axial compressive forces on a neutral neck, while teardrop
and compression-wedge fractures result from compressive
force on a flexed neck Other injury or fracture patterns in
the cervical spine include unilateral (axial rotation, flexing)
and bilateral locked (hyperflexion and distraction) facets,
Chance fracture (flexion, distraction), and unilateral facet
fracture (lateral bending, compression). Odontoid fractures
usually result from hyperflexion injuries and result in anterior displacement of C2 on C3. (Greenberg, pp. 702- 714;
Youmans, pp. 517- 523, 4896- 4897, 4927; Harris, pp. 69-
91).

158
Q

Match the cervical spine fracture with the most
likely mechanism, using each response once, more than
once, or not at all.

Unilateral jumped facet
A. A.”jalloading/compression on neutral neck
B. Hyperextension and axial loading
C. Hyperextension and distraction
D. Hyperflexion and axial rotation
E. I-Iyperflexion and compression
F. Hypertlexion and distraction
G. I-Iypertlexion
H. Lateral bending and compression
I. None of the above

A

**D. Hyperflexion and axial rotation **

Occipital
condylar and burst fractures are often the result of purely
axial compressive forces on a neutral neck, while teardrop
and compression-wedge fractures result from compressive
force on a flexed neck Other injury or fracture patterns in
the cervical spine include unilateral (axial rotation, flexing)
and bilateral locked (hyperflexion and distraction) facets,
Chance fracture (flexion, distraction), and unilateral facet
fracture (lateral bending, compression). Odontoid fractures
usually result from hyperflexion injuries and result in anterior displacement of C2 on C3. (Greenberg, pp. 702- 714;
Youmans, pp. 517- 523, 4896- 4897, 4927; Harris, pp. 69-
91).

159
Q

Match the cervical spine fracture with the most
likely mechanism, using each response once, more than
once, or not at all.

Burst fracture
A. A.”jalloading/compression on neutral neck
B. Hyperextension and axial loading
C. Hyperextension and distraction
D. Hyperflexion and axial rotation
E. I-Iyperflexion and compression
F. Hypertlexion and distraction
G. I-Iypertlexion
H. Lateral bending and compression
I. None of the above

A

**A. A.”jalloading/compression on neutral neck **

Occipital
condylar and burst fractures are often the result of purely
axial compressive forces on a neutral neck, while teardrop
and compression-wedge fractures result from compressive
force on a flexed neck Other injury or fracture patterns in
the cervical spine include unilateral (axial rotation, flexing)
and bilateral locked (hyperflexion and distraction) facets,
Chance fracture (flexion, distraction), and unilateral facet
fracture (lateral bending, compression). Odontoid fractures
usually result from hyperflexion injuries and result in anterior displacement of C2 on C3. (Greenberg, pp. 702- 714;
Youmans, pp. 517- 523, 4896- 4897, 4927; Harris, pp. 69-
91).

160
Q

Match the cervical spine fracture with the most
likely mechanism, using each response once, more than
once, or not at all.

Bilateral facet dislocation
A. A.”jalloading/compression on neutral neck
B. Hyperextension and axial loading
C. Hyperextension and distraction
D. Hyperflexion and axial rotation
E. I-Iyperflexion and compression
F. Hypertlexion and distraction
G. I-Iypertlexion
H. Lateral bending and compression
I. None of the above

A

**F. Hypertlexion and distraction **

Occipital
condylar and burst fractures are often the result of purely
axial compressive forces on a neutral neck, while teardrop
and compression-wedge fractures result from compressive
force on a flexed neck Other injury or fracture patterns in
the cervical spine include unilateral (axial rotation, flexing)
and bilateral locked (hyperflexion and distraction) facets,
Chance fracture (flexion, distraction), and unilateral facet
fracture (lateral bending, compression). Odontoid fractures
usually result from hyperflexion injuries and result in anterior displacement of C2 on C3. (Greenberg, pp. 702- 714;
Youmans, pp. 517- 523, 4896- 4897, 4927; Harris, pp. 69-
91).

161
Q

Match the cervical spine fracture with the most
likely mechanism, using each response once, more than
once, or not at all.

Unilateral facet fracture
A. A.”jalloading/compression on neutral neck
B. Hyperextension and axial loading
C. Hyperextension and distraction
D. Hyperflexion and axial rotation
E. I-Iyperflexion and compression
F. Hypertlexion and distraction
G. I-Iypertlexion
H. Lateral bending and compression
I. None of the above

A

**H. Lateral bending and compression **

Occipital
condylar and burst fractures are often the result of purely
axial compressive forces on a neutral neck, while teardrop
and compression-wedge fractures result from compressive
force on a flexed neck Other injury or fracture patterns in
the cervical spine include unilateral (axial rotation, flexing)
and bilateral locked (hyperflexion and distraction) facets,
Chance fracture (flexion, distraction), and unilateral facet
fracture (lateral bending, compression). Odontoid fractures
usually result from hyperflexion injuries and result in anterior displacement of C2 on C3. (Greenberg, pp. 702- 714;
Youmans, pp. 517- 523, 4896- 4897, 4927; Harris, pp. 69-
91).

162
Q

Match the cervical spine fracture with the most
likely mechanism, using each response once, more than
once, or not at all.

Compression wedge fracture
A. A.”jalloading/compression on neutral neck
B. Hyperextension and axial loading
C. Hyperextension and distraction
D. Hyperflexion and axial rotation
E. I-Iyperflexion and compression
F. Hypertlexion and distraction
G. I-Iypertlexion
H. Lateral bending and compression
I. None of the above

A

**E. I-Iyperflexion and compression **

Occipital
condylar and burst fractures are often the result of purely
axial compressive forces on a neutral neck, while teardrop
and compression-wedge fractures result from compressive
force on a flexed neck Other injury or fracture patterns in
the cervical spine include unilateral (axial rotation, flexing)
and bilateral locked (hyperflexion and distraction) facets,
Chance fracture (flexion, distraction), and unilateral facet
fracture (lateral bending, compression). Odontoid fractures
usually result from hyperflexion injuries and result in anterior displacement of C2 on C3. (Greenberg, pp. 702- 714;
Youmans, pp. 517- 523, 4896- 4897, 4927; Harris, pp. 69-
91).

163
Q

Match the cervical spine fracture with the most
likely mechanism, using each response once, more than
once, or not at all.

OCCipital condylar fracture
A. A.”jalloading/compression on neutral neck
B. Hyperextension and axial loading
C. Hyperextension and distraction
D. Hyperflexion and axial rotation
E. I-Iyperflexion and compression
F. Hypertlexion and distraction
G. I-Iypertlexion
H. Lateral bending and compression
I. None of the above

A

**A. A.”jalloading/compression on neutral neck **

Occipital
condylar and burst fractures are often the result of purely
axial compressive forces on a neutral neck, while teardrop
and compression-wedge fractures result from compressive
force on a flexed neck Other injury or fracture patterns in
the cervical spine include unilateral (axial rotation, flexing)
and bilateral locked (hyperflexion and distraction) facets,
Chance fracture (flexion, distraction), and unilateral facet
fracture (lateral bending, compression). Odontoid fractures
usually result from hyperflexion injuries and result in anterior displacement of C2 on C3. (Greenberg, pp. 702- 714;
Youmans, pp. 517- 523, 4896- 4897, 4927; Harris, pp. 69-
91).

164
Q

Match the cervical spine fracture with the most
likely mechanism, using each response once, more than
once, or not at all.

Odontoid fracture
A. A.”jalloading/compression on neutral neck
B. Hyperextension and axial loading
C. Hyperextension and distraction
D. Hyperflexion and axial rotation
E. I-Iyperflexion and compression
F. Hypertlexion and distraction
G. I-Iypertlexion
H. Lateral bending and compression
I. None of the above

A

**G. I-Iypertlexion **

Occipital
condylar and burst fractures are often the result of purely
axial compressive forces on a neutral neck, while teardrop
and compression-wedge fractures result from compressive
force on a flexed neck Other injury or fracture patterns in
the cervical spine include unilateral (axial rotation, flexing)
and bilateral locked (hyperflexion and distraction) facets,
Chance fracture (flexion, distraction), and unilateral facet
fracture (lateral bending, compression). Odontoid fractures
usually result from hyperflexion injuries and result in anterior displacement of C2 on C3. (Greenberg, pp. 702- 714;
Youmans, pp. 517- 523, 4896- 4897, 4927; Harris, pp. 69-
91).

165
Q

What is depicted in the photomicrograph below
(Figure 8.165Q)?
A. Choroid plexus papilloma
B. Ependymoma
C. Adaman tinomatous craniopharyngioma
D. Chordoma
E. Angiomatous meningioma

A

**C. Adaman tinomatous craniopharyngioma **

Note the squamous cells, peripheral palisading of
nuclei, and nodules of wet keratin in this adamantinomatous
craniopharyngioma (Ellison, pp. 724-727; WHO, pp. 244-
246).

166
Q

All of the following entities may be considered in
the differential diagnosis of vertigo of peripheral origin
EXCEPT?
A. Meniere’s disease
B. Benign paroxysmal positional vertigo
C. Secondary endolymphatic hydrops
D. Vestibular neuronitis
E. Cralliovertebraljunction abnormality

A

E. Cralliovertebraljunction abnormality

Approximately 90% of cases of vertigo are likely secondary to lesions of the vestibular end organs or vestibular
nerves, while the rest usually originate from the central
nervous system, including craniocervical junction abnormalities (Merritt, pp. 28-30).

167
Q

Primarily branches of what nerve(s) innervate the
supratentorial dura?
1. Upper cervical spinal nerves
2. Glossopharyngealnerve
3. Vagus nerve
4. Trigeminal nerve

A. 1,2, and J are correct
B. 1 and J are correct
C. 2 and 4 are correct
D. Only 4 is correct
E. All of the above

A

**D. Only 4 is correct **

The supratentorial dura is innervated primarily by
branches of the trigeminal nerve, while the infra tentorial
dura is innervated by branches of the vagus and lower cervical spinal nerves (Carpenter, p. 2).

168
Q

A wedding singer from your town presents to your office
with a I-week history of necl( pain and slight left ann and
hand numbness. His IvIRI is depicted below (Figure 8.168-
8.169Q). What is the most likely diagnosis ?
A. Synovial cyst
B. Juxtafacet cyst
C. Hypertrophy of the ligamentum f1avum
D. Disc herniation
E. Ossification of the posterior longitudinal ligament
(OPLL)

A

**D. Disc herniation **

Depicted here is an axial and parasagittal MIU
showing a soft (mainly disc material) posterolateral disc
herniation of the cervical spine that is eccentric to the left.
Considering that this disc herniation is not purely central , it
may be runenable to a posterolateral procedure in an attempt
to preserve vocal cord function (posterior keyhole laminotomy) in this wedding singer. Over 90% of patients with
acute cervical radiculopathy will improve with nonsurgical
therapy including adequate pain medication and antiinllammatories. Surgery is indicated for those who fail to
improve or develop progressive neurologic deficits while
undergoing nonsurgical therapy. A number of large series
have reported good or excellent results in 90 to 96% of
patients who underwent a posterior approach for such disc
herniations (Greenberg, pp. 310-314).

169
Q

The patient’s symptomatology does not improve after
8 weeks of nonsurgical therapy. He returns to your office
to explore surgical options. You inform him that the best
surgical option for him should include what procedure, if
possible?
A. Anterior cervical discectomy and fusion
B. Posterior cervical laminectomy
C. Posterior keyhole laminotomy
D. Anterior cervical corpectomy and fusion
E. Costotransversectomy

A

C. Posterior keyhole laminotomy

Depicted here is an axial and parasagittal MIU
showing a soft (mainly disc material) posterolateral disc
herniation of the cervical spine that is eccentric to the left.
Considering that this disc herniation is not purely central , it
may be runenable to a posterolateral procedure in an attempt
to preserve vocal cord function (posterior keyhole laminotomy) in this wedding singer. Over 90% of patients with
acute cervical radiculopathy will improve with nonsurgical
therapy including adequate pain medication and antiinllammatories. Surgery is indicated for those who fail to
improve or develop progressive neurologic deficits while
undergoing nonsurgical therapy. A number of large series
have reported good or excellent results in 90 to 96% of
patients who underwent a posterior approach for such disc
herniations (Greenberg, pp. 310-314).

170
Q

Which of the following are associated with yolk sac
tumors?
1. AFP positivity
2. p-HCG positivity
3. Schiller-Duval bodies
4. Placental alkaline phosphatase positivity

A. 1,2, and 3 are correct
B. 1 and 3 are correct
C. 2 and 4 are correct
D. Only 4 is correct
E. All of the above

A

**B. 1 and 3 are correct **

Yolk sac tumor is a germ cell tumor that exhibits
loosely arranged cells with clear cytoplasm and prominent
eosinophilic bodies. Yolk sac tumors are positive for AFP,
and they often exhibit Schiller-Duval bodies (Ellison, p. 683).

171
Q

Patients with acromegaly require a meticulous preoperative anesthetic workup because they may often have
which of the following clinical findings?
1. lvlacroglossia
2. Elevated angiotensin-converting enzyme
3. Cardiomyopathy
4. Nasal telangiectasias

A. 1, 2, and 3 are correct
B. 1 and 3 are correct
C. 2 and 4 are correct
D. Only 4 is correct
E. All of the above

A

**B. 1 and 3 are correct **

IHeticulous preoperative anesthetic workup is
extremely important for acromegalic patients, since they
frequently exhibit cardiomyopathy and macroglossia, which
can be associated with a difficult airway (Youmans, p. 569).

172
Q

Huntington’s disease primarily affects which projections from the striatum to the external segment of the globus
pallidus?
A. Cholinergic
B. Adrenergic
C. GABA/enkephalin
D. Glutamate
E. Serotonin

A

**C. GABA/enkephalin **

HD primarily affects the GABA/enkephalin projections from the striatum to the external segment of the globus
pallidus (indirect pathway), resulting in thalamic facilitation
of motor cortical areas and hyperkinesia (Merritt, pp. 659-
662).

173
Q

What is depicted in Figure 8.173Q?
A. Pleomorphic xanthoactrocytoma
B. Ependymoma
C. Choriocarcinoma
D. Medulloblastoma
E. Adenohypophysis

A

E. Adenohypophysis

Note the prominent lobules of cells with an intervening vascular network of sinusoids in this photomicrograph of
a normal adenohypophysis (Ellison, p. 716).

174
Q

A newborn infant is diagnosed with Meckel-Gruber syndrome. What is the most likely etiology of this abnor
A. Maternal diabetes
B. Excessive alcohol consumption by the mother during
the third gestational week
C. J’vIaternal hyperthermia on gestational days 20 to 26
D. Folate deficiency
E. None of the above

A

**C. J’vIaternal hyperthermia on gestational days 20 to 26 **

Meckel-Gruber syndrome typically includes encephalocele, microcephaly, microphthalmia , cleft lip, polydactyly, polycystic kidneys, congenital heart disease , and
ambiguous genitalia; it is most often associated with maternal
hyperthermia on gestational days 20 to 26 (Rudolph, p. 156).

175
Q

The release of a single quantum of ace tylcholine produces what type of postsynaptic response ?
A. Endplate potential
B. Temporal summation
C. Spatial summation
D. J'liniature endplate potential
E. Temporal dispersion

A

**D. J'liniature endplate potential **

The release of a single quantum of acetylcholine
(10,000 molecules of ACh) produces a miniature endplate
potential (J’I’IEPP) in the postsynaptic membrane, which is
not significant enough to generate an action potential. The
summation of several quanta is required to induce enough
depolarization in the postsynaptic cell membrane to result in
an action potential (Kandel, pp. 255-262).

176
Q

Nitric oxide (NO) results in the generation of what
second messenger?
A. cAMP
B. cGMP
C. Diacylglycerol (DAG)
D. Inositol 1, 4, S-triphosphate (IP.,)
E. Protein kinase C

A

**B. cGMP **

Nitric oxide (NO) is produced in neurons by the
Ca2
+/calmodulin-dependent enzyme NO synthase. NO, in
turn, stimulates the synthesis of cGMP through the action of
guanylyl cyclase, an enzyme that converts GTP to cGMP. NO
acts locally and is primarily released from endothelial cells to
induce smooth muscle relaxation and blood vessel dilation
(Kandel, p. 239).

177
Q

Functions of the Golgi complex include all of the following EXCEPT which?
A. N- and O-linked glycosylation
B. Proteoglycan formation
C. Sulfation of arginine residues
D. Attachment of fatty acids to proteins
E. Polysaccharide phosphorylation

A

**C. Sulfation of arginine residues **

Neuronal cytoplasmic organelles include rough and
smooth endoplasmic reticulum, endosomes, secretory vesicles, lysosomes, peroxisomes, mitochondria, and the Golgi
apparatus. Proteins and phospholipids destined for secretion
are initially synthesized in the rough endoplasmic reticulum
(rER). These products are then transported to the Golgi
apparatus via transport vesicles for processing (although
N-linked glycosylation and glycolipid conjugation are initiated in the rER). The Golgi complex further modifies these
proteins by adding polysaccharides, which can direct specific
proteins to secretory vesicles, lysosomes, and the plasma
membrane. Golgi processing includes glycosylation reactions (O-linked and N-linked glycosylation), proteoglycan
formation, polysaccharide phosphorylation, attachment of
fatty acids, and sulfation of tyrosine (not arginine) and sugar
residues. This processing increases the hydrophilicity (solubility) of these proteins, increases their biological activity, or
helps delay their degradation by proteases. Clathrin coats
facilitate the budding of vesicles from the Golgi complex.
Secretory vesicles (dense-core vesicles) are targeted primarily to axon terminals, where they participate in calciumregulated exocytosis after action potential propagation
(Kandel, pp. 67-71, 94- 97)

178
Q

What is mainly responsible for the high selectivity of
the blood-brain barrier (BBB) ?
A. Astrocytic foot processes covering fenestrated
endothelial cells
B. Tight junctions between nonfenestrated endothelial
cells
C. The extensive basal lamina that surrounds endothelial
cells
D. The basement membrane, capillary endothelium, and
astrocytic foot processes
E. The exclusion of foreign antigens by a superselective
neuronal membrane

A

**B. Tight junctions between nonfenestrated endothelial
cells **

The BBB is primarily composed of tight junctions
between nonfenestrated endothelial cells. These endothelial
cells are also deficient in vesicular transport compared to
endothelial cells elsewhere in the body, which further contributes to the selectivity of the BBB. The resistance provided by the tight junctions between endothelial cells in the
brain is extremely high . Substances may cross the BBB by
diffUSion, active transport, carrier-mediated transport, and
through ion channels. Lipid-soluble substances readily diffuse across endothelial cell membranes into the brain; hence
the permeability of many substances is directly related to
their lipid-solubility. Specific carrier-mediated transport is
responsible for the entry of most substances into the brain.
The glucose transporter (Glutl) is energy-independent, thus
transporting only glucose down its concentration gradient
from the bloodstream into the brain. There are also three distinct carrier systems for amino acid transport across the
BBB. The L system transports large neutral branched-chain
amino acids and L-DOPA into the brain. A transport system
that is a member of the multiple-drug-resistance (MDR)
transporter family found in tumor cells removes a wide range
of hydrophobic toxins and chemotherapeutic agents from
the brain. Additionally, specific ion channels allow the movement of electrolytes across the BBB. A metabolic BBB also
exists due to the presence of certain enzymes that rapidly
metabolize substances as they enter the CNS. An example is
the high concentration of DOPA decarboxylase in epithelial
cells that rapidly metabolize L-DOPA as it enters the brain

179
Q

What structure of the brain has the highest concentration of substance P?
A. Thalamus
B. Pineal gland
C. Hypothalamus
D. Substantia nigra
E. Amygdala

A

**D. Substantia nigra **

The substantia nigra has the highest concentration
of substance P in the brain. Striatonigral projections from the
caudate nucleus and putamen contain GABA, substance P,
and en kephalin (Carpenter, pp. 215- 221).

180
Q

A 54-year-old male presents to the emergency department with a ge ne ralized tonic-clonic seizure and a 6-month
history of behavioral changes. His lateral angiogram is
depicted below (Figure 8.180- 8.181Q). What is the most
likely diagnosis ?
A. IIydrocephalus from a third ventricular tumor
obstructing the foramen of Monro
B. Glioblastoma
C. Olfactory groove meningioma
D. Craniopharyngioma
E. Arterial-venous malformation

A

**C. Olfactory groove meningioma **

Note that the anterior cerebral arteries are
being pushed upward on this lateral angiogram, depicting
an olfactory groove meningioma. These lesions often grow
insidiously, causing gradual compression of the frontal lobes;
thus they are quite large and bilateral by the time of presentation. Common signs/symptoms may include headaches,
personality changes, visual loss, anosmia, and seizures. A
bifrontal transbasal approach is frequently used in resecting
these tumors, although a unilateral subfrontal or frontotemporal (pterional) craniotomy can also be used. The anterior
and posterior ethmoid arteries typically supply olfactory
groove meningiomas (Kaye and Blacl(, pp. 523-532;
Youmans, pp. 1115-1115).

181
Q

The most common blood supply to this lesion originates
from what blood vessel(s) ?
A. Superior hypophyseal artery
B. Anterior meningeal branches from the maxillary
artery
C. Superficial temporal artery
D. Anterior meningeal branches from the cavernous
internal carotid artery
E. The anterior and posterior ethmoidal arteries

A

**E. The anterior and posterior ethmoidal arteries **

Note that the anterior cerebral arteries are
being pushed upward on this lateral angiogram, depicting
an olfactory groove meningioma. These lesions often grow
insidiously, causing gradual compression of the frontal lobes;
thus they are quite large and bilateral by the time of presentation. Common signs/symptoms may include headaches,
personality changes, visual loss, anosmia, and seizures. A
bifrontal transbasal approach is frequently used in resecting
these tumors, although a unilateral subfrontal or frontotemporal (pterional) craniotomy can also be used. The anterior
and posterior ethmoid arteries typically supply olfactory
groove meningiomas (Kaye and Blacl(, pp. 523-532;
Youmans, pp. 1115-1115).

182
Q

Match the disorder with the most common
inheritance pattern, using each answer once, more than
once, or not at all

Duchene muscular dystrophy (OMO)
A. X-linked recessive
B. Autosomal dominant
C. Autosomal recessive

A

A. X-linked recessive

Charcot-Ma rie-Tooth
(CMT) disease, or peroneal muscular atrophy, accounts for
about 90% of all hereditary neuropathies, of which there
are three types. ClI’IT-l, the most common , is autosomal
dominant and is a demyelinating disease that results in a
dista l sensorimotor neuropathy. It results from mutations in
peripheral myelin protein 22 (PMP-22), exhibits slowed
nerve conduction velocities, and exhibits onion-bulb formations on histopathologic studies. CMT-2 resembles C)'IT-l
but results in axonal degeneration instead of demyelination.
CMT-3 , also Imown as Dejerine-Sottas syndrome, is autosomal dominant and results in a hypertrophic demyelinating
neuropathy. DMD is an X-linked recessive condition that
results from mutation of the dystrophin gene, located at
Xp21. Patients often have difficulty rising from the ground
and rely heavily on the anTIS to raise the torso and legs
(Gowers’ sign). These patients may also exhibit toe-walking
and develop an exaggerated lumbar lordosis or scoliosis
as well as pseudohypertrophy of the calves from fibrosis
and fatty infiltration of degenerating muscle. MD is a trinucleotide repeat disorder that results from mutation of the
dystrophia myotonica protein kinase gene on chromosome
19. It is a pleiotropiC, autosomal dominant disorder affecting
the skeletal muscle, heart, eyes, and exocrine glands. The
myopathy of MD affects several cranial nerves (ptosis,
dysarthria, dysphagia) and the distal extremities (finger
flexors and extensors). Patients exhibit myotonia (impaired
muscle relaxation), cataracts, frontal balding, testicular
atrophy, retinal degeneration, cardiomyopathy, and an
increased incidence of mental retardation. FSHMD is an
autosomal dominant disorder that results in scapular winging, facial wealmess, shoulder girdle wealmess, and lower
extremity wealmess (Merritt, pp. 737- 746)

183
Q

Match the disorder with the most common
inheritance pattern, using each answer once, more than
once, or not at all

Fascioscapulohumeralmuscular dystrophy (FSI-IMO)
A. X-linked recessive
B. Autosomal dominant
C. Autosomal recessive

A

**B. Autosomal dominant **

Charcot-Ma rie-Tooth
(CMT) disease, or peroneal muscular atrophy, accounts for
about 90% of all hereditary neuropathies, of which there
are three types. ClI’IT-l, the most common , is autosomal
dominant and is a demyelinating disease that results in a
dista l sensorimotor neuropathy. It results from mutations in
peripheral myelin protein 22 (PMP-22), exhibits slowed
nerve conduction velocities, and exhibits onion-bulb formations on histopathologic studies. CMT-2 resembles C)'IT-l
but results in axonal degeneration instead of demyelination.
CMT-3 , also Imown as Dejerine-Sottas syndrome, is autosomal dominant and results in a hypertrophic demyelinating
neuropathy. DMD is an X-linked recessive condition that
results from mutation of the dystrophin gene, located at
Xp21. Patients often have difficulty rising from the ground
and rely heavily on the anTIS to raise the torso and legs
(Gowers’ sign). These patients may also exhibit toe-walking
and develop an exaggerated lumbar lordosis or scoliosis
as well as pseudohypertrophy of the calves from fibrosis
and fatty infiltration of degenerating muscle. MD is a trinucleotide repeat disorder that results from mutation of the
dystrophia myotonica protein kinase gene on chromosome
19. It is a pleiotropiC, autosomal dominant disorder affecting
the skeletal muscle, heart, eyes, and exocrine glands. The
myopathy of MD affects several cranial nerves (ptosis,
dysarthria, dysphagia) and the distal extremities (finger
flexors and extensors). Patients exhibit myotonia (impaired
muscle relaxation), cataracts, frontal balding, testicular
atrophy, retinal degeneration, cardiomyopathy, and an
increased incidence of mental retardation. FSHMD is an
autosomal dominant disorder that results in scapular winging, facial wealmess, shoulder girdle wealmess, and lower
extremity wealmess (Merritt, pp. 737- 746)

184
Q

Match the disorder with the most common
inheritance pattern, using each answer once, more than
once, or not at all

MyotOltiC muscular dystrophy (MO)
A. X-linked recessive
B. Autosomal dominant
C. Autosomal recessive

A

**B. Autosomal dominant **

Charcot-Ma rie-Tooth
(CMT) disease, or peroneal muscular atrophy, accounts for
about 90% of all hereditary neuropathies, of which there
are three types. ClI’IT-l, the most common , is autosomal
dominant and is a demyelinating disease that results in a
dista l sensorimotor neuropathy. It results from mutations in
peripheral myelin protein 22 (PMP-22), exhibits slowed
nerve conduction velocities, and exhibits onion-bulb formations on histopathologic studies. CMT-2 resembles C)'IT-l
but results in axonal degeneration instead of demyelination.
CMT-3 , also Imown as Dejerine-Sottas syndrome, is autosomal dominant and results in a hypertrophic demyelinating
neuropathy. DMD is an X-linked recessive condition that
results from mutation of the dystrophin gene, located at
Xp21. Patients often have difficulty rising from the ground
and rely heavily on the anTIS to raise the torso and legs
(Gowers’ sign). These patients may also exhibit toe-walking
and develop an exaggerated lumbar lordosis or scoliosis
as well as pseudohypertrophy of the calves from fibrosis
and fatty infiltration of degenerating muscle. MD is a trinucleotide repeat disorder that results from mutation of the
dystrophia myotonica protein kinase gene on chromosome
19. It is a pleiotropiC, autosomal dominant disorder affecting
the skeletal muscle, heart, eyes, and exocrine glands. The
myopathy of MD affects several cranial nerves (ptosis,
dysarthria, dysphagia) and the distal extremities (finger
flexors and extensors). Patients exhibit myotonia (impaired
muscle relaxation), cataracts, frontal balding, testicular
atrophy, retinal degeneration, cardiomyopathy, and an
increased incidence of mental retardation. FSHMD is an
autosomal dominant disorder that results in scapular winging, facial wealmess, shoulder girdle wealmess, and lower
extremity wealmess (Merritt, pp. 737- 746)

185
Q

Match the disorder with the most common
inheritance pattern, using each answer once, more than
once, or not at all

Charcot-Marie-Tooth disease
A. X-linked recessive
B. Autosomal dominant
C. Autosomal recessive

A

**B. Autosomal dominant **

Charcot-Ma rie-Tooth
(CMT) disease, or peroneal muscular atrophy, accounts for
about 90% of all hereditary neuropathies, of which there
are three types. ClI’IT-l, the most common , is autosomal
dominant and is a demyelinating disease that results in a
dista l sensorimotor neuropathy. It results from mutations in
peripheral myelin protein 22 (PMP-22), exhibits slowed
nerve conduction velocities, and exhibits onion-bulb formations on histopathologic studies. CMT-2 resembles C)'IT-l
but results in axonal degeneration instead of demyelination.
CMT-3 , also Imown as Dejerine-Sottas syndrome, is autosomal dominant and results in a hypertrophic demyelinating
neuropathy. DMD is an X-linked recessive condition that
results from mutation of the dystrophin gene, located at
Xp21. Patients often have difficulty rising from the ground
and rely heavily on the anTIS to raise the torso and legs
(Gowers’ sign). These patients may also exhibit toe-walking
and develop an exaggerated lumbar lordosis or scoliosis
as well as pseudohypertrophy of the calves from fibrosis
and fatty infiltration of degenerating muscle. MD is a trinucleotide repeat disorder that results from mutation of the
dystrophia myotonica protein kinase gene on chromosome
19. It is a pleiotropiC, autosomal dominant disorder affecting
the skeletal muscle, heart, eyes, and exocrine glands. The
myopathy of MD affects several cranial nerves (ptosis,
dysarthria, dysphagia) and the distal extremities (finger
flexors and extensors). Patients exhibit myotonia (impaired
muscle relaxation), cataracts, frontal balding, testicular
atrophy, retinal degeneration, cardiomyopathy, and an
increased incidence of mental retardation. FSHMD is an
autosomal dominant disorder that results in scapular winging, facial wealmess, shoulder girdle wealmess, and lower
extremity wealmess (Merritt, pp. 737- 746)

186
Q

Match the disorder with the most common
inheritance pattern, using each answer once, more than
once, or not at all

Oejerine-Sottas disease
A. X-linked recessive
B. Autosomal dominant
C. Autosomal recessive

A

**B. Autosomal dominant **

Charcot-Ma rie-Tooth
(CMT) disease, or peroneal muscular atrophy, accounts for
about 90% of all hereditary neuropathies, of which there
are three types. ClI’IT-l, the most common , is autosomal
dominant and is a demyelinating disease that results in a
dista l sensorimotor neuropathy. It results from mutations in
peripheral myelin protein 22 (PMP-22), exhibits slowed
nerve conduction velocities, and exhibits onion-bulb formations on histopathologic studies. CMT-2 resembles C)'IT-l
but results in axonal degeneration instead of demyelination.
CMT-3 , also Imown as Dejerine-Sottas syndrome, is autosomal dominant and results in a hypertrophic demyelinating
neuropathy. DMD is an X-linked recessive condition that
results from mutation of the dystrophin gene, located at
Xp21. Patients often have difficulty rising from the ground
and rely heavily on the anTIS to raise the torso and legs
(Gowers’ sign). These patients may also exhibit toe-walking
and develop an exaggerated lumbar lordosis or scoliosis
as well as pseudohypertrophy of the calves from fibrosis
and fatty infiltration of degenerating muscle. MD is a trinucleotide repeat disorder that results from mutation of the
dystrophia myotonica protein kinase gene on chromosome
19. It is a pleiotropiC, autosomal dominant disorder affecting
the skeletal muscle, heart, eyes, and exocrine glands. The
myopathy of MD affects several cranial nerves (ptosis,
dysarthria, dysphagia) and the distal extremities (finger
flexors and extensors). Patients exhibit myotonia (impaired
muscle relaxation), cataracts, frontal balding, testicular
atrophy, retinal degeneration, cardiomyopathy, and an
increased incidence of mental retardation. FSHMD is an
autosomal dominant disorder that results in scapular winging, facial wealmess, shoulder girdle wealmess, and lower
extremity wealmess (Merritt, pp. 737- 746)

187
Q

Which of the following is/a re correct about the common
peroneal nerve ?
1. Arises from the dOl’sal divisions of the sacral plexus
(L4, LS, Sl, and S2)
2. Begins at the rostral margin of the popliteal fossa
3. Follows the medial border of the biceps femoris muscle
4. Leaves the popliteal fossa by passing superficial to the
lateral head of the gastrocnemius muscle

A. 1,2, and3 are correct
B. 1 and3 are correct
C. 1 and 4 are correct
D. Only 4 is correct
E. All of the above

A

E. All of the above

The common peroneal nerve (L4-S2) innervates the
extensors and adductors of the leg (and part of the biceps
femoris) and gives rise to the latera l sural cutaneous nerve
(to the inferolateral leg), the deep peroneal ne rve, and the
superficial peroneal nerve. The deep peroneal nerve innervates the tibiaUs anterior, extensor hallucis longus, and
extensor cligitorum longus muscles. The superficial peroneal
nerve innervates the peroneus longus/brevis (foot eversion)
and the skin of the distal anterior leg, dorsum of the foot , and
digits. Lesions of the common peroneal nerve mainly result
in paralysis of dorsiflexion (footdrop) and foot eversion
(Patten, p. 311; Greenberg, pp. 522, 545).

188
Q

The most common pathology in mesial temporal lobe
epilepsy is hippocampal sclerosis. The least amount of damage is usually seen in what sector(s) of the hippocampus ?
A. CAl
B. CA2
C. CA3
D. CA4
E. A and 0 are correct

A

**B. CA2 **

Hippocampal sclerosis usually has a very characteristic pattern in patients with mesial temporal lobe epilepsy.
The greatest amount of damage is usually seen in the CAl
and CA4 sectors, while the least amount of damage occurs in
CA2. Synaptic reorganization of granule cell mossy fibe rs is
usually a characteristic feature of hippocampal sclerosis
(Committee on Education in Neurological Surgery, pp. 20, 110;
Mathem et al., pp. 105- 113).

189
Q

All of the following are correct about proximal (type II)
renal tubular acidosis EXCEPT?
A. The proximal renal tubule cannot reabsorb HCO:;
properly
B. There is usually low to normal serum 1<’
C. Nephrocalcinosis can accompany this disorder
D. Urine pI! < 6 during periods of acidosis
E. Often caused by toxic injury to the renal tubules
(heavy metals, Bence Jones proteins)

A

**C. Nephrocalcinosis can accompany this disorder **

Nephrocalcinosis frequently accompanies type I,
or distal, renal tubular acidosis (RTA). In type 1 RTA, the
proximal reabsorption of ICO~ is adequate, but the ability of
the distal tubule to secrete H+ ions is inadequate. The urine
pH remains above 5.5, and hypolwlemia, hypercalcemia ,
nephrocalcinosis, and osteomalacia frequently accompany
this abnormality. lvloderate amounts of bicarbonate therapy
may correct the acidosis. With type II RTA, the ability of the
proximal tubule to reabsorb HCO:; is compromised; HCO~ is
lost in the urine and acidemia develops secondary to the
inability of the distal tubule to reabsorb the flood of HCO~.
Eventually, the serum I-ICO:; decreases to a point whe re the
proximal renal tubule can reabsorb most of the reduced
HCO~ load, while the remainder is reclaimed in the distal
tubule, This maintains the urine pH < 5.5. W is frequently
low with proximal type II RT A. Type IV RTA is seen with
hyporeninemic hypoaldosteronism, which is characterized
by a mild acidosis associated with increased Ie levels compared to the low levels seen with types I and II RTA. The
elevated K+ suppresses ammonia production, which sustains
the acidosis (Fishman, pp. 138-139).

190
Q

What is the diagnosis of the lesion depicted in this
photomicrograph (Figure 8.190-8.192Q)?
A. Oysembryoplastic neuroepithelial tumor (ONT)
B. Central neurocytoma
C. Pleomorphic xanthoastrocytoma (PXA)
D. Giant cell astrocytoma (GCA)
E. Chordoid glioma of the third ventricle

A

A. Oysembryoplastic neuroepithelial tumor (ONT)

This lesion is most consistent with a DNT
(WIIO grade I). Patients usually present with long-standing
drug-resistant partial seizures that begin before the age of 20.
They are usually found in the temporal lobe or other supratentorial location and typically encompass the cerebral
cortex. On occasion, they appear to deform the overlying calvarium, a finding that furthe r supports the diagnosis of DNT.
The histologic hallmark of this tumor is the glioneuronal
element, which is shown here to consist of a free-floating
neuron in a microcyst surrounded by oligodendroglia I-like
cells (Ellison, pp. 659- 661; WHO, pp. 103-106).

191
Q

What is the histologic hallmark of this lesion?
A. Rosenthal fibers
B. Alzheimer I! astrocytes
C. Glioneuronal element
D. Synaptophysin reactivity
E. Bizarre-appearing nuclea ted cells

A

**C. Glioneuronal element **

This lesion is most consistent with a DNT
(WIIO grade I). Patients usually present with long-standing
drug-resistant partial seizures that begin before the age of 20.
They are usually found in the temporal lobe or other supratentorial location and typically encompass the cerebral
cortex. On occasion, they appear to deform the overlying calvarium, a finding that furthe r supports the diagnosis of DNT.
The histologic hallmark of this tumor is the glioneuronal
element, which is shown here to consist of a free-floating
neuron in a microcyst surrounded by oligodendroglia I-like
cells (Ellison, pp. 659- 661; WHO, pp. 103-106).

192
Q

Patients harboring this lesion typically present with
A. Meningitis
B. Seizures
C. Hemorrhage
D. Leptomeningeal dissemination
E. Hydrocephalus

A

**B. Seizures **

This lesion is most consistent with a DNT
(WIIO grade I). Patients usually present with long-standing
drug-resistant partial seizures that begin before the age of 20.
They are usually found in the temporal lobe or other supratentorial location and typically encompass the cerebral
cortex. On occasion, they appear to deform the overlying calvarium, a finding that furthe r supports the diagnosis of DNT.
The histologic hallmark of this tumor is the glioneuronal
element, which is shown here to consist of a free-floating
neuron in a microcyst surrounded by oligodendroglia I-like
cells (Ellison, pp. 659- 661; WHO, pp. 103-106).

193
Q

All of the following may be seen with hypochromic
microcytic anemia EXCEPT?
A. Low mean corpuscular volume (MCV)
B. POikilocytosis
C. Elevated total iron-binding capacity (TIBC)
D. Early decrease in mean corpuscular hemoglobin concentration (MCHC)
E. Low serum ferritin

A

**D. Early decrease in mean corpuscular hemoglobin concentration (MCHC) **

Although the MCV progressively decreases as the
anemia becomes more severe , the MCHC usually remains
normal until the hematocrit values drop below 30%. As the
anemia becomes more marked, the red blood cells become
progressively more distorted (poi!,ilocytosis), the TIBC
begins to increase, the ferritin levels drop, the serum iron
stores begin to fall , and the bone marrow iron stores become
depleted. Causes of anemia with low MCV include iron
deficiency (pregnancy, GI bleeding), thalassemia , anemia of
chronic inflammation, sideroblastic anemia, and aluminum
toxicity (Fishman, pp. 345- 347; Barl~er, pp. 620-623).

194
Q

The serum level of valproate may increase following the
administration of which of the following medications?
1. Phenytoin
2. Clozapine
3. Phenobarbital
4. Ethosuximide

A. 1,2, and3 are correct
B. 1 and3 are correct
C. 2 and 4 are correct
D. Only 4 is correct
E. All of the above

A

**B. 1 and3 are correct **

The addition of phenytoin and phenobarbital most
often increases the serum concentration of valproate (Geyer,
p.213).

195
Q

All of the following are true about polymyositis and
inclusion body myositis (IBM) EXCEPT?
A. IBM is more lill:ely to affect the distal muscles of the
legs thin polymyositis
B. IBM is less often seen in association with collagen
vascular or autoimmune diseases
C. Polymyositis rarely responds to steroids, whereas IBi’vI
may respond in some cases
D. IBM affects older patients
E. Jvluscle biopsy in patients with IBM demonstrates
inflammation and inclusion bodies with “rimmed
vacuoles” that contain amyloid

A

**C. Polymyositis rarely responds to steroids, whereas IBi’vI
may respond in some cases **

IBM affects proximal limb muscles, but in contrast to
polymyositis, is much more likely to affect the distal muscles
of the legs. It commonly occurs in men after the age of SO and
has less of an association with autoimmune and collagen vascular diseases than polymyositis. Whereas polymyositis does
respond to steroids, IBM has no widely accepted the rapy, as
steroids have shown minimal benefit (Merritt, pp. 767- 768).

196
Q

All of the following may cause carpa l tunnel syndrome
EXCEPT?
A. Rheumatoid arthritis
B. Pregnancy
C. Acromegaly
D. Amyloidosis
E. Oraves’ disease

A

E. Oraves’ disease

Graves’ disease (hyperthyroidism) does not typically
cause carpal tunnel syndrome but is more likely to produce
ophthalmopathy, including a “stare “ and “lid lag,” and thyroid exophthalmos from mucinous and cellular infiltration
of the extraocular llluscles (inferior and medial recti most
commonly affected). Common e tiologies of carpal tunnel
syndrome include rheumatoid arthritis, pregnancy, acromegaly, amyloidosis, myxedema, and birth control pills
(Greenberg, pp. 536- 539).

197
Q

A high school gymnast develops a sudden headache,
dizziness, left-sided arm and leg clumsiness, and left facial
and right body numbness following a practice session. What
is the most likely diagnosis?
A. Labyrinthitis
B. Benign paroxysmal positional vertigo
C. Vertebral artery dissection
D. Complex migraine headache
E. Multiple sclerosis

A

**C. Vertebral artery dissection **

This constellation of signs/symptoms is most consistent with the lateral medullary syndrome (of Wallenberg).
In a young patient who was subjected to strenuous exercise
and extreme neck movements, the most lll,ely diagnosis is
vertebral artery dissection (Greenberg, pp. 849-850).

198
Q

What is the primary ligand that binds to epidermal
growth factor receptor (EOFR) in gliomas ?
A. Fibronectin
B. Epidermal growth factor (EOF)
C. Tumor growth factor (1’01”-0:)
D. Vascular endothelial growth factor (VEOl”)
E. Fibroblast growth factor (1”01”)

A

**C. Tumor growth factor (1’01”-0:) **

EGF and TGF-a. bind to EGFR with equal affinity;
however, whereas EGFR is commonly upregulated in highgrade malignancies, EGF is rarely over-expressed. In contrast, TGF-a. is frequently expressed in gliomas, which binds
to EGFR and stimulates tyrosine kinase-specific activity,
leading to further cell transformation. Thus, it is believed
that TGF-a. is the primary ligand binding to EGFR in malignant gliomas (Youmans, pp. 726-727).

199
Q

All of the following neurochemical and cellular mediators
may be elevated after severe closed head injury EXCEPT?
A. TNF-o:
B. IL-IP
C. IL-6
D. Potassium
E. Magnesium

A

E. Magnesium

TNF-a., IL-lb, IL-6, and potassium levels have been
shown to increase after DAI, while magnesium-which is
involved in glycolysis, oxidative phosphorylation, cellular
respiration, and the synthesis of DNA, RNA, and proteinshas been shown to decrease after severe head injury (Marion,
pp.40-45).

200
Q

After denervation, the first evidence of fibrillation
potentials in human muscle usually occurs how many days
after the insult ?
A. 5 to 10 days
B. 14 to 21 days
C. 24 to 32 days
D. 42 days
E. 6 months

A

**B. 14 to 21 days **

Denervation of muscle results in fibrillation potentials and positive sharp waves within approximately 2 to 3
weel(s and 8 days, respectively. These findings persist until
the muscle fibers are reinnervated (usually 3 to 4 months
after mild insults) or until the denervated muscl e undergoes
complete atrophy (Youmans, p. 3856).

201
Q

This axial CT scan (Figure 8.201Q) demonstrates
A. Arachnoiditis
B. Hypertrophy of the ligamentulll f1avulll
C. Synovial cyst
D. Laminar fracture
E. Hemangioma

A

**A. Arachnoiditis **

Note the multiple prominent irregularities and
loculations located circumferentially around the margins of
the dura on this CT myelogram, which is most consistent
with arachnoiditis (Ramsey, pp. 739-741).

202
Q

Which of the following is/are true about diagnostic
studies for neurosarcoidosis?
1. Elevation in IgO and IgO index
2. Elevated angiotensin-converting enzyme levels
3. CNS granulomas on MRI
4. Oligoclonal bands found in CSl”
5. A. 1, 2, and3 are correct
B. 1 and3 are correct
C. 2 and4 are correct
D. Only 4 is correct
E. All of the above

A

E. All of the above

Sarcoidosis is a systemic granulomatous disease that
involves the nervous system in 5% of patients (neurosarcoidosis). CNS granulomas in neurosarcoidosis can involve the
cranial nerves, meninges, hypothalamus, brain parenchyma,
and spinal cord. The skull base is frequently affected, and
granulomas in this location may result in obstructive hydrocephalus. Cranial nerve palsies may also occur; the facial
nerve is most commonly involved. Hypothalamic granulomas
can result in diabetes insipidus, galactorrhea, amenorrhea,
and changes in behavior, sleep patterns, and appetite.
Sarcoidosis can also cause peripheral neuropathies, such as
mononeuropathy multiplex. MRI often reveals the presence
of CNS granulomas, and CSF exhibits a lymphocytic pleocytosis with elevations in IgG and the IgG index. Oligoclonal
bands and elevated angiotensin-converting enzyme (ACE)
levels can also be found in the CSF. Neurosarcoidosis
can present as a self-limited monophasic illness or a chronic
disease with relapses and remissions. Treatment of neurosarcoidosis involves corticosteroids (prednisone), with
immunosuppressant drugs added in refractory cases (azathioprine, methotrexa te, cyclosporine) (Merritt pp. 180- 181)

203
Q

The EEG below (Figure 8.203Q) is most consistent
with?
A. 1, 2, and3 are correct
B. 1 and3 are correct
C. 2 and4 are correct
D. Only 4 is correct
E. All of the above

A

A. Alcohol intoxication
B. Left frontal lobe mass
C. A lethal closed head injury
D. Hepatic encephalopathy
E. Delta rhythm

Electrocerebral inactivity is consistent with brain
death in the setting of a detailed brain death exam. It is
defined as no cerebral electrical potentials greater than 2 ~IV.
It is not required to pronounce brain death and is used as an
ancillary test when the diagnosis is not clear. Drug intoxication (e.g., phenobarbital) and hypothermia may result in
reversible electrocerebral inactivity (Greenberg, p. 130).

204
Q

What abnormality is depicted on the ECG below (Figure
8.204Q)?
9.A. Torsades de polites
B. Hyperkalemia
C. Digoxin toxicity
D. First-degree heart block
E. Multifocal atrial tachycardia

A

A. Torsades de polites
B. Hyperkalemia
C. Digoxin toxicity
D. First-degree heart block
E. Multifocal atrial tachycardia

The most potentially serious complication of hyperkalemia is slowing of electrical heart conduction. The ECG
begins to change when the serum K+ reaches approximately
6.0 mEq/L and is always abnormal when it is > 8.0 mEq/L.
The earliest ECG abnormality is a tall, tapering l’ wave that is
most evident in the precordial leads V2 and V”. As W levels
increase further, the P-wave amplitude decreases and the PR
interval lengthens. The P waves may eventually disappear
and the QRS complex widen . The final event is ventricular
asystole (Marino, pp. 654- 655).

205
Q

Match the visual field cut (Figure 8.20S-8.209Q)
with the most likely lesion site

Temporallobe

A

E

Occlusion of the anterior
choroidal artery causes a homonymous defect in the upper and lower quadrants, with sparing of the horizontal sector
(quadruple sectoranopia, B), which is usually characteristic
of a lateral geniculate body infarct that is supplied by the
anterior choroidal artery. The central portion of the lateral
geniculate body receives blood flow primarily from the lateral posterior choroidal artery. Interruption of this vessel
causes a horizontal homonymous sector defect (wedgeshaped, D). Superior homonymous quadran tic defects (“piein the sky, “ E) may result from a lesion along i-Ieyer’s loop
(after temporal lobectomy) or along the inferior bank of
the calcarine fissure . The anterior chiasm or junctional
syndrome results in a unilateral optic nerve defect of one
eye and a superior temporal defect in the other eye (A) due
to the loop made by the inferonasal retina of the other eye
(\Villebrand’s knee). Lesions located in the most anterior
portion of the calcarine cortex cause a crescent-shaped
defect restricted to the temporal field of the contralateral eye
(monocular temporal crescent, C). This is the only retrochiasmatic lesion that may result in a strictly unilateral visual
field defect (Bralis, pp. 132-140).

206
Q

Match the visual field cut (Figure 8.20S-8.209Q)
with the most likely lesion site

Lateral posterior choroidal artery

A

D

Occlusion of the anterior
choroidal artery causes a homonymous defect in the upper and lower quadrants, with sparing of the horizontal sector
(quadruple sectoranopia, B), which is usually characteristic
of a lateral geniculate body infarct that is supplied by the
anterior choroidal artery. The central portion of the lateral
geniculate body receives blood flow primarily from the lateral posterior choroidal artery. Interruption of this vessel
causes a horizontal homonymous sector defect (wedgeshaped, D). Superior homonymous quadran tic defects (“piein the sky, “ E) may result from a lesion along i-Ieyer’s loop
(after temporal lobectomy) or along the inferior bank of
the calcarine fissure . The anterior chiasm or junctional
syndrome results in a unilateral optic nerve defect of one
eye and a superior temporal defect in the other eye (A) due
to the loop made by the inferonasal retina of the other eye
(\Villebrand’s knee). Lesions located in the most anterior
portion of the calcarine cortex cause a crescent-shaped
defect restricted to the temporal field of the contralateral eye
(monocular temporal crescent, C). This is the only retrochiasmatic lesion that may result in a strictly unilateral visual
field defect (Bralis, pp. 132-140).

207
Q

Match the visual field cut (Figure 8.20S-8.209Q)
with the most likely lesion site

Anterior chiasm

A

A

Occlusion of the anterior
choroidal artery causes a homonymous defect in the upper and lower quadrants, with sparing of the horizontal sector
(quadruple sectoranopia, B), which is usually characteristic
of a lateral geniculate body infarct that is supplied by the
anterior choroidal artery. The central portion of the lateral
geniculate body receives blood flow primarily from the lateral posterior choroidal artery. Interruption of this vessel
causes a horizontal homonymous sector defect (wedgeshaped, D). Superior homonymous quadran tic defects (“piein the sky, “ E) may result from a lesion along i-Ieyer’s loop
(after temporal lobectomy) or along the inferior bank of
the calcarine fissure . The anterior chiasm or junctional
syndrome results in a unilateral optic nerve defect of one
eye and a superior temporal defect in the other eye (A) due
to the loop made by the inferonasal retina of the other eye
(\Villebrand’s knee). Lesions located in the most anterior
portion of the calcarine cortex cause a crescent-shaped
defect restricted to the temporal field of the contralateral eye
(monocular temporal crescent, C). This is the only retrochiasmatic lesion that may result in a strictly unilateral visual
field defect (Bralis, pp. 132-140).

208
Q

Match the visual field cut (Figure 8.20S-8.209Q)
with the most likely lesion site

Anterior calcarine cortex

A

C

Occlusion of the anterior
choroidal artery causes a homonymous defect in the upper and lower quadrants, with sparing of the horizontal sector
(quadruple sectoranopia, B), which is usually characteristic
of a lateral geniculate body infarct that is supplied by the
anterior choroidal artery. The central portion of the lateral
geniculate body receives blood flow primarily from the lateral posterior choroidal artery. Interruption of this vessel
causes a horizontal homonymous sector defect (wedgeshaped, D). Superior homonymous quadran tic defects (“piein the sky, “ E) may result from a lesion along i-Ieyer’s loop
(after temporal lobectomy) or along the inferior bank of
the calcarine fissure . The anterior chiasm or junctional
syndrome results in a unilateral optic nerve defect of one
eye and a superior temporal defect in the other eye (A) due
to the loop made by the inferonasal retina of the other eye
(\Villebrand’s knee). Lesions located in the most anterior
portion of the calcarine cortex cause a crescent-shaped
defect restricted to the temporal field of the contralateral eye
(monocular temporal crescent, C). This is the only retrochiasmatic lesion that may result in a strictly unilateral visual
field defect (Bralis, pp. 132-140).

209
Q

Match the visual field cut (Figure 8.20S-8.209Q)
with the most likely lesion site

Anterior choroidal artery

A

B

Occlusion of the anterior
choroidal artery causes a homonymous defect in the upper and lower quadrants, with sparing of the horizontal sector
(quadruple sectoranopia, B), which is usually characteristic
of a lateral geniculate body infarct that is supplied by the
anterior choroidal artery. The central portion of the lateral
geniculate body receives blood flow primarily from the lateral posterior choroidal artery. Interruption of this vessel
causes a horizontal homonymous sector defect (wedgeshaped, D). Superior homonymous quadran tic defects (“piein the sky, “ E) may result from a lesion along i-Ieyer’s loop
(after temporal lobectomy) or along the inferior bank of
the calcarine fissure . The anterior chiasm or junctional
syndrome results in a unilateral optic nerve defect of one
eye and a superior temporal defect in the other eye (A) due
to the loop made by the inferonasal retina of the other eye
(\Villebrand’s knee). Lesions located in the most anterior
portion of the calcarine cortex cause a crescent-shaped
defect restricted to the temporal field of the contralateral eye
(monocular temporal crescent, C). This is the only retrochiasmatic lesion that may result in a strictly unilateral visual
field defect (Bralis, pp. 132-140).

210
Q

The musculocutaneous nerve controls what muscle
actions?
1. Elbow flexion
2. Forearm pronation
3. Forearm supination
4. Wrist flexion

A. 1, 2, and 3 are correct
B. 1 and 3 are correct
C. 2 and 4 are correct
D. Only 4 is correct
E. All of the above

A

A. 1, 2, and 3 are correct
B. 1 and 3 are correct
C. 2 and 4 are correct
D. Only 4 is correct
E. All of the above

The musculocutaneous nerve innervates the biceps,
brachialis, and coracobrachialis muscles, which control
elbow flexion and forearm supination (Bralis, p. 60).

211
Q

Where is the lesion that may cause ipsilateral upper
extremity wealmess and contralateral leg wealmess (cruciate
paralysis) most lil_ely located?

A. Ventrolateral pons
B. Basis pontis
C. Posterior internal capsule
D. Cervicomeclullary junction
E. Ventral lateral pons

A

A. Ventrolateral pons
B. Basis pontis
C. Posterior internal capsule
**D. Cervicomeclullary junction **
E. Ventral lateral pons

Where the pyramidal tract decussates at the cervicomedullary junction with segregation of arm fibers (rostral)
and leg fibers (caudal), a lesion can cause the unique COI11-
bination of ipsilateral arm weakness and contralateral leg
paresis (cruciate paralysis) (Bralis, p. 95; Greenberg, p. 95).

212
Q

What is the diagnosis ?
A. j\lobar holoprosencephaly
B. Lobar holoprosencephaly
C. Hydranencephaly
D. Severe hydrocephalus
E . Anencephaly

A

A. j\lobar holoprosencephaly
B. Lobar holoprosencephaly
**C. Hydranencephaly **
D. Severe hydrocephalus
E * . Anencephaly

Hydranencephaly describes a brain replaced by
CSF rather than compressed by expansion of the ventricles,
as in severe hydrocephalus. Moreover, with severe hydrocephalus, there is usually a thin mantle of brain, which is
absent in hydranencephaly. The absel1ce of a cortical mantle
with hydranencephaly is most often associated with angiographic evidence of supra clinoid occlusion of the carotid
arteries, which strongly suggests a vascular pathogenesis.
The posterior fossa and thalami are often preserved due to
preserved posterior circulation. The craniulll of afilicted
children usually fails to grow and remains small. The
incidence is estimated to be 1 in 6000, and affected infants
rarely live longer than a yea r (Albright, pp. 155- 156).

213
Q

What is the most likely etiology of this finding?
A. Obstl’l1ctive hydrocephalus
B. Failure of disjunction
C. Disorder of cellular migration
D. Bilateral in utero disl’l1ption of the anterior circulation
E. Failure of neural tube closure

A

A. Obstl’l1ctive hydrocephalus
B. Failure of disjunction
**C. Disorder of cellular migration **
D. Bilateral in utero disl’l1ption of the anterior circulation
E. Failure of neural tube closure

Hydranencephaly describes a brain replaced by
CSF rather than compressed by expansion of the ventricles,
as in severe hydrocephalus. Moreover, with severe hydrocephalus, there is usually a thin mantle of brain, which is
absent in hydranencephaly. The absel1ce of a cortical mantle
with hydranencephaly is most often associated with angiographic evidence of supra clinoid occlusion of the carotid
arteries, which strongly suggests a vascular pathogenesis.
The posterior fossa and thalami are often preserved due to
preserved posterior circulation. The craniulll of afilicted
children usually fails to grow and remains small. The
incidence is estimated to be 1 in 6000, and affected infants
rarely live longer than a yea r (Albright, pp. 155- 156).

214
Q

Diagnostic peritoneal lavage (DPL) is relatively fast,
safe, and reliable for patients with blunt injury and stab
wounds to the anterior abdominal wall. This diagnostic
procedure is considered positive with all of the following
EXCEPT?
A. Greater than 10 mL of gross blood is aspirated from the
abdomen
B. Microscopic evaluation of fluid aspirate reveals
> 100,000hllmJ of red blood cells
C. Greater than 500/m111.1 white blood cells
D. Presence of bile or particulate matter in aspirate
E. An amylase level that is less than the normal Sel’l1111
level

A

A. Greater than 10 mL of gross blood is aspirated from the
abdomen
B. Microscopic evaluation of fluid aspirate reveals
> 100,000hllmJ of red blood cells
C. Greater than 500/m111.1 white blood cells
D. Presence of bile or particulate matter in aspirate
E. An amylase level that is less than the normal Sel’l1111
level

DPL is considered positive if greater than 10 mL of
gross blood is aspirated from the peritoneal cavity or if 1 L
of lactated Ringer’s solution is infused into the peritoneal
cavity and microscopic evaluation reveals red blood cells
> 100,000/ml11’\ white blood cells> 500/ml11’\ or the presence
of bile, particulate matter, or amylase greater than the normal serum value. It can be used for penetrating stab wounds
after positive local exploration or suspected hollow viscus
injury, following an abnormal ultrasound examination, or
following a normal ultrasound when the patient exhibits
hemodynamic instability or signs and symptoms suggestive
of an intra-abdominal injury (Nwariaku, pp. 64-65).

215
Q

What is depicted by the photomicrograph below (Figure 8.215Q)?
A. Gemistocytic astrocytoma
B. I-Iemangioblastoma
C. Chordoma
D. Clear cell meningioma
E. Desmoplastic medulloblastoma

A

**A. Gemistocytic astrocytoma **
B. I-Iemangioblastoma
C. Chordoma
D. Clear cell meningioma
E. Desmoplastic medulloblastoma

Note the large, glassy eosinophilic cell bodies with
an angular shape on this photomicrograph, which depicts a
gemistocytic astrocytoma (Ellison, p. 627; WHO, p. 25).

216
Q

In a patient with conductive hearing loss on the right,
which of the following apply?
A. Weber lateralizes to the left, Rinne is negative (b > a),
Schwa bach test is normal
B. Weber lateralizes to the right, Rinne is positive (a > b),
Schwa bach test is normal
C. Weber lateralizes to the right, Rinne is negative,
Schwa bach test is normal or slightly prolonged
D. Weber lateralizes to the right, Rinne is positive,
Schwa bach test is abnormal
E. Weber lateralizes to the left, Rinne is positive,
Schwa bach test is abnormal

A

A. Weber lateralizes to the left, Rinne is negative (b > a),
Schwa bach test is normal
B. Weber lateralizes to the right, Rinne is positive (a > b),
Schwa bach test is normal
**C. Weber lateralizes to the right, Rinne is negative,
Schwa bach test is normal or slightly prolonged **
D. Weber lateralizes to the right, Rinne is positive,
Schwa bach test is abnormal
E. Weber lateralizes to the left, Rinne is positive,
Schwa bach test is abnormal

In conductive hearing loss on the right, the Weber
test lateralizes to the right, the Rinne test is negative (bone
conduction is better than air conduction), and the
Schwabach test is normal or prolonged (the patient can hear
the tuning fork longer than the examiner can) (Brazis,
pp. 298-299)

217
Q

In performing the Nylen-Bantny maneuver, all of the
following suggest peripheral disease EXCEPT?
A. Latency of appearance of nystagmus of 2 to 12 seconds
B. The vertigo and nystagmus usually disappear 10 to
15 seconds after their appearance
C. Change in direction of nystagmus with the head down
and persistence of nystagmus
D. Habituation of the response with repeated maneuvers
E. The reproducibility of abnormalities noted with this
maneuver becomes inconsistent

A

A. Latency of appearance of nystagmus of 2 to 12 seconds
B. The vertigo and nystagmus usually disappear 10 to
15 seconds after their appearance
**C. Change in direction of nystagmus with the head down
and persistence of nystagmus **
D. Habituation of the response with repeated maneuvers
E. The reproducibility of abnormalities noted with this
maneuver becomes inconsistent

With peripheral lesions, severe rotational vertigo
associated with nausea and vomiting and nystagmus appear
approximately 2 to 15 seconds after performing the NylenBarany maneuver. There is usually habituation of response
with repeated maneuvers, and the reproducibility of abnormalities wi th this maneuver becomes inconsistent with
repeated testing. Cochlear and other neurologic symptoms
are usually absent. The vertigo and nystagmus typically
resolve within 10 to 15 seconds after their appearance; after
the patient is rapidly brought bacl, to a sitting position,
vertigo recurs and nystagmus develops in the opposite
direction (Brazis, pp. 305- 306).

218
Q

A 35-year-old male with the photomicrograph depicted
below (Figure 8.218Q) presents with fever, chills, and a
seizure. What is the diagnosis?
A. Creutzfeldt-Jakob disease
B. Herpes encephalitis
C. Malaria
D. Ganglioglioma
E. Lymphoma

A

A. Creutzfeldt-Jakob disease
B. Herpes encephalitis
**C. Malaria **
D. Ganglioglioma
E. Lymphoma

Note the occlusion of the blood vessel, which is
due to the presence of many ghost-like red blood cells
that contain parasites on this I-I & E section depicting
malaria. Patients with this disease can present with seizures,
headache, somnolence, confUSion, photophobia, and almost
any focal neurologic deficit. Without urgent treatment, these
patients usually progress to coma and brain death (Ellison,
pp. 653- 656).

219
Q

The lesion depicted below (Figure 8.219Q) is most often
associated with what other abnormality?
A. Castleman’s syndrome
B. Tuberous sclerosis
C. Temporal lobe epilepsy
D. Cafe aulait spots
E. Cobblestone skin pattern

A

A. Castleman’s syndrome
**B. Tuberous sclerosis **
C. Temporal lobe epilepsy
D. Cafe aulait spots
E. Cobblestone skin pattern

This photomicrograph depicts a subependymal giant
cell astrocytoma. This lesion is usually located adjacent
to the foramen of ~'Ionro and is associated with tuberous
sclerosis (dominant mutation in TSC 1 on 9p or TSC 2 on
16p). It usually resembles gemistocytic astrocytoma, but
is not infiltrative. Note the abundant cytoplasm, abundant
perivascular fibrillar zone, and prominent nucleolus in this
section. The nuclei are eccentric, which distinguishes this
from gemistocytic astrocytoma (Ell ison, pp. 637-639).

220
Q

Paralysis of upward gaze, light-near dissociation of pupil
response, convergence retraction nystagmus, and pathologic
lid retraction are most consistent with what abnormality?
A. Hydrocephalus
B. A 3-cmmeningioma of the falx
C. Pituitary adenoma
D. Temporal lobe porencephalic cyst
E. Occipital lobe arterial-venous malformation

A

**A. Hydrocephalus **
B. A 3-cmmeningioma of the falx
C. Pituitary adenoma
D. Temporal lobe porencephalic cyst
E. Occipital lobe arterial-venous malformation

This constellation of clinical findings is most consistent with Parinaud’s syndrome, which is often associated
with pineal region masses causing direct compression of
the tectal pia te or from compression of the mesencephalic
tectum by a dilated supra pineal recess in cases of hydrocephalus (Greenberg, p. 87)

221
Q

All of the following are true of the secondary
somatosensory cortex (SII) EXCEPT?
A. Lies along the superior banlt of the lateral sulcus and
extends posteriorly into the parietal lobe
B. Representation of body parts is in the reverse
sequence to that found in the primary somesthetic
area with the two face areas adjacent
C. Representation of the body is bilateral in SII, although
contralateral predominates
D. The efferent cortical projections are mainly to SI and
the primary motor cortex
E. Lesions typically cause contralateral weakness

A

A. Lies along the superior banlt of the lateral sulcus and
extends posteriorly into the parietal lobe
B. Representation of body parts is in the reverse
sequence to that found in the primary somesthetic
area with the two face areas adjacent
C. Representation of the body is bilateral in SII, although
contralateral predominates
D. The efferent cortical projections are mainly to SI and
the primary motor cortex
**E. Lesions typically cause contralateral weakness **

SII lies along the superior bank of the lateral sulcus
and extends posteriorly into the parietal lobe; it has repre sentation of body parts in the reverse sequence to that found
in the primary somesthetic area , with the two face areas
being adjacent. Representation of the body is bilateral in
SII (although contralateral predominates), and the efferent
cortical projections are mainly to SI and the primary motor
cortex (Carpenter, pp. 404-405).

222
Q

In the callosal syndrome (interhemispheric disconnection syndrome) clue to surgery, all of the following may
be seen in a left hemisphere- dominant patient EXCEPT?
A. Inability to name objects (kept from view) palpated by
the right hand
B. Inability to name objects presented to the left
hemifield
C. No interference with most activities of daily living
D. Inability to copy a complex design with the right hand
E. Unable to execute a command with the left hand

A

**A. Inability to name objects (kept from view) palpated by
the right hand **
B. Inability to name objects presented to the left
hemifield
C. No interference with most activities of daily living
D. Inability to copy a complex design with the right hand
E. Unable to execute a command with the left hand

The corpus callosum functions to transfer information from one hemisphere to the other. After sectioning, the
patient cannot execute a command with the left hand but
will do so consistently with the right. The left hemisphere
understands the command but is unable to transfer that
information to the right side of the brain. The right hemisphere does not typically understand the command because
it lacks the ability to understand language . Therefore a
patient presented with an object with the right hand should
be able to reliably name it, since these fibers are relayed to
the contralateral cerebral cortex (DeMyer, pp. 316, 319).

223
Q

Which one of the following correctly identifies the
most to least common locations for hypertensive intracranial
hemorrhage ?
A. Pons, thalamus, putamen, cerebellum, lobar
B. Putamen, thalamus, pons, cerebellum, lobar
C. Putamen , thalamus, cerebellum, pons, lobar
D. Thalamus, putamen, cerebellum, lobar, pons
E. Thalamus, putamen, cerebellum, pons, lobar

A

A. Pons, thalamus, putamen, cerebellum, lobar
**B. Putamen, thalamus, pons, cerebellum, lobar **
C. Putamen , thalamus, cerebellum, pons, lobar
D. Thalamus, putamen, cerebellum, lobar, pons
E. Thalamus, putamen, cerebellum, pons, lobar

The most common locations for hypertensive intracranial hemorrhage include the putamen, followed by the
thalamus, pons, cerebellum, and lobar regions. The origin of
some “hypertensive” putaminal hemorrhages are believed to
result frommicroaneurysms of “Charcot-Bouchard,” but this
is somewhat controversial (Greenberg, p. 815).

224
Q

All of the following are characteristics of normal pressure hydrocephalus EXCEPT?
A. Wide-based “magnetic” gait
B. Bradyphrenia
C. Incontinence
D. Headaches
E. Transient increases in intracranial pressure with continuous monitoring

A

A. Wide-based “magnetic” gait
B. Bradyphrenia
C. Incontinence
**D. Headaches **
E. Transient increases in intracranial pressure with continuous monitoring

Hydrocephalus, gait disturbance (“magnetic gait”
with short, shuffling steps), dementia (primarily memory
impairment with bradyphrenia or slowness of thought),
incontinence, and transient elevations in intracranial pressure (with monitoring) may all occur with normal pressure
hydrocephalus (NPH). Headache and papilledema are typically not seen in patients with NPH; their presence should
raise suspicion of another diagnosis (Greenberg, pp. 191-
194)

225
Q

The EEG below (Figure 8.225Q) is most consistent with
what diagnosis?
A. Right frontal tumor
B. Left occipital tumor
C. Uremia
D. Sleep
E. Large hemispheric insult

A

A. Right frontal tumor
B. Left occipital tumor
C. Uremia
D. Sleep
E. Large hemispheric insult

The seizure spikes are over a fairly wide area of the
right parasagittal region (ma.ximal F4, C4) consistent with a
right frontal onset corresponding with a right parasagittal
meningioma. In order to localize seizure foci, it is imperative
for clinicians to memorize standard electrode placement and
designations. Note that by convention, electrodes designated
with odd numbers are on the left, while with even numbers
on the right. The standard electrode designations are as
follows: Fpl/Fp2 = frontopolar or prefrontal; F3/F4 = midfrontal; C3/C4 = central (roughly over central sulcus); P3/P4
= parietal; 01/02 = OCCipital; F7/F8 = inferior frontal (sometimes called anterior frontal); T3/T4 = mid temporal (records
activity over anterior and midtemporal activity, important
for temporal lobe epilepsy); 1’5/1’6 = posterior temporal; Fz,
Cz, pz = midline electrodes in frontal and parietal regions
(record mesial surfaces of hemispheres); AlIA2 = ear reference electrodes (while used for references also record
mid temporal activity); TI/T2 = so-called true anterior temporal electrodes; Spl/Sp2 = sphenoidal electrodes (record activity from inferomesial surface of the temporal lobes)
(Rowan, pp. 4- 7).

226
Q

The following is a schematic diagram through the
pallidofugal fiber system in a coronal plane. Match the fibers
(numbered items) with the appropriate letterhead (Figure
8.226-8.231Q), using each answer either once, more than
once, or not at all.

Ansa lenticularis

A

D

Fibers of the ansa
lenticularis (D) leave the outer part of the medial globus
pallidus, pass around the internal capsule, and enter the prerubral field (field H of Forel, not labeled) prior to merging
with the lenticular fasciculus (H2). Fibers of the lenticular
fasciculus originate from the inner part of the medial globus
pallidus, traverse the posterior limb of the anterior capsule,
and pass medially, dorsal to the subthalamic nucleus, to also
enter the prerubral field. The ansa lenticularis and lenticular
fasciculus then travel together dorsal to the zona incerta as
components of the thalamic fasciculus (A). The subthalamic
fasciculus (C) is comprised of pallidosubthalamic fibers
originating from the lateral or external pallidal segment and
subthalamopallidal fibers that terminate in the medullary
lamina of both pallidal segments. Both components of the
subthalamic fasciculus cross the internal capsule. Thalamostriate fibers from the centromedian nucleus project to the
putamen (E) (Carpenter, pp. 337- 345).

227
Q

The following is a schematic diagram through the
pallidofugal fiber system in a coronal plane. Match the fibers
(numbered items) with the appropriate letterhead (Figure
8.226-8.231Q), using each answer either once, more than
once, or not at all.

Thalamostriate fibers

A

E

Fibers of the ansa
lenticularis (D) leave the outer part of the medial globus
pallidus, pass around the internal capsule, and enter the prerubral field (field H of Forel, not labeled) prior to merging
with the lenticular fasciculus (H2). Fibers of the lenticular
fasciculus originate from the inner part of the medial globus
pallidus, traverse the posterior limb of the anterior capsule,
and pass medially, dorsal to the subthalamic nucleus, to also
enter the prerubral field. The ansa lenticularis and lenticular
fasciculus then travel together dorsal to the zona incerta as
components of the thalamic fasciculus (A). The subthalamic
fasciculus (C) is comprised of pallidosubthalamic fibers
originating from the lateral or external pallidal segment and
subthalamopallidal fibers that terminate in the medullary
lamina of both pallidal segments. Both components of the
subthalamic fasciculus cross the internal capsule. Thalamostriate fibers from the centromedian nucleus project to the
putamen (E) (Carpenter, pp. 337- 345).

228
Q

The following is a schematic diagram through the
pallidofugal fiber system in a coronal plane. Match the fibers
(numbered items) with the appropriate letterhead (Figure
8.226-8.231Q), using each answer either once, more than
once, or not at all.

Optic tract

A

F

Fibers of the ansa
lenticularis (D) leave the outer part of the medial globus
pallidus, pass around the internal capsule, and enter the prerubral field (field H of Forel, not labeled) prior to merging
with the lenticular fasciculus (H2). Fibers of the lenticular
fasciculus originate from the inner part of the medial globus
pallidus, traverse the posterior limb of the anterior capsule,
and pass medially, dorsal to the subthalamic nucleus, to also
enter the prerubral field. The ansa lenticularis and lenticular
fasciculus then travel together dorsal to the zona incerta as
components of the thalamic fasciculus (A). The subthalamic
fasciculus (C) is comprised of pallidosubthalamic fibers
originating from the lateral or external pallidal segment and
subthalamopallidal fibers that terminate in the medullary
lamina of both pallidal segments. Both components of the
subthalamic fasciculus cross the internal capsule. Thalamostriate fibers from the centromedian nucleus project to the
putamen (E) (Carpenter, pp. 337- 345).

229
Q

The following is a schematic diagram through the
pallidofugal fiber system in a coronal plane. Match the fibers
(numbered items) with the appropriate letterhead (Figure
8.226-8.231Q), using each answer either once, more than
once, or not at all.

Subthalamic fasciculus

A

C

Fibers of the ansa
lenticularis (D) leave the outer part of the medial globus
pallidus, pass around the internal capsule, and enter the prerubral field (field H of Forel, not labeled) prior to merging
with the lenticular fasciculus (H2). Fibers of the lenticular
fasciculus originate from the inner part of the medial globus
pallidus, traverse the posterior limb of the anterior capsule,
and pass medially, dorsal to the subthalamic nucleus, to also
enter the prerubral field. The ansa lenticularis and lenticular
fasciculus then travel together dorsal to the zona incerta as
components of the thalamic fasciculus (A). The subthalamic
fasciculus (C) is comprised of pallidosubthalamic fibers
originating from the lateral or external pallidal segment and
subthalamopallidal fibers that terminate in the medullary
lamina of both pallidal segments. Both components of the
subthalamic fasciculus cross the internal capsule. Thalamostriate fibers from the centromedian nucleus project to the
putamen (E) (Carpenter, pp. 337- 345).

230
Q

The following is a schematic diagram through the
pallidofugal fiber system in a coronal plane. Match the fibers
(numbered items) with the appropriate letterhead (Figure
8.226-8.231Q), using each answer either once, more than
once, or not at all.

Thalamic fasciculus (HI)

A

A

Fibers of the ansa
lenticularis (D) leave the outer part of the medial globus
pallidus, pass around the internal capsule, and enter the prerubral field (field H of Forel, not labeled) prior to merging
with the lenticular fasciculus (H2). Fibers of the lenticular
fasciculus originate from the inner part of the medial globus
pallidus, traverse the posterior limb of the anterior capsule,
and pass medially, dorsal to the subthalamic nucleus, to also
enter the prerubral field. The ansa lenticularis and lenticular
fasciculus then travel together dorsal to the zona incerta as
components of the thalamic fasciculus (A). The subthalamic
fasciculus (C) is comprised of pallidosubthalamic fibers
originating from the lateral or external pallidal segment and
subthalamopallidal fibers that terminate in the medullary
lamina of both pallidal segments. Both components of the
subthalamic fasciculus cross the internal capsule. Thalamostriate fibers from the centromedian nucleus project to the
putamen (E) (Carpenter, pp. 337- 345).

231
Q

The following is a schematic diagram through the
pallidofugal fiber system in a coronal plane. Match the fibers
(numbered items) with the appropriate letterhead (Figure
8.226-8.231Q), using each answer either once, more than
once, or not at all.

Lenticular fasciculus (H2)

A

B

Fibers of the ansa
lenticularis (D) leave the outer part of the medial globus
pallidus, pass around the internal capsule, and enter the prerubral field (field H of Forel, not labeled) prior to merging
with the lenticular fasciculus (H2). Fibers of the lenticular
fasciculus originate from the inner part of the medial globus
pallidus, traverse the posterior limb of the anterior capsule,
and pass medially, dorsal to the subthalamic nucleus, to also
enter the prerubral field. The ansa lenticularis and lenticular
fasciculus then travel together dorsal to the zona incerta as
components of the thalamic fasciculus (A). The subthalamic
fasciculus (C) is comprised of pallidosubthalamic fibers
originating from the lateral or external pallidal segment and
subthalamopallidal fibers that terminate in the medullary
lamina of both pallidal segments. Both components of the
subthalamic fasciculus cross the internal capsule. Thalamostriate fibers from the centromedian nucleus project to the
putamen (E) (Carpenter, pp. 337- 345).

232
Q

A 31-year-old immunocompromised male with a history of intravenous drug abuse presents with back pain. His
sagittal postcontrasted Tl-weighted MRI is depicted below
(Figure 8.232Q). What is the most likely diagnosis?
A. Leukemia
B. Lymphoma
C. Epidural abscess
D. Epidural lipomatosis
E. IIemangioblastoma

A

A. Leukemia
B. Lymphoma
**C. Epidural abscess **
D. Epidural lipomatosis
E. IIemangioblastoma

Note the prominent ring enhancement of this spinal
epidural abscess, with compression of the adjacent spinal
cord . The most lil{ely causative organism in a patient with a
history of intravenous drug abuse is Staph. aU1”eliS (Ramsey,
pp.498- 502)

233
Q

Necrotizing wound infections are most commonly
produced by what organism(s)?
1. Clost.,-idiwn
2. Staphylococcus
3. p-hemolytic streptococci
4. Corynebacterium

A. 1,2, and 3 are correct
B. 1 and 3 are correct
C. 2 and 4 are correct
D. Only 4 is correct
E. All of the above

A

A. 1,2, and 3 are correct
**B. 1 and 3 are correct **
C. 2 and 4 are correct
D. Only 4 is correct
E. All of the above

Necrotizing wound infections are produced by
Clostridiwn and ~-hemolytic streptococcal species. Unlike
other wound infections that may appear a few days to 1 week
after surgery, necrotizing infections are evident in the first
few postoperative days; they are characterized by skin
crepitance and fluid-filled bullae. Spread to deeper tissues is
a major concern, as this may result in rhabdomyolysis and
myoglobinuric renal failure (Marino, p. 489).

234
Q

All of the following blood vessels originate from the
intracavernous internal carotid artery (ICA) EXCEPT?

A. Tentorial artery (of Bernasconi and Cassinari)
B. Dorsal meningeal artery
C. Inferior hypophyseal artery
D. Vidian artery
E. McConnell’s capsular artery

A

A. Tentorial artery (of Bernasconi and Cassinari)
B. Dorsal meningeal artery
C. Inferior hypophyseal artery
**D. Vidian artery **
E. McConnell’s capsular artery

There are three main branches that originate from
the intracavernous portion of the ICA: the meningohypophyseal trunk, the artery of the inferior cavernous sinus, and
the artery of McConnell. The meningohypophyseal trunk
trifurcates into the tentorial artery (of Bernasconi and
Cassinari), the dorsal meningeal artery (supplies dura over
dorsum sellae), and the inferior hypophyseal artery, which
supplies the posterior pituitary gland and sellar floor.
!vlcConnell’s artery runs along the dura covering the sellar
floor to supply the anterior pituitary gland, while the artery
of the inferior cavernous sinus provides blood supply to the
third, fourth , and sixth cranial nerves as well as the gasserian
ganglion and cavernous sinus dura. The vidian artery originates from the petrous lCA (Osborn DCA, pp. 86- 87).

235
Q

One day after a high school football game, the starting
quarterback presents to the emergency department complaining of diffuse right shoulder pain. The pain is centered
over the lateral scapula and posterior shoulder and is aggravated by right upper arm abduction (first 15 degrees) and
external rotation (particularly when the elbow is held in
90 degrees of flexion). What nerve is most likely injured?

A. Suprascapular
B. Axillary
C. Spinal accessory
D. Posterior interosseous
E. Dorsal scapula

A

**A. Suprascapular **
B. Axillary
C. Spinal accessory
D. Posterior interosseous
E. Dorsal scapula

Patients with entrapment or trauma to the suprascapular nerve often complain of shoulder pain centered over
the lateral scapula and posterior shoulder, which is most often aggravated by arm abduction during the first 15 degrees
(supraspinatus muscle) and external arm rotation when the
elbow is fl exed at 90 degrees (infraspinatus muscle). Atrophy
of the supraspinatus and infraspinatus muscles may also
be apparent on inspection . EMG reveals dene rvation potentials in the supraspinatus and infraspinatus [uuscles and is
the best diagnostic test. Conservative treatment measures
include shoulder exercises and local injections of steroids
and analgesics. Operative decompression usually includes
sectioning of the suprascapular ligament (Youmans, pp. 3932-
3933)

236
Q

1\ 33-year-old male has a 5-month history of dull low
back pain that is exacerbated at night. An axial CT scan of
the lumbar spine reveals a 1.4-cmlytic lesion in the vertebral
body and pedicle of L4 that is most consistent with an
osteoid osteoma. Why does the pain associated with these
lesions typically respond to aspirin?
A. Aspirin decreases the amount of PDGF secreted by
tumor cells
B. Aspirin decreases prostaglandin synthesis in fibroblasts adjacent to the tumor
C. Aspirin halts the production of prostaglandin by the
tUlllor cells
D. Aspirin decreases the amount of substance P in the
dorsal horn of spinal cord cells
E. Aspirin significantly reduces prostaglandin synthesis
in the vasculature supplying the lesion

A

A. Aspirin decreases the amount of PDGF secreted by
tumor cells
B. Aspirin decreases prostaglandin synthesis in fibroblasts adjacent to the tumor
**C. Aspirin halts the production of prostaglandin by the
tUlllor cells **
D. Aspirin decreases the amount of substance P in the
dorsal horn of spinal cord cells
E. Aspirin significantly reduces prostaglandin synthesis
in the vasculature supplying the lesion

Patients with osteoid osteoma involving the lumbar
spine typically present with back pain that is exacerbated at
night. This condition is believed to result fr0111 prostaglandin
production by the tumor. Aspirin has classically been shown
to relieve the pain (Youmans, p. 4297).

237
Q

A 4-year-old male continues to have medically refractory generalized seizures (“drop attacks”) despite being
placed on numerous anticonvulsant medication regimens.
What would be the best surgical option for this patient at this
point?
A. Anatomic hemispherectomy
B. Functional hemispherectomy
C. Sectioning of the corpus callosum
D. Multiple subpial transection
E. Depth electrode placement for monitoring

A

A. Anatomic hemispherectomy
B. Functional hemispherectomy
**C. Sectioning of the corpus callosum **
D. Multiple subpial transection
E. Depth electrode placement for monitoring

Atonic seizures, also called “drop attacl{s,” are
characterized by total loss of muscle tone. When they are
preceded by a brief myoclonic seizure or tonic spasm, an
acceleratory force is added to the fall, furth er contributing
to the high rate of self-injury with this type of seizure.
Most patients referred for corpus callosotomy have severe,
medically refractory seizures usually accompanied by
mental retardation and a severely abnormal EEG. UnW,e
lesionectomy, corpus callosotomy is palliative, not curative.
Nevertheless, thjs procedure can be highly effective for
generalized seizures, with 80% of patients experiencing
complete or nearly comple te cessation of atonic , tonic, and
tonic-clonic attacks in some reports (Merritt, pp. 814, 827).

238
Q

A 67-year-old female is referred to your office for a
left shoulder droop that she has experienced since undergoing a recent lymph node biopsy. She has difficulty raising
her arm above the horizontal and has some numbness in
the occiput behind the ear on the right.
What is the most likely diagnosis ?
A. A:’(illary nerve injury
B. Suprascapular nerve injury
C. Dorsal scapular nerve injury
D. Spinal accessory nerve injury
E. Brachial plexopathy

A

A. A:’(illary nerve injury
B. Suprascapular nerve injury
C. Dorsal scapular nerve injury
**D. Spinal accessory nerve injury **
E. Brachial plexopathy

Atonic seizures, also called “drop attacl{s,” are
characterized by total loss of muscle tone. When they are
preceded by a brief myoclonic seizure or tonic spasm, an
acceleratory force is added to the fall, furth er contributing
to the high rate of self-injury with this type of seizure.
Most patients referred for corpus callosotomy have severe,
medically refractory seizures usually accompanied by
mental retardation and a severely abnormal EEG. UnW,e
lesionectomy, corpus callosotomy is palliative, not curative.
Nevertheless, thjs procedure can be highly effective for
generalized seizures, with 80% of patients experiencing
complete or nearly comple te cessation of atonic , tonic, and
tonic-clonic attacks in some reports (Merritt, pp. 814, 827).

239
Q

A 67-year-old female is referred to your office for a
left shoulder droop that she has experienced since undergoing a recent lymph node biopsy. She has difficulty raising
her arm above the horizontal and has some numbness in
the occiput behind the ear on the right.

What is the most likely reason for the numbness behind
the right ear?
1. Greater auricular nerve injury
2. Lesser auricular nerve injury
3. Lesser occipital nerve injury
4. Lesser auricular nerve injury
A. 1,2, and 3 are correct
B. 1 and 3 are correct
C. 2 and4 are correct
D. Only 4 is correct
E. All of the above

A

A. 1,2, and 3 are correct
**B. 1 and 3 are correct **
C. 2 and4 are correct
D. Only 4 is correct
E. All of the above

Atonic seizures, also called “drop attacl{s,” are
characterized by total loss of muscle tone. When they are
preceded by a brief myoclonic seizure or tonic spasm, an
acceleratory force is added to the fall, furth er contributing
to the high rate of self-injury with this type of seizure.
Most patients referred for corpus callosotomy have severe,
medically refractory seizures usually accompanied by
mental retardation and a severely abnormal EEG. UnW,e
lesionectomy, corpus callosotomy is palliative, not curative.
Nevertheless, thjs procedure can be highly effective for
generalized seizures, with 80% of patients experiencing
complete or nearly comple te cessation of atonic , tonic, and
tonic-clonic attacks in some reports (Merritt, pp. 814, 827).

240
Q

All of the following are advantages of magnetoencephalography (MEG) over electroencephalography (EEG)
EXCEPT?
A. MEG provides better spatial and temporal resolution
for localization of cortical neuronal activity
B. Mass lesions or other pathologic changes do not
significantly distort the signal detected by MEG
C. MEG signa ls are not as readily attenuated as compared
to EEG signals
D. MEG includes data points from below the surface,
whereas EEG reflects only the cortical surface
E. MEG includes both a tangential and radial component
of neuronal current, whereas EEG includes only a
tangential component, which significantly reduces the
complexity of the signal

A

A. MEG provides better spatial and temporal resolution
for localization of cortical neuronal activity
B. Mass lesions or other pathologic changes do not
significantly distort the signal detected by MEG
C. MEG signa ls are not as readily attenuated as compared
to EEG signals
D. MEG includes data points from below the surface,
whereas EEG reflects only the cortical surface
**E. MEG includes both a tangential and radial component
of neuronal current, whereas EEG includes only a
tangential component, which significantly reduces the
complexity of the signal **

MEG has several advantages over EEG recording.
First, unlil{e EEG-based techniques, which measure electrical potentials mainly from extracellular volume currents,
the magnetic fields measured with MEG primarily refl ec t
intracellular current flow and are not attenuated by the inhomogeneously conducting layers of bone, scalp, and other
extracerebral tissue . This provides excellent spatial and temporal resolution for localization of neuronal activity, and it
more accurately represents the localization of eloquent cortex
than EEG. Second, mass lesions or other pathologic changes
in the brain, which may significantly distort the signal detected

241
Q

What is the most likely
diagnosis of this patient?
A. Sphenoid wing meningioma
B. Chondrosarcoma
C. Fibrous dysplasia
D. Esthenioblastoma
E. Osteochondroma

A

A. Sphenoid wing meningioma
B. Chondrosarcoma
C. Fibrous dysplasia
D. Esthenioblastoma
E. Osteochondroma

Note the expansion of bone (sphenoid) and
“ground glass appearance” on this noncontrasted CT scan in
a patient with fibrous dysplasia. Malignant degeneration,
usually to osteosarcoma, has been reported to occur in
approximately 0.5% of patients with fibrous dysplasia
(Albright, pp. 456- 457).

242
Q

Malignant degeneration of this lesion is most likely to
produce what type of neoplasm?
A. Osteochondroma
B. Osteoblastoma
C. Chloroma
D. Osteosarcoma
E. Neuroblastoma

A

A. Osteochondroma
B. Osteoblastoma
C. Chloroma
**D. Osteosarcoma **
E. Neuroblastoma

Note the expansion of bone (sphenoid) and
“ground glass appearance” on this noncontrasted CT scan in
a patient with fibrous dysplasia. Malignant degeneration,
usually to osteosarcoma, has been reported to occur in
approximately 0.5% of patients with fibrous dysplasia
(Albright, pp. 456- 457).

243
Q

The stylopharyngeus muscle is derived from which
brachial arch?
A. First
B. Second
C. Third
D. Fourth
E. Sixth

A

A. First
B. Second
C. Third
D. Fourth
E. Sixth

The stylopharyngeus muscle is innervated by the
glossopharyngeal nerve (IX) and is the only muscle derived
from the third brachial arch . The muscles of facial expression are all innervated by the facial nerve (VII) and are
derived from the second brachial arch, whereas the thyroarytenoid muscle and intrinsic laryngeal musculature
(except cricothyroid muscle, fourth arch derivative) are
derived from the sbah brachial arch. The muscles of mastication are derived from the first brachial arch and are innervated by the mandibular division of the trigeminal nerve
(April, p. 556).

244
Q

An 18-year-old male presents to the emergency department with morning headaches associated with nausea and
right arm numbness; his CT scan showed a low-density area
in the left parietal region. The angiogram is depicted below
(Figure 8.244Q). What is the most likely diagnosis ?
A. Atherosclerosis
B. Malignant brain tumor
C. Fibromuscular dysplasia
D. lvloyamoya disease
E. Embolic infarct

A

A. Atherosclerosis
B. Malignant brain tumor
C. Fibromuscular dysplasia
**D. lvloyamoya disease **
E. Embolic infarct

Note the paucity of blood flow in the distal internal
carotid and cerebral artery vasculature and the presence
of prominent collaterals that resemble a “puff of smoke” on
this lateral angiogram, which depicts moyamoya (Albright,
pp. 1053- 1069).

245
Q

Which of the following cells give rise to axons that
compose the optic nerve ?
A. Rods and cones of the retina
B. Bipolar cells
C. Ganglion cells
D. Horizontal cells
E. Amacrine cells

A

A. Rods and cones of the retina
B. Bipolar cells
**C. Ganglion cells **
D. Horizontal cells
E. Amacrine cells

The ganglion cells in the retina gives rise to the optic
nerve. The ganglion cells receive impulses from the bipolar
cells, which, in turn, receive signals from the light-sensitive
rods and cones (Kandel, pp. 517 -521).

246
Q

Match the questions with the most likely type of
nerve injury, using each letterhead either once, more than
once, or not at all.
A. Neurotmesis
B. A.’wnotmesis
C. Neurapraxia
D. A and B
E. None of the above

A

A. Neurotmesis
B. A.’wnotmesis
**C. Neurapraxia **
D. A and B
E. None of the above

With neurotmesis, there is complete anatomic transection of the nerve accompanied by
Wallerian degeneration. These injuries almost always require
surgical repair, although the timing remains unclear. For
sharply divided nerves, immediate repair appears to be ideal,
whereas for bluntly injured nerves, some advocate waiting
approximately 3 to 4 weeks prior to intervening surgically to
allow for better proximal and distal nerve stump delineation.
With a:mnotmesis, there is a loss of axonal continuity, but
the soma remains continuous. Wallerian degeneration also
occurs with this type of insult. With neurapraxia, conduction
ceases without structural damage to the nerve. There is a
physiologic transection, which is often accompanied by
defective axonal transport. This type of nerve injury frequently recovers by 6 to 8 weeks on average. Stimulating
distal to a nerve lesion at about 1 week postinjury results
in no action potentials if Wallerian degeneration occurs
(Greenberg, p. 532; Youmans, pp. 3825- 3826).

247
Q

Match the questions with the most likely type of
nerve injury, using each letterhead either once, more than
once, or not at all.
A. Neurotmesis
B. A.’wnotmesis
C. Neurapraxia
D. A and B
E. None of the above

A

A. Neurotmesis
B. A.’wnotmesis
C. Neurapraxia
D. A and B
E. None of the above

With neurotmesis, there is complete anatomic transection of the nerve accompanied by
Wallerian degeneration. These injuries almost always require
surgical repair, although the timing remains unclear. For
sharply divided nerves, immediate repair appears to be ideal,
whereas for bluntly injured nerves, some advocate waiting
approximately 3 to 4 weeks prior to intervening surgically to
allow for better proximal and distal nerve stump delineation.
With a:mnotmesis, there is a loss of axonal continuity, but
the soma remains continuous. Wallerian degeneration also
occurs with this type of insult. With neurapraxia, conduction
ceases without structural damage to the nerve. There is a
physiologic transection, which is often accompanied by
defective axonal transport. This type of nerve injury frequently recovers by 6 to 8 weeks on average. Stimulating
distal to a nerve lesion at about 1 week postinjury results
in no action potentials if Wallerian degeneration occurs
(Greenberg, p. 532; Youmans, pp. 3825- 3826).

248
Q

Match the questions with the most likely type of
nerve injury, using each letterhead either once, more than
once, or not at all.
A. Neurotmesis
B. A.’wnotmesis
C. Neurapraxia
D. A and B
E. None of the above

A

A. Neurotmesis
B. A.’wnotmesis
C. Neurapraxia
**D. A and B **
E. None of the above

With neurotmesis, there is complete anatomic transection of the nerve accompanied by
Wallerian degeneration. These injuries almost always require
surgical repair, although the timing remains unclear. For
sharply divided nerves, immediate repair appears to be ideal,
whereas for bluntly injured nerves, some advocate waiting
approximately 3 to 4 weeks prior to intervening surgically to
allow for better proximal and distal nerve stump delineation.
With a:mnotmesis, there is a loss of axonal continuity, but
the soma remains continuous. Wallerian degeneration also
occurs with this type of insult. With neurapraxia, conduction
ceases without structural damage to the nerve. There is a
physiologic transection, which is often accompanied by
defective axonal transport. This type of nerve injury frequently recovers by 6 to 8 weeks on average. Stimulating
distal to a nerve lesion at about 1 week postinjury results
in no action potentials if Wallerian degeneration occurs
(Greenberg, p. 532; Youmans, pp. 3825- 3826).

249
Q

Match the questions with the most likely type of
nerve injury, using each letterhead either once, more than
once, or not at all.
A. Neurotmesis
B. A.’wnotmesis
C. Neurapraxia
D. A and B
E. None of the above

A

**A. Neurotmesis **
B. A.’wnotmesis
C. Neurapraxia
D. A and B
E. None of the above

With neurotmesis, there is complete anatomic transection of the nerve accompanied by
Wallerian degeneration. These injuries almost always require
surgical repair, although the timing remains unclear. For
sharply divided nerves, immediate repair appears to be ideal,
whereas for bluntly injured nerves, some advocate waiting
approximately 3 to 4 weeks prior to intervening surgically to
allow for better proximal and distal nerve stump delineation.
With a:mnotmesis, there is a loss of axonal continuity, but
the soma remains continuous. Wallerian degeneration also
occurs with this type of insult. With neurapraxia, conduction
ceases without structural damage to the nerve. There is a
physiologic transection, which is often accompanied by
defective axonal transport. This type of nerve injury frequently recovers by 6 to 8 weeks on average. Stimulating
distal to a nerve lesion at about 1 week postinjury results
in no action potentials if Wallerian degeneration occurs
(Greenberg, p. 532; Youmans, pp. 3825- 3826).

250
Q

A 67-year-old male underwent an anterior cervical
fusion in the remote past and presents with a 3-month history of new-onset neck pain and deltoid wealmess on the
right. His sagittal T2-weighted .MRJ is depicted on the next
page (Figure 8.2S0Q). What is shown on this sagittal MRJ
scan that may account for this new clinical picture?
A. Pseudoarthrosis
B. Osteomyelitis
C. End-fusion degenerative changes
D. Spondylolysis
E. Subsidence

A

A. Pseudoarthrosis
B. Osteomyelitis
**C. End-fusion degenerative changes **
D. Spondylolysis
E. Subsidence

The acquisition of bony spinal fusion increases
motion and stress at adjacent Illation segments, which can
accelerate degenerative changes. This process is further
enhanced by failure to repair sagittal balance during surgery.
Although not the ideal study to evaluate bony fUSion, this
MRl does not demonstrate pseudoarthrosis at the prior fusion site, subsidence, basilar invagination , or osteomyelitis.
It shows end-fusion degenerative changes and loss of the
normal lordotic curve of the cervical spine, which may be
contributing to this patients’ new clinical problems (Benzel,
p.130).

251
Q

Figure 8.251-8.257Q depicts the right internal
auditory canal and surrounding petrous temporal bone .
l'latch the following anatomic structures with the appropriate letterhead, using each answer only once.

Superior division of vestibular nerve

A

B

This diagram
shows the structures that traverse the internal auditory
canal (lAC). The facial nerve (A) and nervus intermedius are
separated from the superior vestibular nerve by a vertical
crest of bone known as Bill’s bar (F). Bill’s bar arises from the
transverse crest (G) within the lateral aspect of the lAC.
The cochlear nerve (C) and inferior vestibular nerve enter
the lAC inferior to the transverse crest, as depicted in the
diagram (Will<ins, pp. 1063-1071, 1101- 1114).

252
Q

Figure 8.251-8.257Q depicts the right internal
auditory canal and surrounding petrous temporal bone .
l'latch the following anatomic structures with the appropriate letterhead, using each answer only once.

Cochlear nerve

A

C

This diagram
shows the structures that traverse the internal auditory
canal (lAC). The facial nerve (A) and nervus intermedius are
separated from the superior vestibular nerve by a vertical
crest of bone known as Bill’s bar (F). Bill’s bar arises from the
transverse crest (G) within the lateral aspect of the lAC.
The cochlear nerve (C) and inferior vestibular nerve enter
the lAC inferior to the transverse crest, as depicted in the
diagram (Will<ins, pp. 1063-1071, 1101- 1114).

253
Q

Figure 8.251-8.257Q depicts the right internal
auditory canal and surrounding petrous temporal bone .
l'latch the following anatomic structures with the appropriate letterhead, using each answer only once.

Inferior division of vestibular nerve

A

D

This diagram
shows the structures that traverse the internal auditory
canal (lAC). The facial nerve (A) and nervus intermedius are
separated from the superior vestibular nerve by a vertical
crest of bone known as Bill’s bar (F). Bill’s bar arises from the
transverse crest (G) within the lateral aspect of the lAC.
The cochlear nerve (C) and inferior vestibular nerve enter
the lAC inferior to the transverse crest, as depicted in the
diagram (Will<ins, pp. 1063-1071, 1101- 1114).

254
Q

Figure 8.251-8.257Q depicts the right internal
auditory canal and surrounding petrous temporal bone .
l'latch the following anatomic structures with the appropriate letterhead, using each answer only once.

Facial nerve

A

A

This diagram
shows the structures that traverse the internal auditory
canal (lAC). The facial nerve (A) and nervus intermedius are
separated from the superior vestibular nerve by a vertical
crest of bone known as Bill’s bar (F). Bill’s bar arises from the
transverse crest (G) within the lateral aspect of the lAC.
The cochlear nerve (C) and inferior vestibular nerve enter
the lAC inferior to the transverse crest, as depicted in the
diagram (Will<ins, pp. 1063-1071, 1101- 1114).

255
Q

Figure 8.251-8.257Q depicts the right internal
auditory canal and surrounding petrous temporal bone .
l'latch the following anatomic structures with the appropriate letterhead, using each answer only once.

Nervous intermedius

A

E

This diagram
shows the structures that traverse the internal auditory
canal (lAC). The facial nerve (A) and nervus intermedius are
separated from the superior vestibular nerve by a vertical
crest of bone known as Bill’s bar (F). Bill’s bar arises from the
transverse crest (G) within the lateral aspect of the lAC.
The cochlear nerve (C) and inferior vestibular nerve enter
the lAC inferior to the transverse crest, as depicted in the
diagram (Will<ins, pp. 1063-1071, 1101- 1114).

256
Q

Figure 8.251-8.257Q depicts the right internal
auditory canal and surrounding petrous temporal bone .
l'latch the following anatomic structures with the appropriate letterhead, using each answer only once.

Bill’s bar

A

F

This diagram
shows the structures that traverse the internal auditory
canal (lAC). The facial nerve (A) and nervus intermedius are
separated from the superior vestibular nerve by a vertical
crest of bone known as Bill’s bar (F). Bill’s bar arises from the
transverse crest (G) within the lateral aspect of the lAC.
The cochlear nerve (C) and inferior vestibular nerve enter
the lAC inferior to the transverse crest, as depicted in the
diagram (Will<ins, pp. 1063-1071, 1101- 1114).

257
Q

Figure 8.251-8.257Q depicts the right internal
auditory canal and surrounding petrous temporal bone .
l'latch the following anatomic structures with the appropriate letterhead, using each answer only once.

Transverse crest

A

G

This diagram
shows the structures that traverse the internal auditory
canal (lAC). The facial nerve (A) and nervus intermedius are
separated from the superior vestibular nerve by a vertical
crest of bone known as Bill’s bar (F). Bill’s bar arises from the
transverse crest (G) within the lateral aspect of the lAC.
The cochlear nerve (C) and inferior vestibular nerve enter
the lAC inferior to the transverse crest, as depicted in the
diagram (Will<ins, pp. 1063-1071, 1101- 1114).

258
Q

A 5-year-old male is referred to your office for
evaluation of urinary incontinence and a sacral dimple. The
dimple was explored shortly after birth and was amputated
after it was found to track to the level of the lumbodorsal
fascia. On examination, notable findings include a smaller
left foot with a high plantar arch, a small scar overlying
the coccyx, hypoactive deep tendon reflexes in the legs,
and mild scoliosis. His J’I’IRI is most consistent with a
lipomyelomeningocele with a tethered cord.

‘Vhat process of embryologic development is usually
disrupted to produce this abnormality?
A. Disjunction
B. Migration
C. J’vIyelination
D. Cleavage
E. Transverse segmentation

A

**A. Disjunction **
B. Migration
C. J’vIyelination
D. Cleavage
E. Transverse segmentation

Three forms of lipomyelomeningocele
have been described that have clinical relevance: dorsal,
caudal (terminal), and transitional. They are believed to
arise from faulty disjunction of the neuroectoderm from
the overlying ectodenn, which leaves gaps between these
two developing layers. Subsequently, mesenchymal cells are
believed to ingress through these defective areas into the
central canal, where they are induced to form a bulk of
lipomatous tissue, which subsequently prevents fusion or
complete closure of the neural tube.

259
Q

A 5-year-old male is referred to your office for
evaluation of urinary incontinence and a sacral dimple. The
dimple was explored shortly after birth and was amputated
after it was found to track to the level of the lumbodorsal
fascia. On examination, notable findings include a smaller
left foot with a high plantar arch, a small scar overlying
the coccyx, hypoactive deep tendon reflexes in the legs,
and mild scoliosis. His J’I’IRI is most consistent with a
lipomyelomeningocele with a tethered cord.
258. ‘Vhat process of embryologic development is usually
disrupted to produce this abnormality?

Vhere are the dorsal roots usually located in relation to
the fatty stalk in patients with dorsallipomyelomeningocele?
A. Posterior
B. Dorsola teral
C. Ven trola teral
D. Dorsomedial
E. Rostral

A

A. Posterior
B. Dorsola teral
**C. Ven trola teral **
D. Dorsomedial
E. Rostral

Three forms of lipomyelomeningocele
have been described that have clinical relevance: dorsal,
caudal (terminal), and transitional. They are believed to
arise from faulty disjunction of the neuroectoderm from
the overlying ectodenn, which leaves gaps between these
two developing layers. Subsequently, mesenchymal cells are
believed to ingress through these defective areas into the
central canal, where they are induced to form a bulk of
lipomatous tissue, which subsequently prevents fusion or
complete closure of the neural tube.

260
Q

A 5-year-old male is referred to your office for
evaluation of urinary incontinence and a sacral dimple. The
dimple was explored shortly after birth and was amputated
after it was found to track to the level of the lumbodorsal
fascia. On examination, notable findings include a smaller
left foot with a high plantar arch, a small scar overlying
the coccyx, hypoactive deep tendon reflexes in the legs,
and mild scoliosis. His J’I’IRI is most consistent with a
lipomyelomeningocele with a tethered cord.
258. ‘Vhat process of embryologic development is usually
disrupted to produce this abnormality?

Dorsal lipomas may be safely resected by staying in
what relationship to the dorsal root entry zone?
A. Ventromedial
B. Dorsolateral
C. Dorsomedial
D. Rostral
E. Ventrolateral

A

A. Ventromedial
B. Dorsolateral
**C. Dorsomedial **
D. Rostral
E. Ventrolateral

Three forms of lipomyelomeningocele
have been described that have clinical relevance: dorsal,
caudal (terminal), and transitional. They are believed to
arise from faulty disjunction of the neuroectoderm from
the overlying ectodenn, which leaves gaps between these
two developing layers. Subsequently, mesenchymal cells are
believed to ingress through these defective areas into the
central canal, where they are induced to form a bulk of
lipomatous tissue, which subsequently prevents fusion or
complete closure of the neural tube.

261
Q

The primary bones that make up the nasal septum
include the
1. Vomer
2. Nasal
3. Ethmoid
4. Frontal

A. 1,2, and 3 are correct
B. 1 and 3 are correct
C. 2 and 4 are correct
D. Only 4 is correct
E. All of the above

A

A. 1,2, and 3 are correct
**B. 1 and 3 are correct **
C. 2 and 4 are correct
D. Only 4 is correct
E. All of the above

The ethmoid bone (vertical plate) and vomer contribute to the anterosuperior and posteroinferior portions
of the nasal septum, respectively. The nasal bones and nasal
crests of the frontal bones mal{e smaller contributions
(Moore, p. 945)

262
Q

What is the most common organism that causes viral
(aseptic) meningitis ?
A. Arbovirus
B. Myxovirus
C. Ente rovirus
D. Arenavirus
E. Togavirus

A

A. Arbovirus
B. Myxovirus
**C. Ente rovirus **
D. Arenavirus
E. Togavirus

Viral infections of the CNS can result in meningitis,
ventriculitis, encephalitis, and myelitis. CSF in patients
with viral syndromes of the CNS reveals increased pressure,
lymphocytic pleocytosis, mild elevations in protein, and
normal glucose levels. Viral (aseptic) meningitis usually
peaks in the summer and fall seasons, whereas bacterial
meningitis is more common during the winter. The most
cOlllmon causes of viral meningitis are the enteroviruses, but
togaviruses are also frequent pathogens. Encephalitis often
results from infections with herpes simplex virus, mumps, or
arboviruses (Merritt, pp. 134-138).

263
Q

What stage(s) of sleep is associated with night terrors ?
A. Stage 1
B. Stage 3
C. Stage 4
D. REM
E. Band C

A

A. Stage 1
B. Stage 3
C. Stage 4
D. REM
**E. Band C **

Night terrors most commonly occur during stages 3
and 4 of deep sleep. During a night terror, the child appears
extremely frightened and agitated and, although seemingly
awake , is actually in deep sleep (stage 3 or 4) and is difficult
to rouse. Upon awakening, the child has no apparent
memory of the event. Although concerning to parents, night

264
Q

A 77-year-old male presents to your office with a
4-month history of unilateral epistaxis and nasal discharge.
His MRI is depicted below (Figure 8.264Q). Histopathologic
analysis of the tumor revealed un i form small cells with round
nuclei, scant cytoplasm, a prominent reticular core, and
scattered Homer-Wright rosettes. Immunohistochemistry
was positive for neuron-specific enolase and S-100 but was
cytokeratin, CD20, and CD79a negative. What is the most
likely diagnosis?
A. Lymphoma
B. Squamous cell carcinoma
C. Esthesioneuroblastoma
D. Adenocarcinoma
E. Rl1abdomyosarcoma

A

A. Lymphoma
B. Squamous cell carcinoma
**C. Esthesioneuroblastoma **
D. Adenocarcinoma
E. Rl1abdomyosarcoma

The clinical history of this patient and destructive
MRI appearance of tIllS lesion are highly suggestive of a
malignant neoplasm involVing the paranasal sinuses. The
histologic and immunohistochemical markers are most
consistent with esthesioneuroblastoma (Kaye and Laws,
pp. 885- 889).

265
Q

An abducens palsy associated with an ipsilateral
Horner’s syndrome suggests a lesion in what location ?
A. Pon tine tegmentulll
B. Dorelio’s canal
C. Prepontine cistern
D. Cave rnous sinus
E. Region of the ligament of GrUber

A

A. Pon tine tegmentulll
B. Dorelio’s canal
C. Prepontine cistern
**D. Cave rnous sinus **
E. Region of the ligament of GrUber

Sympathetic nervous system fibers (traveling with
the internal carotid artery) and cranial nerve VI are in close
proximity in the cavernous sinus. Tumors or other lesions
in this location may produce oculosympathetic paralysis
(without anhidrosis) (Brazis, p. 265).

266
Q

The principal premotor structure concerned with
voluntary vertical sacca des is the
A. Paramedian pontine reticular formation (PPRF)
B. Rostral interstitial nucleus of the medial longitudinal
fasciculus (MLF)
C. Rostral interstitial nucleus of Cajal
D. Nucleus of Roller
E. Nucleus of Collier

A

A. Paramedian pontine reticular formation (PPRF)
**B. Rostral interstitial nucleus of the medial longitudinal
fasciculus (MLF) **
C. Rostral interstitial nucleus of Cajal
D. Nucleus of Roller
E. Nucleus of Collier

Vertical and torsional saccadic eye movements
are generated by the rostral interstitial nucleus of the
MLF (rLMLF), located in the prerubral field of the ventral
diencephalomesencephalic junction, rostral to the tractus
retroflexus and ventral to the nucleus of Darkschewitsch.
Although cells involved with downward saccades are different from the ones involved with upward saccades, they
are dispersed throughout the riMLF without separation of
upward and downward pools (Brazis, pp. 199- 200).

267
Q

The observation of various adverse effects during deep brain stimulation (DES) often characterizes the
extent of current spread to adjacent structures. For each
stimulation-induced adverse effect, match the associated
direction of current spread, using each answer once, more
than once, or not at all

Flushing and perspiration after STN DES
A. Anterior
B. Posterior
C. Medial
D. Lateral
E. Anterior and lateral
F. Inferior and medial
G. Posterior and medial
H. None of the above

A

**A. Anterior **
B. Posterior
C. Medial
D. Lateral
E. Anterior and lateral
F. Inferior and medial
G. Posterior and medial
H. None of the above

268
Q

The observation of various adverse effects during deep brain stimulation (DES) often characterizes the
extent of current spread to adjacent structures. For each
stimulation-induced adverse effect, match the associated
direction of current spread, using each answer once, more
than once, or not at all

Dysarthria after STN DES
A. Anterior
B. Posterior
C. Medial
D. Lateral
E. Anterior and lateral
F. Inferior and medial
G. Posterior and medial
H. None of the above

A

A. Anterior
B. Posterior
C. Medial
**D. Lateral **
E. Anterior and lateral
F. Inferior and medial
G. Posterior and medial
H. None of the above

269
Q

The observation of various adverse effects during deep brain stimulation (DES) often characterizes the
extent of current spread to adjacent structures. For each
stimulation-induced adverse effect, match the associated
direction of current spread, using each answer once, more
than once, or not at all

Paresthesias after GPi DES
A. Anterior
B. Posterior
C. Medial
D. Lateral
E. Anterior and lateral
F. Inferior and medial
G. Posterior and medial
H. None of the above

A

A. Anterior
B. Posterior
C. Medial
D. Lateral
E. Anterior and lateral
F. Inferior and medial
**G. Posterior and medial **
H. None of the above

270
Q

The observation of various adverse effects during deep brain stimulation (DES) often characterizes the
extent of current spread to adjacent structures. For each
stimulation-induced adverse effect, match the associated
direction of current spread, using each answer once, more
than once, or not at all

Photopsia and nausea after Gpi DES
A. Anterior
B. Posterior
C. Medial
D. Lateral
E. Anterior and lateral
F. Inferior and medial
G. Posterior and medial
H. None of the above

A

**A. Anterior **
B. Posterior
C. Medial
D. Lateral
E. Anterior and lateral
F. Inferior and medial
G. Posterior and medial
H. None of the above

271
Q

The observation of various adverse effects during deep brain stimulation (DES) often characterizes the
extent of current spread to adjacent structures. For each
stimulation-induced adverse effect, match the associated
direction of current spread, using each answer once, more
than once, or not at all

Diplopia after STN DES

A

A. Anterior
B. Posterior
C. Medial
D. Lateral
E. Anterior and lateral
**F. Inferior and medial **
G. Posterior and medial
H. None of the above

272
Q

The observation of various adverse effects during deep brain stimulation (DES) often characterizes the
extent of current spread to adjacent structures. For each
stimulation-induced adverse effect, match the associated
direction of current spread, using each answer once, more
than once, or not at all

Tonic contraction after Vim DBS
A. Anterior
B. Posterior
C. Medial
D. Lateral
E. Anterior and lateral
F. Inferior and medial
G. Posterior and medial
H. None of the above

A

A. Anterior
B. Posterior
C. Medial
**D. Lateral **
E. Anterior and lateral
F. Inferior and medial
G. Posterior and medial
H. None of the above

273
Q

The observation of various adverse effects during deep brain stimulation (DES) often characterizes the
extent of current spread to adjacent structures. For each
stimulation-induced adverse effect, match the associated
direction of current spread, using each answer once, more
than once, or not at all

Blepharospasm after STN DES
A. Anterior
B. Posterior
C. Medial
D. Lateral
E. Anterior and lateral
F. Inferior and medial
G. Posterior and medial
H. None of the above

A

A. Anterior
B. Posterior
C. Medial
D. Lateral
E. Anterior and lateral
F. Inferior and medial
G. Posterior and medial
H. None of the above

274
Q

The observation of various adverse effects during deep brain stimulation (DES) often characterizes the
extent of current spread to adjacent structures. For each
stimulation-induced adverse effect, match the associated
direction of current spread, using each answer once, more
than once, or not at all

Dysequilibrium and gait ataxia (without limb at<uia)
after STN DES
A. Anterior
B. Posterior
C. Medial
D. Lateral
E. Anterior and lateral
F. Inferior and medial
G. Posterior and medial
H. None of the above

A

A. Anterior
B. Posterior
**C. Medial **
D. Lateral
E. Anterior and lateral
F. Inferior and medial
G. Posterior and medial
H. None of the above

275
Q

The observation of various adverse effects during deep brain stimulation (DES) often characterizes the
extent of current spread to adjacent structures. For each
stimulation-induced adverse effect, match the associated
direction of current spread, using each answer once, more
than once, or not at all

Tonic contraction after STN DBS
A. Anterior
B. Posterior
C. Medial
D. Lateral
E. Anterior and lateral
F. Inferior and medial
G. Posterior and medial
H. None of the above

A

A. Anterior
B. Posterior
C. Medial
D. Lateral
**E. Anterior and lateral **
F. Inferior and medial
G. Posterior and medial
H. None of the above

276
Q

The observation of various adverse effects during deep brain stimulation (DES) often characterizes the
extent of current spread to adjacent structures. For each
stimulation-induced adverse effect, match the associated
direction of current spread, using each answer once, more
than once, or not at all

Paresthesias after Vim DBS
A. Anterior
B. Posterior
C. Medial
D. Lateral
E. Anterior and lateral
F. Inferior and medial
G. Posterior and medial
H. None of the above

A

A. Anterior
**B. Posterior **
C. Medial
D. Lateral
E. Anterior and lateral
F. Inferior and medial
G. Posterior and medial
H. None of the above

277
Q

What histological finding is pathognomonic for denervation followed by reinnervation of muscle fibers?
A. Atrophic muscle fibers
B. Angular fibers
C. Eosinophilic infiltration
D. Target cells
E. Type-specific grouping of muscle fibers

A

A. Atrophic muscle fibers
B. Angular fibers
C. Eosinophilic infiltration
D. Target cells
E. Type-specific grouping of muscle fibers

Denervation typically causes atrophy of muscle
fibers, which eventually become angulated, as well as the formation of distinctive fibers with three unique zones often
referred to as “target cells. “ Reinnervation is characterized
by type-specific grouping of fibers, which is in contrast to the mixed, “ch eckerboard” pattern of types 1 and 2 skeletal
fibers and the atrophic, angulated cells of denervated fibers.
Eosinophils in muscle cells are often seen with parasitic
infections (Merritt, p. 736).

278
Q

The photomicrograph depicted below (Figure 8.278Q)
is most consistent with what diagnosis ?
A. Tuberculosis
B. Chronic infarct
C. Multiple sclerosis
D. Histoplasmosis
E. Pilocytic astrocytoma

A

**A. Tuberculosis **
B. Chronic infarct
C. Multiple sclerosis
D. Histoplasmosis
E. Pilocytic astrocytoma

Within this tuberculoma, note the prominent region
of caseating necrosis (left), in which no cellular detail can be
ascertained. A peripheral rim of lymphocytes and a fibrous
capsule surround the granuloma, and an occasional histiocyte is observed (Ellison, pp. 339- 342).

279
Q

The gross specimen depicted below (Figure 8.279Q)
reveals ?
A. Neurocysticercosis
B. Third ventricular tumor
C. Metastatic disease
D. Toxoplasmosis
E. Multiple sclerosis

A

A. Neurocysticercosis
B. Third ventricular tumor
**C. Metastatic disease **
D. Toxoplasmosis
E. Multiple sclerosis

Note the multiple lesions at the gray-white junction,
which is most consistent with metastatic disease (Ellison,
pp.743-750).

280
Q

Which of the following muscles abduct the vocal cords?
1. Lateral cricoarytenoid
2. Transverse arytenoid
3. Cricothyroid
4. Posterior cricoarytenoid

A. 1,2, and 3 are correct
B. 1 and 3 are correct
C. 2 and 4 are correct
D. Only 4 is correct
E. All of the above

A

A. 1,2, and 3 are correct
B. 1 and 3 are correct
C. 2 and 4 are correct
**D. Only 4 is correct **
E. All of the above

The right and left posterior cricoarytenoid muscles
abduct the vocal cords (Moore, p. 1060).

281
Q

Installation of cold water in the right ear elicits what
type of response?

A. Left beating nystagmus
B. Eye deviation to the left
C. Right beating nystagmus
D. Convergence retraction nystagmus
E. WEEINO response

A

**A. Left beating nystagmus **
B. Eye deviation to the left
C. Right beating nystagmus
D. Convergence retraction nystagmus
E. WEEINO response

Injecting cold water into the right ear causes the
endolymph to move away from the ampulla of the horizontal
canal. This causes a decreased tone of input into the left
abducens nucleus, which results in slow conjugate eye movements toward the right due to right lateral rectus and left
medial rectus contraction. Subsequently, there will be compensatory fast eye movements (left-beating nystagmus) to
the left. Warm water injected into the right ear will produce
the opposite response. The mnemonic COWS (cold opposite;
warm same) helps define the fast phase of eye movements
during caloric testing (Kline, pp. 57- 59).

282
Q

Match the following, using each answer only
once.

No effect on Horner’s pupil

A. Dilute (1/8%) pilocarpine
B. 1% Hydroxyamphetamine
C. 10% Cocaine
D. Atropine
E. Dilute epinephrine (1:1000)

A

A. Dilute (1/8%) pilocarpine
B. 1% Hydroxyamphetamine
**C. 10% Cocaine **
D. Atropine
E. Dilute epinephrine (1:1000)

283
Q

Match the following, using each answer only
once.

Dilates second-order Horner’s pupil
A. Dilute (1/8%) pilocarpine
B. 1% Hydroxyamphetamine
C. 10% Cocaine
D. Atropine
E. Dilute epinephrine (1:1000)

A

A. Dilute (1/8%) pilocarpine
**B. 1% Hydroxyamphetamine **
C. 10% Cocaine
D. Atropine
E. Dilute epinephrine (1:1000)

284
Q

Match the following, using each answer only
once.

Dilates third-order Horner’s pupil
A. Dilute (1/8%) pilocarpine
B. 1% Hydroxyamphetamine
C. 10% Cocaine
D. Atropine
E. Dilute epinephrine (1:1000)

A

A. Dilute (1/8%) pilocarpine
B. 1% Hydroxyamphetamine
C. 10% Cocaine
**D. Atropine **
E. Dilute epinephrine (1:1000)

285
Q

Match the following, using each answer only
once.

Dilates normal pupil
A. Dilute (1/8%) pilocarpine
B. 1% Hydroxyamphetamine
C. 10% Cocaine
D. Atropine
E. Dilute epinephrine (1:1000)

A

A. Dilute (1/8%) pilocarpine
B. 1% Hydroxyamphetamine
C. 10% Cocaine
**D. Atropine **
E. Dilute epinephrine (1:1000)

286
Q

Match the following, using each answer only
once.

Constricts AcHe’s pupil
A. Dilute (1/8%) pilocarpine
B. 1% Hydroxyamphetamine
C. 10% Cocaine
D. Atropine
E. Dilute epinephrine (1:1000)

A

**A. Dilute (1/8%) pilocarpine **
B. 1% Hydroxyamphetamine
C. 10% Cocaine
D. Atropine
E. Dilute epinephrine (1:1000)

287
Q

Match the following questions with the appropriate diagnostic technique, using each answer once, more than
once, or not at all.

Based on reflection of sound waves off of moving targets
A. B-mocle ultrasonography
B. Doppler ultrasonography
C. Both
D. None of the above

A

A. B-mocle ultrasonography
B. Doppler ultrasonography
C. Both
D. None of the above

Brightness modulation
ultrasonography (B mode), based on reflection of sound
waves off tissue interfaces, is better used to view anatomic
detail and can be used to measure blood vessel diameter and
evaluate plaques. It typically shows the carotid system as a
pulsatile luminal structure, with a thin echogenic line representing the intimal surface. The basic principle underlying
this mode of imaging is the variable impedance that different body tissues naturally possess. Doppler shift ultrasonography is based on the reflection of sound waves off
moving targets such as red blood cells. It is better for evaluating flow dynamics such as blood velocity, which typically
increases in segments of stenosis. Although the combination
of B-mode and Doppler ultrasonography improved the
ability to localize the source of the reflected signal when
it was involved with complicated blood velocity patterns
(vasospasm, atherosclerosis), the signal was still quit difficult
to interpret. lI’IOl’e recently, through the use of fast Fourier
transformations, the complex signal was separated into a
number of single-frequency components, which made it
easier to interpret. These technologies were later integrated,

288
Q

Match the following questions with the appropriate diagnostic technique, using each answer once, more than
once, or not at all.

Best used to evaluate flow dynamics
A. B-mocle ultrasonography
B. Doppler ultrasonography
C. Both
D. None of the above

A

A. B-mocle ultrasonography
**B. Doppler ultrasonography **
C. Both
D. None of the above

Brightness modulation
ultrasonography (B mode), based on reflection of sound
waves off tissue interfaces, is better used to view anatomic
detail and can be used to measure blood vessel diameter and
evaluate plaques. It typically shows the carotid system as a
pulsatile luminal structure, with a thin echogenic line representing the intimal surface. The basic principle underlying
this mode of imaging is the variable impedance that different body tissues naturally possess. Doppler shift ultrasonography is based on the reflection of sound waves off
moving targets such as red blood cells. It is better for evaluating flow dynamics such as blood velocity, which typically
increases in segments of stenosis. Although the combination
of B-mode and Doppler ultrasonography improved the
ability to localize the source of the reflected signal when
it was involved with complicated blood velocity patterns
(vasospasm, atherosclerosis), the signal was still quit difficult
to interpret. lI’IOl’e recently, through the use of fast Fourier
transformations, the complex signal was separated into a
number of single-frequency components, which made it
easier to interpret. These technologies were later integrated,

289
Q

Match the following questions with the appropriate diagnostic technique, using each answer once, more than
once, or not at all.

Based on reflec tion of sound waves off of tissue interfaces that are stationary
A. B-mocle ultrasonography
B. Doppler ultrasonography
C. Both
D. None of the above

A

**A. B-mocle ultrasonography **
B. Doppler ultrasonography
C. Both
D. None of the above

Brightness modulation
ultrasonography (B mode), based on reflection of sound
waves off tissue interfaces, is better used to view anatomic
detail and can be used to measure blood vessel diameter and
evaluate plaques. It typically shows the carotid system as a
pulsatile luminal structure, with a thin echogenic line representing the intimal surface. The basic principle underlying
this mode of imaging is the variable impedance that different body tissues naturally possess. Doppler shift ultrasonography is based on the reflection of sound waves off
moving targets such as red blood cells. It is better for evaluating flow dynamics such as blood velocity, which typically
increases in segments of stenosis. Although the combination
of B-mode and Doppler ultrasonography improved the
ability to localize the source of the reflected signal when
it was involved with complicated blood velocity patterns
(vasospasm, atherosclerosis), the signal was still quit difficult
to interpret. lI’IOl’e recently, through the use of fast Fourier
transformations, the complex signal was separated into a
number of single-frequency components, which made it
easier to interpret. These technologies were later integrated,

290
Q

Match the following questions with the appropriate diagnostic technique, using each answer once, more than
once, or not at all.

Typically shows the carotid system as a pulsatile
luminal structure with a thin echogenic line representing
the intimal surface
A. B-mocle ultrasonography
B. Doppler ultrasonography
C. Both
D. None of the above

A

A. B-mocle ultrasonography
B. Doppler ultrasonography
C. Both
D. None of the above

Brightness modulation
ultrasonography (B mode), based on reflection of sound
waves off tissue interfaces, is better used to view anatomic
detail and can be used to measure blood vessel diameter and
evaluate plaques. It typically shows the carotid system as a
pulsatile luminal structure, with a thin echogenic line representing the intimal surface. The basic principle underlying
this mode of imaging is the variable impedance that different body tissues naturally possess. Doppler shift ultrasonography is based on the reflection of sound waves off
moving targets such as red blood cells. It is better for evaluating flow dynamics such as blood velocity, which typically
increases in segments of stenosis. Although the combination
of B-mode and Doppler ultrasonography improved the
ability to localize the source of the reflected signal when
it was involved with complicated blood velocity patterns
(vasospasm, atherosclerosis), the signal was still quit difficult
to interpret. lI’IOl’e recently, through the use of fast Fourier
transformations, the complex signal was separated into a
number of single-frequency components, which made it
easier to interpret. These technologies were later integrated,

291
Q

Match the following questions with the appropriate diagnostic technique, using each answer once, more than
once, or not at all.

Can sample blood flow within a vessel while simultaneously displaying vessel wall anatomy
A. B-mocle ultrasonography
B. Doppler ultrasonography
C. Both
D. None of the above

A

A. B-mocle ultrasonography
B. Doppler ultrasonography
C. Both
D. None of the above

Brightness modulation
ultrasonography (B mode), based on reflection of sound
waves off tissue interfaces, is better used to view anatomic
detail and can be used to measure blood vessel diameter and
evaluate plaques. It typically shows the carotid system as a
pulsatile luminal structure, with a thin echogenic line representing the intimal surface. The basic principle underlying
this mode of imaging is the variable impedance that different body tissues naturally possess. Doppler shift ultrasonography is based on the reflection of sound waves off
moving targets such as red blood cells. It is better for evaluating flow dynamics such as blood velocity, which typically
increases in segments of stenosis. Although the combination
of B-mode and Doppler ultrasonography improved the
ability to localize the source of the reflected signal when
it was involved with complicated blood velocity patterns
(vasospasm, atherosclerosis), the signal was still quit difficult
to interpret. lI’IOl’e recently, through the use of fast Fourier
transformations, the complex signal was separated into a
number of single-frequency components, which made it
easier to interpret. These technologies were later integrated,

292
Q

What gene is abnormal in spinobulbar muscular
atrophy (Kennedy’s disease)?
A. Androgen receptor gene
B. Dystrophin gene
c. Superoxide dismutase gene
D. TGF-~ receptor gene
E. Mitochondrial complex 1 gene

A

**A. Androgen receptor gene **
B. Dystrophin gene
c. Superoxide dismutase gene
D. TGF-~ receptor gene
E. Mitochondrial complex 1 gene

Spinobulbar muscular atrophy (SBMA), or Kennedy’s
disease, is a trinucleotide repeat disorder (CAG) that
involves mutations of the androgen receptor gene, which is
located on the X chromosome. It is an X-linked recessive
disorder that typically affects adult males in the third decade
of life and results in lower motor neuron degeneration only.
Symptoms are similar to ALS and include dysarthria, dysphagia, limb wealmess, hyporeflexia , and tongue fasciculations. 5MBA progresses much more slowly than ALS, and
patients may survive for extended periods after initial symptom onset (Merritt, pp. 709-710).

293
Q

All of the following are true about levodopa EXCEPT?
A. Vitamin B” decreases effective dose of levodopa
B. MAO inhibitors can exaggerate the central dopamine
effects by decreasing metabolism of dopamine
C. Can cause orthostatic hypotension
D. Carbidopa can increase the effectiveness of levodopa
by inhibiting a peripheral decarboxylase inhibitor
E. Antipsychotics increase the efficacy oflevodopa

A

A. Vitamin B” decreases effective dose of levodopa
B. MAO inhibitors can exaggerate the central dopamine
effects by decreasing metabolism of dopamine
C. Can cause orthostatic hypotension
D. Carbidopa can increase the effectiveness of levodopa
by inhibiting a peripheral decarboxylase inhibitor
**E. Antipsychotics increase the efficacy oflevodopa **

Antipsychotics decrease the efficacy of levodopa by
inhibiting D2 receptors. Dose-related, reversible side effects
include nausea , vomiting, orthostatic hypotension, dyskinesias, restlessness, amdety, athetosis, insomnia, hallucinations,
mania , nightmares, and dystonia (Merritt, pp. 689- 691).

294
Q

Ramsay I-Iunt syndrome usually involves which cranial
nerve?
A. Central seventh cranial nerve
B. Lower seventh cranial nerve
C. Olfactory nerve
D. Mandibular division of the trigeminal nerve
E. Trochlear nerve

A

A. Central seventh cranial nerve
**B. Lower seventh cranial nerve **
C. Olfactory nerve
D. Mandibular division of the trigeminal nerve
E. Trochlear nerve

The Ramsay Hunt syndrome is associated with
herpetic infections of the geniculate ganglion. Herpetic
eruptions may appear in the pinna , EAC, and possibly on
the tympanic membrane. It is most often associated with a
lowe r seventh cranial nerve palsy, although there may also
be decreased hearing, tinnitus, andlor vertigo from involvement of CN VIII (Greenberg, p. 381).

295
Q

The positive symptoms of schizophrenia are a result
of increased dopamine activity in which of the following
pathways?
A. Nigrostriatal
B. Tuberoinfundibular
C. Mesocortical
D. Mesolimbic
E. Tuberolimbic

A

A. Nigrostriatal
B. Tuberoinfundibular
C. Mesocortical
**D. Mesolimbic **
E. Tuberolimbic

The positive symptoms of schizophrenia, such as
hallucinations, are the direct result of dopamine overactivity
in the mesolimbic pathway. Most an tipsychotics work by
blocking the dopamine receptors in this pathway and often
work well for the positive symptoms of schizophrenia. The
nega tive symptoms (flat affect, decreased motivation) are
the result of decreased dopamine in the mesocortical system
and are often not improved with the commencement of
antipsychotics (Kandel, p. 1203).

296
Q

\That muscles are involved with elevation of the
mandible or mouth closure?
1. Medial pterygoid muscle
2. Masseter muscle
3. Temporalis muscle
4. Buccinator muscle

A. 1,2, and 3 are correct
B. 1 and3 are correct
C. 2 and 4 are correct
D. Only 4 is correct
E. All of the above

A

**A. 1,2, and 3 are correct **
B. 1 and3 are correct
C. 2 and 4 are correct
D. Only 4 is correct
E. All of the above

The masseter, temporalis, and medial pterygoid
muscles are the prime elevators of the mandible , whereas th e
buccinator muscle compresses the cheek (April, pp. 496,
504).

297
Q

What is depicted on the CT scan below (Figure 8.297Q)?
A. Evidence of posterior interhemispheric blood after
head trauma
B. Empty delta sign of superior sagittal thrombosis associated with a cystic tumor
C. Pineocytoma
D. A hyperdense MCA sign
E. A left posterior temporal infarct

A

A. Evidence of posterior interhemispheric blood after
head trauma
**B. Empty delta sign of superior sagittal thrombosis associated with a cystic tumor **
C. Pineocytoma
D. A hyperdense MCA sign
E. A left posterior temporal infarct

Note the empty delta sign, representing an occluded
superior sagittal sinus. This abnormality may be associated
with pregnancy, dehydration , infections, hypercoagulable
states, and tumors (as in this case) (Merritt, pp. 269-
271).

298
Q

All of the following are true of hyperosmolar hyperglycemic nonl(etotic diabetic ketoacidosis syndrome (I-IHt\TS)
EXCEPT?
A. Most patients are mildly hypernatremic
B. HHNS is often accompanied by severe prerenal
azotemia
C. I-IHNS is associated with an anion gap
D. J'Iost patients are hypokalemic
E. Insulin is always administered in the initial management period

A

A. Most patients are mildly hypernatremic
B. HHNS is often accompanied by severe prerenal
azotemia
C. I-IHNS is associated with an anion gap
D. J'Iost patients are hypokalemic
E. Insulin is always administered in the initial management period

Insulin mayor may not be used initially in patients
with HHNS. Serum glucose levels may drop precipitously
with fluid replacement alone. Insulin is required for patients
who are acidotic , hyperkalemic, or in renal failure. If insulin
is used, it must be administered in a low-dose regimen as a
continuous infusion. To avoid overcorrection, glucose levels
must be monitored frequently and the insulin drip stopped
once glucose levels fall below 300 mg/dL or so. At this point
5% dextrose should be added to the infusion. Treatment of
I-IH1\TS focuses primarily 011 replacing fluid losses, which may
be up to 9 to 12 L in some cases, as well as correcting the

299
Q

Match the medication with the potential toxicity,
using each answer only once.

Raynaud’s phenomenon, pulmonary fibrosis
A. Methotrexate
B. Vincristine
C. Bleomycin
D. None of the abovE

A

A. Methotrexate
B. Vincristine
**C. Bleomycin **
D. None of the abovE

Methotrexate may cause leukoencephalopathy,
myelosuppression, nephrotoxicity, and mucositis; if administered intrathecally, it can cause arachnoiditis. Bleomycin
is associated with Raynaud’s phenomenon, myelosuppression, and pulmonary fibrosis, while vincristine can calise a
peripheralnel1l’opathy (Kay and Laws, pp. 381-383).

300
Q

Match the medication with the potential toxicity,
using each answer only once.

Leukoencephalopathy
A. Methotrexate
B. Vincristine
C. Bleomycin
D. None of the abovE

A

**A. Methotrexate **
B. Vincristine
C. Bleomycin
D. None of the abovE

Methotrexate may cause leukoencephalopathy,
myelosuppression, nephrotoxicity, and mucositis; if administered intrathecally, it can cause arachnoiditis. Bleomycin
is associated with Raynaud’s phenomenon, myelosuppression, and pulmonary fibrosis, while vincristine can calise a
peripheralnel1l’opathy (Kay and Laws, pp. 381-383).

301
Q
A